CH 39: Dysrhythmias Flashcards
A 72-year-old male patient is admitted to the ICU with complaints of dizziness, palpitations, and near-syncope. His vital signs are BP 88/60 mmHg, HR 38 bpm, RR 16, and SpO₂ 96% on room air. His ECG shows sinus bradycardia with frequent pauses. He is confused and diaphoretic.
What is the priority nursing intervention?
A. Administer atropine 0.5 mg IV push
B. Prepare for synchronized cardioversion
C. Apply transcutaneous pacing pads
D. Observe the patient and continue monitoring
A. Administer atropine 0.5 mg IV push
Rationale: Atropine is the first-line treatment for symptomatic bradycardia, as it blocks vagal stimulation and increases HR. Since the patient is symptomatic with hypotension and altered mental status, immediate intervention is required to improve cardiac output and perfusion. If atropine is ineffective, transcutaneous pacing or dopamine/epinephrine infusion may be necessary.
A patient in the emergency department presents with sudden onset of palpitations and dizziness. The ECG reveals supraventricular tachycardia (SVT) with a HR of 180 bpm. The patient is alert but diaphoretic with a BP of 90/60 mmHg. What is the next best nursing action?
A. Perform carotid sinus massage
B. Prepare for immediate defibrillation
C. Administer adenosine 6 mg IV push
D. Start a metoprolol IV infusion
C. Administer adenosine 6 mg IV push
Rationale: Adenosine is the first-line treatment for stable SVT, as it temporarily blocks AV node conduction and can terminate the rhythm. It is given rapidly through a large-bore IV followed by a saline flush. If ineffective, a second dose of 12 mg can be given. If the patient becomes unstable, synchronized cardioversion is required.
A nurse is caring for a patient with atrial fibrillation (AF) who has a rapid ventricular response (RVR) at 160 bpm. Which of the following interventions should the nurse anticipate? (Select all that apply.)
A. Administer diltiazem IV
B. Prepare for immediate defibrillation
C. Monitor for signs of stroke
D. Administer anticoagulation therapy
E. Perform carotid sinus massage
A. Administer diltiazem IV
C. Monitor for signs of stroke
D. Administer anticoagulation therapy
Rationale: Diltiazem (a calcium channel blocker) slows AV node conduction, reducing HR in AF with RVR. Atrial fibrillation increases stroke risk due to clot formation in the atria, requiring anticoagulation. Stroke symptoms should be monitored closely. Defibrillation is not appropriate for AF unless it is pulseless. Carotid massage is contraindicated in AF as it does not terminate the rhythm.
A patient is admitted with a diagnosis of ventricular fibrillation (VF). What is the priority nursing action?
A. Administer epinephrine 1 mg IV
B. Perform synchronized cardioversion
C. Initiate chest compressions and defibrillate
D. Administer amiodarone IV bolus
C. Initiate chest compressions and defibrillate
Rationale: VF is a life-threatening dysrhythmia requiring immediate CPR and defibrillation. Defibrillation is the most effective treatment, as it delivers an electrical shock to depolarize the heart and allow the SA node to resume normal rhythm. Epinephrine and antiarrhythmic drugs (e.g., amiodarone) are given after the first defibrillation attempt if VF persists.
A nurse is assessing a patient with a history of myocardial infarction (MI) who suddenly reports feeling lightheaded. The monitor shows a wide QRS complex rhythm with no P waves and a HR of 40 bpm. The nurse checks the pulse and finds it weak and irregular.
What is the priority intervention?
A. Administer amiodarone IV
B. Initiate transcutaneous pacing
C. Give epinephrine 1 mg IV
D. Obtain a 12-lead ECG
B. Initiate transcutaneous pacing
Rationale: A wide QRS complex, bradycardia, and absence of P waves suggest a ventricular escape rhythm, indicating severe conduction system failure. Transcutaneous pacing is the priority intervention to maintain adequate cardiac output until a permanent pacemaker can be placed. Medications like amiodarone are used for tachyarrhythmias, not bradyarrhythmias.
A nurse is educating a patient with a newly implanted permanent pacemaker. Which instructions should be included? (Select all that apply.)
A. Avoid MRI scans
B. Report dizziness or palpitations to the provider
C. Avoid lifting the affected arm above the shoulder for 4-6 weeks
D. Stand directly in front of microwave ovens when using them
E. Carry a pacemaker identification card at all times
A. Avoid MRI scans
B. Report dizziness or palpitations to the provider
C. Avoid lifting the affected arm above the shoulder for 4-6 weeks
E. Carry a pacemaker identification card at all times
Rationale: MRI can interfere with pacemaker function and should be avoided. Symptoms like dizziness or palpitations may indicate pacemaker malfunction and should be reported. Patients should not lift the affected arm to prevent lead displacement. A pacemaker ID card provides essential information in emergencies. Microwave ovens are generally safe with modern pacemakers.
A patient with chronic heart failure is admitted with complaints of shortness of breath and fatigue. The ECG reveals frequent premature ventricular contractions (PVCs). What should the nurse do first?
A. Assess potassium and magnesium levels
B. Prepare for immediate defibrillation
C. Administer atropine IV push
D. Instruct the patient to bear down (Valsalva maneuver)
A. Assess potassium and magnesium levels
Rationale: Electrolyte imbalances, especially hypokalemia and hypomagnesemia, are common causes of PVCs. Identifying and correcting these imbalances can help prevent more serious dysrhythmias, such as ventricular tachycardia or fibrillation. Immediate defibrillation is not required for isolated PVCs. Atropine is used for bradycardia, not PVCs. The Valsalva maneuver is useful for supraventricular tachycardias, not ventricular ectopy.
A 72-year-old patient presents with syncope. The ECG shows a regular rhythm with a ventricular rate of 30 bpm, absent P waves, and wide QRS complexes. The patient is confused and hypotensive. Which intervention should the nurse anticipate first?
A. Administer atropine 0.5 mg IV push
B. Prepare for transcutaneous pacing
C. Give adenosine 6 mg IV push
D. Observe and continue monitoring
B. Prepare for transcutaneous pacing
Rationale: The patient is experiencing a symptomatic idioventricular rhythm, where the ventricles act as the pacemaker due to SA and AV node failure. This results in a slow rate (20-40 bpm), wide QRS, and absent P waves. Since the patient is hypotensive and confused, immediate transcutaneous pacing is necessary to maintain cardiac output. Atropine is typically ineffective in idioventricular rhythms because the issue is below the AV node.
A nurse is educating a student on the characteristics of the AV node in the cardiac conduction system. Which statements by the student indicate correct understanding? (Select all that apply.)
A. “The AV node slows down impulses before transmitting them to the ventricles.”
B. “The AV node can serve as a backup pacemaker if the SA node fails.”
C. “The AV node is located in the right atrium near the SA node.”
D. “The AV node directly stimulates the Purkinje fibers.”
E. “The AV node normally fires at a rate of 40-60 bpm.”
A. “The AV node slows down impulses before transmitting them to the ventricles.”
B. “The AV node can serve as a backup pacemaker if the SA node fails.”
E. “The AV node normally fires at a rate of 40-60 bpm.”
Rationale: The AV node slows electrical conduction to allow for atrial contraction before ventricular depolarization. It also serves as a backup pacemaker (40-60 bpm) if the SA node fails. However, it is located at the junction of the atria and ventricles, not near the SA node. The AV node does not directly stimulate the Purkinje fibers—it transmits impulses to the bundle of His, which then branches into the Purkinje system.
A patient in the ICU develops sudden-onset palpitations and shortness of breath. The ECG shows a regular narrow-complex tachycardia at 190 bpm with no visible P waves. The patient is alert, BP is 112/78 mmHg, and SpO₂ is 97%. What is the priority nursing intervention?
A. Prepare for synchronized cardioversion
B. Instruct the patient to perform the Valsalva maneuver
C. Administer adenosine 6 mg IV push
D. Defibrillate immediately
B. Instruct the patient to perform the Valsalva maneuver
Rationale: The patient is experiencing stable supraventricular tachycardia (SVT), a narrow-complex tachycardia originating above the ventricles. The first-line treatment is vagal maneuvers, such as the Valsalva maneuver, to stimulate the vagus nerve and slow AV conduction. If unsuccessful, adenosine IV is the next step. Synchronized cardioversion is reserved for unstable patients, and defibrillation is inappropriate for SVT.
A patient in the telemetry unit reports dizziness and chest tightness. The ECG shows an irregular ventricular rhythm with absent P waves and fibrillatory waves in place of a normal baseline. The ventricular rate is 160 bpm. Which intervention should the nurse anticipate?
A. Immediate defibrillation
B. Carotid sinus massage
C. Administration of epinephrine IV push
D. Administration of beta-blockers or calcium channel blockers
D. Administration of beta-blockers or calcium channel blockers
Rationale: The ECG findings suggest atrial fibrillation (AF) with rapid ventricular response (RVR). Rate control is the priority in stable AF, achieved with beta-blockers (e.g., metoprolol) or calcium channel blockers (e.g., diltiazem). If the patient becomes unstable (e.g., hypotension, altered mental status), synchronized cardioversion would be needed. Defibrillation is only for pulseless AF or VF, carotid massage is ineffective, and epinephrine would worsen the tachycardia.
A nurse is caring for a patient with a new onset of atrial flutter. The ECG shows a sawtooth pattern of atrial waves with a ventricular rate of 130 bpm. The patient is hemodynamically stable. What is the best initial treatment?
A. Synchronized cardioversion
B. Administration of digoxin IV
C. Vagal maneuvers and rate control medications
D. Immediate transcutaneous pacing
C. Vagal maneuvers and rate control medications
Rationale: Atrial flutter is treated similarly to atrial fibrillation. In stable patients, the priority is controlling the ventricular rate using beta-blockers, calcium channel blockers, or digoxin. Vagal maneuvers can also slow AV conduction. Cardioversion is only used if the patient is unstable or symptomatic despite rate control. Pacing is unnecessary unless there is severe bradycardia or AV block.
A nurse is caring for a patient who suddenly develops sinus bradycardia with a heart rate of 38 bpm. The patient is pale, diaphoretic, and reports dizziness. Which intervention should the nurse anticipate first?
A. Prepare for immediate transcutaneous pacing
B. Instruct the patient to perform the Valsalva maneuver
C. Administer atropine 0.5 mg IV push
D. Give adenosine 6 mg IV push
C. Administer atropine 0.5 mg IV push
Rationale: The patient is experiencing symptomatic sinus bradycardia, likely due to excessive vagal stimulation. Atropine is the first-line treatment, as it blocks vagal stimulation and increases heart rate by enhancing SA node firing. If atropine is ineffective, transcutaneous pacing or other medications (e.g., dopamine, epinephrine) may be needed. Valsalva maneuver would worsen bradycardia, and adenosine is used to treat SVT, not bradycardia.
A nurse is teaching a student about the effects of the autonomic nervous system on the heart. Which statements by the student indicate correct understanding? (Select all that apply.)
A. “The sympathetic nervous system increases heart rate and contractility.”
B. “The vagus nerve is part of the parasympathetic nervous system.”
C. “Stimulation of the vagus nerve slows SA node firing and AV node conduction.”
D. “Sympathetic stimulation decreases cardiac output by reducing stroke volume.”
E. “Parasympathetic stimulation causes vasoconstriction of the coronary arteries.”
A. “The sympathetic nervous system increases heart rate and contractility.”
B. “The vagus nerve is part of the parasympathetic nervous system.”
C. “Stimulation of the vagus nerve slows SA node firing and AV node conduction.”
Rationale: The sympathetic nervous system increases heart rate, conduction velocity, and contractility, while the parasympathetic system (via the vagus nerve) slows SA node firing and AV node conduction, reducing heart rate. However, sympathetic stimulation increases cardiac output by increasing stroke volume, and parasympathetic stimulation does not cause vasoconstriction of coronary arteries.
A patient with neurogenic shock has a heart rate of 42 bpm, BP of 78/42 mmHg, and cool extremities. The provider explains that the bradycardia is due to loss of sympathetic nervous system function. Which nursing intervention is the priority?
A. Administer beta-blockers to increase cardiac contractility
B. Place the patient in high Fowler’s position to improve circulation
C. Prepare for administration of atropine and possible pacing
D. Initiate Valsalva maneuvers to stimulate the vagus nerve
C. Prepare for administration of atropine and possible pacing
Rationale: Neurogenic shock results from a loss of sympathetic tone, leading to unopposed parasympathetic stimulation (bradycardia and hypotension). Atropine blocks vagal effects, increasing heart rate, and pacing may be required if bradycardia persists. Beta-blockers would worsen bradycardia, high Fowler’s position could further reduce BP, and Valsalva maneuvers would exacerbate the problem.
A nurse is reviewing heart rate regulation in a patient receiving a beta-adrenergic blocker (e.g., metoprolol). The nurse knows this medication affects heart rate by:
A. Increasing vagus nerve activity, slowing conduction through the AV node
B. Enhancing SA node automaticity to maintain a stable rhythm
C. Inhibiting vagus nerve stimulation to increase cardiac output
D. Blocking sympathetic stimulation of the SA node, decreasing heart rate
D. Blocking sympathetic stimulation of the SA node, decreasing heart rate
Rationale: Beta-blockers inhibit sympathetic stimulation, decreasing SA node firing, slowing AV conduction, and reducing heart rate. They do not directly increase vagal activity, enhance SA node automaticity, or inhibit vagus nerve function. This reduces myocardial oxygen demand and is commonly used in hypertension, heart failure, and tachydysrhythmias.
A patient with a history of fainting undergoes a carotid sinus massage to evaluate possible causes. Shortly after, the patient becomes pale and diaphoretic with a heart rate of 32 bpm. Which physiological response best explains this change?
A. Increased sympathetic stimulation, leading to excessive vasoconstriction
B. Increased myocardial contractility, reducing stroke volume
C. Parasympathetic overstimulation, slowing SA node firing
D. A sudden increase in cardiac output, leading to reflex bradycardia
C. Parasympathetic overstimulation, slowing SA node firing
Rationale: Carotid sinus massage stimulates the vagus nerve, activating the parasympathetic nervous system. This slows SA node firing and AV node conduction, leading to bradycardia. This maneuver is sometimes used to terminate SVT but can cause excessive vagal stimulation, resulting in profound bradycardia and syncope.
A patient in the ED is experiencing a panic attack with a heart rate of 136 bpm. The nurse explains that this tachycardia is primarily caused by:
A. Increased vagal nerve stimulation from anxiety
B. Excessive parasympathetic tone, leading to reflex tachycardia
C. Direct suppression of the SA node
D. Overstimulation of the sympathetic nervous system
D. Overstimulation of the sympathetic nervous system
Rationale: During a panic attack, the sympathetic nervous system is activated, leading to increased SA node firing, faster AV node conduction, and higher cardiac contractility, resulting in tachycardia. The vagus nerve slows, rather than increases, heart rate. Parasympathetic activity is not the cause of tachycardia, and the SA node is not being suppressed.
A nurse is analyzing an ECG and observes a heart rate of 40 bpm, regular P waves before every QRS, and a PR interval of 0.18 seconds. How should the nurse interpret this rhythm?
A. Sinus bradycardia
B. Junctional rhythm
C. Complete heart block
D. Idioventricular rhythm
A. Sinus bradycardia
Rationale: Sinus bradycardia is defined as a sinus rhythm with a heart rate below 60 bpm, a normal PR interval (0.12–0.20 sec), and a P wave before every QRS complex. A junctional rhythm typically has an absent or inverted P wave, complete heart block has no relationship between P waves and QRS complexes, and idioventricular rhythm originates from the ventricles with a wide QRS.
A nurse is preparing to obtain a 12-lead ECG. Which statements indicate correct understanding? (Select all that apply.)
A. “Limb leads measure electrical activity in the horizontal plane.”
B. “The precordial leads (V1–V6) provide views of the heart in the horizontal plane.”
C. “Lead II is commonly used for continuous monitoring.”
D. “Improper lead placement can lead to misdiagnosis of dysrhythmias or ischemia.”
E. “The aVR lead is placed on the left leg.”
B. “The precordial leads (V1–V6) provide views of the heart in the horizontal plane.”
C. “Lead II is commonly used for continuous monitoring.”
D. “Improper lead placement can lead to misdiagnosis of dysrhythmias or ischemia.”
Rationale:
* B is correct: Precordial leads (V1–V6) measure electrical forces in the horizontal plane.
* C is correct: Lead II is commonly used in continuous ECG monitoring due to its ability to display clear P waves.
* D is correct: Incorrect lead placement can lead to false interpretations of ECG findings.
A nurse is interpreting an ECG strip and notices an irregular rhythm with no identifiable P waves and a chaotic baseline. What dysrhythmia does this indicate?
A. Sinus tachycardia
B. Ventricular fibrillation
C. Atrial fibrillation
D. Second-degree AV block
C. Atrial fibrillation
Rationale: Atrial fibrillation is characterized by an irregular rhythm, no identifiable P waves, and fibrillatory waves (chaotic baseline). Ventricular fibrillation also has a chaotic rhythm but no QRS complexes. Sinus tachycardia has a fast, regular rhythm with P waves, and AV block has P waves with blocked conduction.
A patient’s ECG shows a wide QRS complex (>0.12 sec) and a heart rate of 130 bpm. Which rhythm is most likely present?
A. Atrial fibrillation
B. Supraventricular tachycardia (SVT)
C. Ventricular tachycardia (VT)
D. Sinus tachycardia
C. Ventricular tachycardia (VT)
Rationale: Ventricular tachycardia is a wide-complex tachycardia (QRS > 0.12 sec) with a heart rate >100 bpm. SVT has a narrow QRS, sinus tachycardia originates from the SA node, and atrial fibrillation has an irregular rhythm.
Which of the following ECG findings are consistent with myocardial ischemia? (Select all that apply.)
A. ST segment depression
B. T wave inversion
C. Wide QRS complex
D. Prolonged PR interval
E. ST segment elevation
A. ST segment depression
B. T wave inversion
E. ST segment elevation
Rationale:
* ST depression and T wave inversion suggest ischemia (A, B are correct).
* ST elevation suggests acute injury, such as myocardial infarction (E is correct).
* Wide QRS complex is associated with ventricular conduction delays, not ischemia (C is incorrect).
* A prolonged PR interval indicates AV block, not ischemia (D is incorrect).
Which patient is at highest risk for developing an artifact on ECG monitoring?
A. A patient with hyperkalemia
B. A patient who is diaphoretic
C. A patient with sinus tachycardia
D. A patient who is receiving a calcium channel blocker
B. A patient who is diaphoretic
Rationale: Artifact is often caused by loose electrodes due to sweat, movement, or poor skin contact. Hyperkalemia affects ECG waveforms, but not artifact. Sinus tachycardia and calcium channel blockers do not cause artifact.
A nurse notes that a patient’s ECG shows a prolonged QT interval. Which electrolyte imbalance is most likely responsible?
A. Hypercalcemia
B. Hypernatremia
C. Hypocalcemia
D. Hypokalemia
C. Hypocalcemia
Rationale: Hypocalcemia prolongs the QT interval, increasing the risk of ventricular dysrhythmias. Hypercalcemia shortens the QT interval. Sodium imbalances do not significantly affect QT duration.
A nurse is reviewing ECG findings. Which abnormalities indicate a possible life-threatening arrhythmia? (Select all that apply.)
A. Third-degree AV block
B. Ventricular fibrillation
C. Sinus tachycardia
D. Asystole
E. Atrial flutter
A. Third-degree AV block
B. Ventricular fibrillation
D. Asystole
Rationale:
* A is correct: Third-degree AV block can lead to complete loss of ventricular function.
* B is correct: Ventricular fibrillation is fatal without immediate intervention.
* D is correct: Asystole is the absence of electrical activity, requiring immediate resuscitation.
* C is incorrect: Sinus tachycardia is not typically life-threatening unless associated with shock.
* E is incorrect: Atrial flutter is abnormal but not immediately life-threatening.
A patient presents with a heart rate of 180 bpm, a narrow QRS complex, and no visible P waves. The nurse suspects which dysrhythmia?
A. Atrial flutter
B. Ventricular tachycardia
C. Supraventricular tachycardia (SVT)
D. Sinus tachycardia
C. Supraventricular tachycardia (SVT)
Rationale: SVT is a rapid rhythm (>150 bpm) with a narrow QRS and no distinct P waves. Atrial flutter has a “sawtooth” pattern, VT has a wide QRS, and sinus tachycardia has identifiable P waves.
Which interventions should the nurse prepare for a stable patient with SVT? (Select all that apply.)
A. Vagal maneuvers
B. Administering adenosine
C. Defibrillation
D. Synchronized cardioversion
E. Preparing for CPR
A. Vagal maneuvers
B. Administering adenosine
D. Synchronized cardioversion
Rationale:
* A is correct: Vagal maneuvers (e.g., bearing down) may terminate SVT.
* B is correct: Adenosine is the drug of choice for SVT.
* D is correct: If unstable, synchronized cardioversion is considered.
* C and E are incorrect: Defibrillation is for pulseless VT or VF, and CPR is for cardiac arrest.
A nurse is caring for a patient with atrial flutter. Which ECG characteristic is most consistent with this rhythm?
A. Wide, bizarre QRS complexes
B. Irregularly irregular rhythm
C. Sawtooth-shaped P waves
D. Prolonged PR interval
C. Sawtooth-shaped P waves
Rationale: Atrial flutter is characterized by sawtooth-like P waves (flutter waves). Atrial fibrillation has an irregularly irregular rhythm. Wide QRS suggests VT, and a prolonged PR interval indicates AV block.
Which ECG findings are associated with hyperkalemia? (Select all that apply.)
A. Peaked T waves
B. Prolonged PR interval
C. Widened QRS complex
D. ST segment depression
E. U waves
A. Peaked T waves
B. Prolonged PR interval
C. Widened QRS complex
Rationale:
* A is correct: Hyperkalemia causes peaked T waves.
* B is correct: PR interval prolongation occurs as potassium levels rise.
* C is correct: Severe hyperkalemia leads to QRS widening.
* D and E are incorrect: ST depression and U waves are seen in hypokalemia.
A patient with ventricular fibrillation (VF) is unresponsive. What is the priority intervention?
A. Synchronized cardioversion
B. Administering epinephrine first
C. Vagal maneuvers
D. Immediate defibrillation
D. Immediate defibrillation
Rationale: Defibrillation is the first-line treatment for VF. Synchronized cardioversion is used for unstable SVT or atrial flutter, not VF. Epinephrine is given after the first shock.
A nurse is monitoring a patient with a first-degree AV block. What ECG change is expected?
A. A prolonged PR interval
B. A widened QRS complex
C. More P waves than QRS complexes
D. Irregular rhythm
A. A prolonged PR interval
Rationale: First-degree AV block is characterized by a PR interval >0.20 sec. Widened QRS suggests bundle branch block, and more P waves than QRS complexes are seen in second- and third-degree AV blocks.
Which are possible causes of premature ventricular contractions (PVCs)? (Select all that apply.)
A. Hypokalemia
B. Hypoxia
C. Hypercalcemia
D. Caffeine intake
E. Myocardial ischemia
A. Hypokalemia
B. Hypoxia
D. Caffeine intake
E. Myocardial ischemia
Rationale: PVCs are caused by irritability in the ventricles.
* Hypokalemia (A), hypoxia (B), caffeine (D), and ischemia (E) can trigger PVCs.
* Hypercalcemia does not cause PVCs.
A nurse is assessing a patient with a third-degree AV block. Which symptom is most concerning?
A. Syncope
B. Fatigue
C. Occasional palpitations
D. BP of 130/80 mmHg
A. Syncope
Rationale: Third-degree AV block can cause severe bradycardia and decreased cardiac output, leading to syncope (fainting).
A patient with atrial fibrillation is at highest risk for which complication?
A. Stroke
B. Hypertension
C. Heart failure
D. Pulmonary embolism
A. Stroke
Rationale: Atrial fibrillation increases the risk of stroke due to blood stasis and clot formation in the atria.
Which interventions are appropriate for a patient with ventricular tachycardia (VT) with a pulse? (Select all that apply.)
A. Defibrillation
B. Amiodarone administration
C. Synchronized cardioversion
D. CPR
E. Vagal maneuvers
B. Amiodarone administration
C. Synchronized cardioversion
E. Vagal maneuvers
Rationale: Stable VT is treated with vagal maneuvers, antiarrhythmic drugs (amiodarone), and cardioversion if necessary. Defibrillation and CPR are for pulseless VT.
A patient in pulseless electrical activity (PEA) requires which intervention?
A. Defibrillation
B. Epinephrine and CPR
C. Synchronized cardioversion
D. Vagal maneuvers
B. Epinephrine and CPR
Rationale: PEA is treated with CPR and epinephrine—defibrillation is not indicated.
Which dysrhythmia requires immediate defibrillation?
A. Atrial flutter
B. Sinus bradycardia
C. Ventricular fibrillation
D. First-degree AV block
C. Ventricular fibrillation
Rationale: VF is a life-threatening arrhythmia requiring immediate defibrillation
A nurse notices a QT interval of 0.52 seconds on a patient’s ECG. Which dysrhythmia is the patient at risk for developing?
A. Atrial fibrillation
B. Sinus tachycardia
C. Torsades de Pointes
D. Junctional rhythm
C. Torsades de Pointes
Rationale: A prolonged QT interval (>0.50 sec) increases the risk of Torsades de Pointes, a type of polymorphic VT that can deteriorate into ventricular fibrillation.
A patient is in asystole. What is the priority nursing action?
A. High-quality CPR
B. Immediate defibrillation
C. Synchronized cardioversion
D. Administering adenosine
A. High-quality CPR
Rationale: Asystole is a non-shockable rhythm. The priority is high-quality CPR and administering epinephrine. Defibrillation and cardioversion are ineffective for asystole.
Which dysrhythmia is most likely to cause sudden cardiac death?
A. Atrial flutter
B. Sinus bradycardia
C. Ventricular fibrillation
D. First-degree AV block
C. Ventricular fibrillation
Rationale: Ventricular fibrillation causes immediate loss of cardiac output, leading to sudden cardiac death if not treated with defibrillation.
A patient with chronic atrial fibrillation is prescribed warfarin. What is the primary reason for this medication?
A. To convert the heart to a normal sinus rhythm
B. To increase heart rate
C. To prevent bradycardia
D. To prevent thromboembolism
D. To prevent thromboembolism
Rationale: Atrial fibrillation increases the risk of stroke due to blood pooling in the atria, which can form clots. Warfarin prevents thromboembolism.
Which electrolyte imbalance is most likely to cause Torsades de Pointes?
A. Hypomagnesemia
B. Hyperkalemia
C. Hypocalcemia
D. Hypernatremia
A. Hypomagnesemia
Rationale: Low magnesium levels prolong the QT interval, increasing the risk of Torsades de Pointes.
A 68-year-old patient with a history of hypertension and coronary artery disease is admitted for chest pain and placed on telemetry monitoring. The telemetry technician notifies the nurse of ST-segment depression on the patient’s ECG.
What is the priority nursing intervention?
A. Administer adenosine IV push
B. Immediately initiate defibrillation
C. Assess the patient for chest pain and hemodynamic stability
D. Turn off the telemetry monitor to avoid false alarms
C. Assess the patient for chest pain and hemodynamic stability
Rationale: ST-segment depression may indicate myocardial ischemia. The nurse should first assess the patient’s symptoms, blood pressure, and oxygen saturation before notifying the provider.
A nurse is teaching a new graduate about centralized telemetry monitoring. Which of the following statements are accurate about this system? (Select all that apply.)
A. A telemetry technician or nurse continuously observes ECG rhythms from a central location
B. This system requires patients to be connected to a bedside monitor at all times
C. It can rapidly identify dysrhythmias, ischemia, and infarction
D. The nurse must still assess the patient even if the monitor appears normal
E. This system does not provide real-time monitoring and only stores data
A. A telemetry technician or nurse continuously observes ECG rhythms from a central location
C. It can rapidly identify dysrhythmias, ischemia, and infarction
D. The nurse must still assess the patient even if the monitor appears normal
Rationale: Centralized telemetry monitoring involves continuous ECG observation from a central location. It helps detect dysrhythmias, ischemia, and infarction. However, nurses must still physically assess the patient, as some conditions may not be immediately reflected on telemetry.
A 56-year-old patient admitted for syncope is placed on remote telemetry monitoring. The alarm sounds for ventricular tachycardia, but when the nurse assesses the patient, the patient is awake, alert, and asymptomatic.
What is the most appropriate action?
A. Immediately call a code blue
B. Check the telemetry leads and monitor for artifact
C. Start CPR and prepare for defibrillation
D. Give IV amiodarone as prescribed
B. Check the telemetry leads and monitor for artifact
Rationale: Telemetry monitors can display false alarms due to lead misplacement, muscle activity, or electrical interference. If the patient is asymptomatic, first assess the leads and the monitor before taking further action.
Which statement about remote telemetry monitoring is correct?
A. It requires continuous direct observation by a telemetry technician
B. It does not provide alarms for ischemia or infarction
C. It requires constant ECG lead adjustments for accurate monitoring
D. It automatically stores and detects abnormal heart rhythms
D. It automatically stores and detects abnormal heart rhythms
Rationale: Remote telemetry monitoring does not require continuous observation. It can detect and store abnormal heart rhythms, ischemia, and infarction, allowing nurses to review data as needed.
A patient on telemetry monitoring develops frequent artifacts. What actions should the nurse take? (Select all that apply.)
A. Check if the leads are properly placed and secured
B. Reapply electrodes if the conductive gel has dried
C. Increase the telemetry alarm sensitivity to avoid missing real arrhythmias
D. Assess the patient for excessive movement or shivering
E. Ignore the artifact if the patient is asymptomatic
A. Check if the leads are properly placed and secured
B. Reapply electrodes if the conductive gel has dried
D. Assess the patient for excessive movement or shivering
Rationale: Artifacts can result from loose leads, dry gel, and patient movement. The nurse should secure leads, replace electrodes if needed, and assess for excessive motion. Increasing alarm sensitivity is unnecessary and ignoring artifact may lead to missed true dysrhythmias.
A nurse is monitoring a patient on telemetry and notices frequent premature ventricular contractions (PVCs). The patient reports feeling dizzy and lightheaded.
Which intervention should the nurse implement first?
A. Check the patient’s blood pressure and oxygen saturation
B. Increase the telemetry alarm sensitivity
C. Silence the alarm to reduce patient anxiety
D. Encourage the patient to take deep breaths and relax
A. Check the patient’s blood pressure and oxygen saturation
Rationale: Frequent PVCs may reduce cardiac output, leading to dizziness and hypotension. Assessing BP and oxygenation determines hemodynamic stability before notifying the provider.
A nurse is caring for a post-MI patient placed on continuous telemetry monitoring. The provider asks for a rhythm strip to assess for dysrhythmias.
How should the nurse obtain an accurate rhythm strip?
A. Print a rhythm strip from the ECG monitor and measure the intervals
B. Ask the telemetry technician to verbally report the HR and rhythm
C. Take a manual pulse instead of relying on telemetry
D. Use a 12-lead ECG instead, as rhythm strips are not reliable
A. Print a rhythm strip from the ECG monitor and measure the intervals
Rationale: A rhythm strip from the ECG monitor provides accurate data on heart rhythm, allowing the nurse to measure intervals and identify dysrhythmias.
A 72-year-old patient with a history of myocardial infarction and heart failure is placed on continuous telemetry monitoring. The nurse notes a rhythm with a ventricular rate of 35 beats per minute and no visible P waves. The patient is dizzy and hypotensive.
What is the priority nursing intervention?
A. Monitor the patient closely and reassess in 15 minutes
B. Administer IV beta-blockers as prescribed
C. Prepare for transcutaneous pacing
D. Instruct the patient to perform the Valsalva maneuver
C. Prepare for transcutaneous pacing
Rationale: A ventricular rate of 35 bpm suggests a His-Purkinje system pacemaker (20-40 bpm), indicating severe bradycardia. The patient is symptomatic with hypotension and dizziness, requiring immediate pacing to restore an adequate heart rate.
A nurse is reviewing the electrophysiologic mechanisms of dysrhythmias with a new graduate nurse. Which of the following statements are correct? (Select all that apply.)
A. The SA node normally fires at a rate of 60-100 beats per minute
B. The AV node can act as a secondary pacemaker with an intrinsic rate of 40-60 beats per minute
C. The refractory period is the time when the heart is completely recovered and fully excitable
D. An ectopic focus refers to an electrical impulse originating outside the normal conduction pathway
E. The absolute refractory period is when excitability is highest, making dysrhythmias more likely
A. The SA node normally fires at a rate of 60-100 beats per minute
B. The AV node can act as a secondary pacemaker with an intrinsic rate of 40-60 beats per minute
D. An ectopic focus refers to an electrical impulse originating outside the normal conduction pathway
Rationale: The SA node is the primary pacemaker (60-100 bpm), and the AV node can take over at 40-60 bpm if needed. An ectopic focus is an abnormal site for impulse generation. The refractory period limits excitability, with the absolute refractory phase preventing any stimulation.
A 55-year-old patient with atrial fibrillation suddenly reports palpitations and shortness of breath. The nurse observes a wide-complex tachycardia on telemetry at a rate of 180 bpm.
What is the most appropriate intervention?
A. Administer IV amiodarone as prescribed
B. Perform carotid massage to slow the heart rate
C. Immediately begin chest compressions
D. Observe the patient and document the findings
A. Administer IV amiodarone as prescribed
Rationale: A wide-complex tachycardia in a patient with atrial fibrillation could indicate ventricular tachycardia (VT). IV amiodarone is the first-line antiarrhythmic for hemodynamically stable VT.
Which statement accurately describes reentrant dysrhythmias?
A. They result from a completely blocked conduction pathway
B. They occur when an impulse reenters a previously unexcitable area that has recovered
C. They can only originate from the AV node
D. They are always benign and do not require treatment
B. They occur when an impulse reenters a previously unexcitable area that has recovered
Rationale: Reentrant dysrhythmias occur when an impulse travels through a unidirectional block and reactivates tissue that has recovered, leading to premature beats or rapid rhythms.
A patient has frequent premature ventricular contractions (PVCs). Which factors could contribute to the development of PVCs? (Select all that apply.)
A. Myocardial ischemia
B. Hypokalemia
C. Beta-blocker therapy
D. Anxiety
E. Hypermagnesemia
A. Myocardial ischemia
B. Hypokalemia
D. Anxiety
Rationale: PVCs can result from myocardial ischemia, electrolyte imbalances (e.g., hypokalemia), and increased sympathetic activity (e.g., anxiety). Beta-blockers generally reduce ectopic beats, and hypermagnesemia is not a common cause.
A 38-year-old patient is admitted for paroxysmal supraventricular tachycardia (PSVT). The nurse prepares to administer adenosine.
Which action is most important when giving this medication?
A. Administer the drug slowly over 1-2 minutes
B. Give the drug via an IV line in the hand
C. Have emergency equipment readily available
D. Encourage the patient to cough after administration
C. Have emergency equipment readily available
Rationale: Adenosine can cause transient asystole, bradycardia, or hypotension. Emergency equipment should be on hand in case resuscitation is needed.
A patient with a complete heart block (third-degree AV block) would most likely have which ECG finding?
A. Normal PR intervals with every QRS complex
B. Atrial and ventricular rhythms that are independent of each other
C. P waves consistently preceding each QRS
D. A ventricular rate of 60-100 beats per minute
B. Atrial and ventricular rhythms that are independent of each other
Rationale: In complete heart block, atrial impulses do not reach the ventricles, causing the atria and ventricles to contract independently at different rates.
Which of the following factors can cause triggered beats leading to dysrhythmias? (Select all that apply.)
A. Hypoxia
B. Electrolyte imbalances
C. Fever
D. Vagal stimulation
E. Certain medications like digoxin
A. Hypoxia
B. Electrolyte imbalances
C. Fever
E. Certain medications like digoxin
Rationale: Triggered beats can result from hypoxia, electrolyte imbalances, fever (due to increased metabolic demand), and medications like digoxin, which increase automaticity. Vagal stimulation generally slows conduction rather than triggering ectopic beats.
A 66-year-old patient with heart disease is experiencing recurrent ventricular tachycardia (VT). The provider recommends an implantable cardioverter-defibrillator (ICD).
Which explanation by the nurse is most appropriate?
A. “The ICD will continuously pace your heart at a normal rate.”
B. “The ICD will deliver a shock if a dangerous rhythm is detected.”
C. “You will need weekly maintenance on your ICD to keep it working.”
D. “You can never engage in physical activity after getting an ICD.”
B. “The ICD will deliver a shock if a dangerous rhythm is detected.”
Rationale: ICDs monitor heart rhythms and deliver shocks to correct life-threatening dysrhythmias. Routine maintenance is not weekly, and patients can resume activities after recovery.
Which conduction abnormality is most likely to cause bradycardia with dropped QRS complexes?
A. Sinus tachycardia
B. First-degree AV block
C. Second-degree AV block (Mobitz type II)
D. Atrial fibrillation
C. Second-degree AV block (Mobitz type II)
Rationale: Mobitz type II AV block involves intermittent failure of QRS conduction, leading to dropped beats and bradycardia.
A patient with ventricular fibrillation (VF) is unresponsive and pulseless. What is the priority intervention?
A. Administer IV magnesium sulfate
B. Obtain a 12-lead ECG
C. Start synchronized cardioversion
D. Defibrillate the patient immediately
D. Defibrillate the patient immediately
Rationale: VF is a life-threatening rhythm that requires immediate defibrillation to restore organized electrical activity.
Which statements about automaticity are correct? (Select all that apply.)
A. It refers to the heart’s ability to generate impulses without external stimulation
B. It is highest in the Purkinje fibers
C. It can be increased by ischemia or electrolyte imbalances
D. The SA node normally has the highest automaticity
E. Beta-blockers increase automaticity
A. It refers to the heart’s ability to generate impulses without external stimulation
C. It can be increased by ischemia or electrolyte imbalances
D. The SA node normally has the highest automaticity
Rationale: Automaticity is the heart’s ability to generate impulses. It is highest in the SA node, and conditions like ischemia can enhance it. Beta-blockers decrease automaticity.
A 45-year-old male with a history of hypertension and hyperlipidemia comes to the ED with chest pain and palpitations. His ECG shows a heart rate of 160 bpm, narrow QRS complexes, and regular rhythm. The nurse prepares to administer adenosine but notices the patient is now diaphoretic and hypotensive.
What is the priority intervention?
A. Perform synchronized cardioversion
B. Continue with adenosine administration
C. Start a calcium channel blocker infusion
D. Instruct the patient to bear down
A. Perform synchronized cardioversion
Rationale: Since the patient is now unstable (hypotensive, diaphoretic), immediate synchronized cardioversion is required to restore a normal rhythm. Adenosine is only used for stable patients.
A nurse is explaining the relative refractory period of the cardiac cycle to a student nurse. Which statement by the student requires correction?
A. “It occurs after the absolute refractory period.”
B. “A strong stimulus may generate an abnormal impulse.”
C. “This period increases the risk of dysrhythmias like ventricular tachycardia.”
D. “It is the phase where excitability is completely blocked.”
D. “It is the phase where excitability is completely blocked.”
Rationale: The absolute refractory period is when no impulses can be generated. The relative refractory period follows, during which a strong stimulus can cause an abnormal impulse, increasing the risk of dysrhythmias.
A 60-year-old male presents to the emergency department with complaints of palpitations and dizziness. The ECG shows atrial fibrillation. Which diagnostic test would be most appropriate to further assess the patient’s dysrhythmia?
A) Exercise treadmill testing
B) Holter monitoring
C) Signal-averaged ECG
D) Electrophysiologic study (EPS)
B) Holter monitoring
Rationale: Holter monitoring continuously records the ECG while the patient is ambulatory and performing daily activities. It is useful for correlating symptoms with any dysrhythmias seen on the ECG.
Which of the following is a priority when managing a patient with a symptomatic dysrhythmia in a nonmonitored setting?
A) Administering antidysrhythmic medication
B) Preparing for defibrillation
C) Determining the rhythm by heart monitoring
D) Providing supplemental oxygen
C) Determining the rhythm by heart monitoring
Rationale: In a nonmonitored setting, determining the rhythm by heart monitoring is a high priority if the patient becomes symptomatic (e.g., chest pain, syncope). This helps guide the appropriate management and treatment.
Which of the following tests can assess the effectiveness of antidysrhythmic drug therapy? (Select all that apply)
A) Holter monitoring
B) Event monitoring
C) Electrophysiologic study (EPS)
D) Signal-averaged ECG
A) Holter monitoring
B) Event monitoring
C) Electrophysiologic study (EPS)
Rationale: Holter monitoring, event monitoring, and EPS can assess the effectiveness of antidysrhythmic drug therapy. The signal-averaged ECG identifies late potentials and suggests the risk of developing serious ventricular dysrhythmias.
A patient with a history of syncope is scheduled for an electrophysiologic study (EPS). What is the primary goal of this test?
A) To monitor the patient’s heart rhythm during daily activities
B) To evaluate exercise-induced dysrhythmias
C) To identify late potentials in the ECG
D) To locate accessory pathways and determine the effectiveness of antidysrhythmic drugs
D) To locate accessory pathways and determine the effectiveness of antidysrhythmic drugs
Rationale: An EPS study can identify the causes of heart blocks, tachydysrhythmias, bradydysrhythmias, and syncope. It also locates accessory pathways and determines the effectiveness of antidysrhythmic drugs.
During an exercise treadmill test, the patient’s ECG shows significant ST-segment depression. What does this finding indicate?
A) The patient has a normal heart rhythm during exercise
B) The patient has exercise-induced dysrhythmias
C) The patient is at risk for serious ventricular dysrhythmias
D) The patient has adequate myocardial perfusion
B) The patient has exercise-induced dysrhythmias
Rationale: Significant ST-segment depression during an exercise treadmill test indicates the presence of exercise-induced dysrhythmias or ischemia, which can be evaluated for treatment.
Which of the following symptoms may indicate a dysrhythmia? (Select all that apply)
A) Chest pain
B) Syncope
C) Fever
D) Dizziness
A) Chest pain
B) Syncope
D) Dizziness
Rationale: Symptoms such as chest pain, syncope, and dizziness may indicate a dysrhythmia. Fever is not typically associated with dysrhythmias.
A patient with a Holter monitor reports experiencing palpitations while climbing stairs. What is the next appropriate step in managing this patient?
A) Review the Holter monitor recording for dysrhythmias during the reported event
B) Instruct the patient to avoid physical activity
C) Schedule an EPS
D) Prescribe a beta-blocker
A) Review the Holter monitor recording for dysrhythmias during the reported event
Rationale: Reviewing the Holter monitor recording for dysrhythmias during the reported event helps correlate the symptoms with the ECG findings and guides further management.
Which test is used to identify late potentials in the ECG and assess the risk of developing serious ventricular dysrhythmias?
A) Holter monitoring
B) Event monitoring
C) Exercise treadmill testing
D) Signal-averaged ECG
D) Signal-averaged ECG
Rationale: The signal-averaged ECG identifies late potentials, which suggest that the patient may be at risk for developing serious ventricular dysrhythmias.
What are some common causes of dysrhythmias? (Select all that apply)
A) Electrolyte imbalances
B) Medications
C) Stress
D) High altitude
A) Electrolyte imbalances
B) Medications
C) Stress
Rationale: Electrolyte imbalances, medications, and stress are common causes of dysrhythmias. High altitude is not a typical cause of dysrhythmias.
A patient is admitted for evaluation of syncope. Which test would be most appropriate to identify the cause of the syncope?
A) Holter monitoring
B) Event monitoring
C) Signal-averaged ECG
D) Electrophysiologic study (EPS)
D) Electrophysiologic study (EPS)
Rationale: An EPS study can identify the causes of syncope by evaluating the heart’s electrical conduction system and locating accessory pathways.
A patient with a history of dysrhythmias is using a smartphone app to record ECG readings. What is the primary benefit of this technology?
A) It provides real-time ECG recordings during symptoms
B) It continuously monitors the heart rhythm for 24 hours
C) It can replace the need for an EPS
D) It evaluates exercise-induced dysrhythmias
A) It provides real-time ECG recordings during symptoms
Rationale: Smartphone apps can obtain and save ECG recordings during symptoms, improving the evaluation of dysrhythmias in outpatients.
Which interventions are appropriate for a patient with a symptomatic dysrhythmia in a nonmonitored setting? (Select all that apply)
A) Activate the emergency response system (ERS)
B) Determine the rhythm by heart monitoring
C) Administer oxygen
D) Perform CPR
A) Activate the emergency response system (ERS)
B) Determine the rhythm by heart monitoring
Rationale: In a nonmonitored setting, activating the ERS and determining the rhythm by heart monitoring are high priorities for managing a patient with a symptomatic dysrhythmia. Administering oxygen and performing CPR may be necessary, but the priority is to establish the rhythm.
A patient with a newly diagnosed dysrhythmia is scheduled for Holter monitoring. What instructions should the nurse provide to the patient?
A) Avoid all physical activity during the monitoring period
B) Take all medications as prescribed without informing the healthcare provider
C) Keep a diary of activities and symptoms during the monitoring period
D) Remove the monitor during showering or bathing
C) Keep a diary of activities and symptoms during the monitoring period
Rationale: The patient should keep a diary of activities and symptoms during the Holter monitoring period to correlate events with any dysrhythmias seen on the ECG.
Which diagnostic test can locate accessory pathways and evaluate the effectiveness of antidysrhythmic drugs?
A) Holter monitoring
B) Event monitoring
C) Electrophysiologic study (EPS)
D) Exercise treadmill testing
C) Electrophysiologic study (EPS)
Rationale: An EPS study can locate accessory pathways and determine the effectiveness of antidysrhythmic drugs, providing valuable information for managing dysrhythmias.
What are the primary goals of continuous ECG monitoring during hospitalization for a patient with a dysrhythmia? (Select all that apply)
A) Assess the heart rhythm
B) Monitor changes in rhythm
C) Evaluate the patient’s clinical status
D) Provide real-time heart rate data
A) Assess the heart rhythm
B) Monitor changes in rhythm
Rationale: Continuous ECG monitoring during hospitalization aims to assess the heart rhythm, monitor changes in rhythm, and evaluate the patient’s clinical status. Providing real-time heart rate data is a benefit but not a primary goal.
Which of the following accurately describes normal sinus rhythm?
A) The HR is below 60 beats/min
B) The P wave is inverted and follows the QRS complex
C) The QRS complex has a normal shape and duration
D) The SA node fires irregularly
C) The QRS complex has a normal shape and duration
Rationale: Normal sinus rhythm refers to a rhythm that starts in the SA node at a rate of 60 to 100 beats/min and follows the normal conduction pathway. The P wave is normal, the PR interval is normal, and the QRS complex has a normal shape and duration.
What are the characteristics of sinus arrhythmia? (Select all that apply)
A) The conduction pathway is the same as in sinus rhythm
B) The SA node fires irregularly
C) The HR increases during inspiration
D) The HR decreases during exhalation
all of the choices are correct
Rationale: In sinus arrhythmia, the conduction pathway is the same as in sinus rhythm, but the SA node fires irregularly. The HR increases slightly during inspiration and decreases slightly during exhalation.
A 45-year-old female presents with a heart rate of 70 beats/min, normal P waves preceding each QRS complex, and normal PR and QRS intervals. What is the most likely diagnosis?
A) Sinus bradycardia
B) Sinus tachycardia
C) Sinus arrhythmia
D) Normal sinus rhythm
D) Normal sinus rhythm
Rationale: The patient’s heart rate is within the normal range (60 to 100 beats/min), and the ECG shows normal P waves, PR interval, and QRS complex. This indicates normal sinus rhythm.
Which rhythm is characterized by the SA node firing irregularly due to changes in intrathoracic pressure during breathing?
A) Sinus arrhythmia
B) Normal sinus rhythm
C) Sinus tachycardia
D) Atrial fibrillation
A) Sinus arrhythmia
Rationale: Sinus arrhythmia is characterized by the SA node firing irregularly, often resulting from changes in intrathoracic pressure during breathing. The heart rate increases during inspiration and decreases during exhalation.
Which features are typical of normal sinus rhythm? (Select all that apply)
A) The HR is between 60 to 100 beats/min
B) The P wave follows the QRS complex
C) The PR interval is normal
D) The QRS complex is wide and irregular
A) The HR is between 60 to 100 beats/min
C) The PR interval is normal
Rationale: Normal sinus rhythm is characterized by a heart rate between 60 to 100 beats/min, normal P waves preceding each QRS complex, and normal PR and QRS intervals.
A healthy 30-year-old male undergoes an ECG that reveals a heart rate of 80 beats/min, with the SA node firing irregularly and the heart rate increasing during inspiration and decreasing during exhalation. What is the most likely diagnosis?
A) Atrial flutter
B) Sinus arrhythmia
C) Sinus tachycardia
D) Normal sinus rhythm
B) Sinus arrhythmia
Rationale: The patient’s heart rate increases during inspiration and decreases during exhalation, indicating sinus arrhythmia. This condition is common in healthy adults.
Which statement is true about sinus arrhythmia?
A) It is a type of dysrhythmia that requires immediate medical intervention
B) It is common in healthy adults and related to breathing patterns
C) The HR is always above 100 beats/min
D) The QRS complex is wide and irregular
B) It is common in healthy adults and related to breathing patterns
Rationale: Sinus arrhythmia is common in healthy adults and is related to changes in intrathoracic pressure during breathing, causing the heart rate to increase during inspiration and decrease during exhalation.
Which ECG findings would indicate normal sinus rhythm? (Select all that apply)
A) The P wave precedes each QRS complex
B) The HR is below 60 beats/min
C) The QRS complex has a normal shape and duration
D) The PR interval is prolonged
A) The P wave precedes each QRS complex
C) The QRS complex has a normal shape and duration
Rationale: Normal sinus rhythm is indicated by P waves that precede each QRS complex, a heart rate of 60 to 100 beats/min, normal PR interval, and normal shape and duration of the QRS complex.
Which of the following is true about sinus bradycardia?
A) The SA node fires at a rate greater than 100 beats/min
B) The conduction pathway is different from that in sinus rhythm
C) The SA node fires at a rate less than 60 beats/min
D) The QRS complex is prolonged
C) The SA node fires at a rate less than 60 beats/min
Rationale: In sinus bradycardia, the conduction pathway is the same as that in sinus rhythm, but the SA node fires at a rate less than 60 beats/min.
A 65-year-old male presents with a heart rate of 55 beats/min, dizziness, and fatigue. The ECG shows normal P waves and QRS complexes, but the SA node is firing at a slower rate. What condition is this most likely indicating?
A) Sinus tachycardia
B) Atrial fibrillation
C) Sinus bradycardia
D) Ventricular tachycardia
C) Sinus bradycardia
Rationale: The patient has a heart rate of less than 60 beats/min with symptoms of inadequate perfusion (dizziness and fatigue). The ECG shows normal P waves and QRS complexes with the SA node firing at a slower rate, indicating sinus bradycardia.
Which of the following symptoms might a patient with symptomatic sinus bradycardia experience?
A) Palpitations
B) Syncope
C) Fever
D) Hypertension
B) Syncope
Rationale: Symptomatic bradycardia refers to a heart rate of less than 60 beats/min that causes symptoms of inadequate perfusion, such as fatigue, dizziness, chest pain, and syncope.
A patient with sinus bradycardia has a heart rate of 50 beats/min and reports chest pain and lightheadedness. What is the primary concern with this patient’s condition?
A) Potential for hypertension
B) Risk of atrial fibrillation
C) Increased myocardial oxygen demand
D) Inadequate cardiac output and perfusion
D) Inadequate cardiac output and perfusion
Rationale: The primary concern with sinus bradycardia, especially when symptomatic, is inadequate cardiac output and perfusion, leading to symptoms such as chest pain and lightheadedness.
Which diagnostic finding is consistent with sinus bradycardia on an ECG?
A) The P wave is absent
B) The heart rate is above 100 beats/min
C) The QRS complex is normal in shape and duration
D) The PR interval is prolonged
C) The QRS complex is normal in shape and duration
Rationale: In sinus bradycardia, the conduction pathway is the same as that in sinus rhythm, so the QRS complex is normal in shape and duration. The heart rate is less than 60 beats/min, but the P wave is normal and precedes each QRS complex.
Which of the following individuals might have sinus bradycardia as a normal finding?
A) An elderly patient with hypothyroidism
B) An aerobically trained athlete
C) A patient with increased intracranial pressure
D) A patient experiencing inferior myocardial infarction (MI)
B) An aerobically trained athlete
Rationale: Sinus bradycardia may be a normal finding in aerobically trained athletes and in some people during sleep. It can be a sign of high cardiovascular fitness in athletes.
A 40-year-old female presents with a heart rate of 55 beats/min, fatigue, and dizziness. She reports taking β-blockers for hypertension. What is the most likely cause of her sinus bradycardia?
A) Hypothermia
B) Increased intraocular pressure
C) Vagal stimulation
D) β-blocker medication
D) β-blocker medication
Rationale: Certain drugs, such as β-blockers, can cause sinus bradycardia by slowing the heart rate. The patient’s symptoms and medication history suggest that the β-blockers are the likely cause.
Which maneuver or condition is known to trigger sinus bradycardia?
A) Valsalva maneuver
B) High ambient temperature
C) Decreased intraocular pressure
D) Hyperthyroidism
A) Valsalva maneuver
Rationale: Sinus bradycardia can occur in response to a Valsalva maneuver, which increases intrathoracic pressure and stimulates the vagus nerve, slowing the heart rate.
A patient with sinus bradycardia is found to have increased intracranial pressure. What is a likely reason for this patient’s condition?
A) Hyperthyroidism
B) Hypothermia
C) Increased vagal stimulation
D) Inferior myocardial infarction (MI)
C) Increased vagal stimulation
Rationale: Increased intracranial pressure can lead to increased vagal stimulation, which in turn can cause sinus bradycardia. This mechanism slows the heart rate in response to the increased pressure.
Which of the following disease states is commonly associated with sinus bradycardia?
A) Hypothyroidism
B) Hyperthyroidism
C) Hyperglycemia
D) Hypertension
A) Hypothyroidism
Rationale: Common disease states associated with sinus bradycardia include hypothyroidism, increased intracranial pressure, and inferior myocardial infarction (MI). Hypothyroidism slows the metabolic rate and can lead to a slower heart rate.
Which of the following ECG findings is characteristic of sinus bradycardia?
A) Heart rate above 100 beats/min
B) Irregular heart rhythm
C) P wave preceding each QRS complex with a normal shape
D) Prolonged PR interval
C) P wave preceding each QRS complex with a normal shape
Rationale: In sinus bradycardia, the heart rate is less than 60 beats/min, and the rhythm is regular. The P wave precedes each QRS complex and has a normal shape and duration. The PR interval and QRS complex are normal.
A 70-year-old male presents with a heart rate of 55 beats/min. His ECG shows regular rhythm, normal P waves preceding each QRS complex, and normal PR and QRS intervals. What condition does this describe?
A) Sinus tachycardia
B) Atrial fibrillation
C) Sinus bradycardia
D) Ventricular tachycardia
C) Sinus bradycardia
Rationale: The patient’s heart rate is less than 60 beats/min, with a regular rhythm, normal P waves, and normal PR and QRS intervals. This describes sinus bradycardia.
What is the typical heart rate in sinus bradycardia?
A) Less than 60 beats/min
B) 60 to 100 beats/min
C) 100 to 120 beats/min
D) Greater than 120 beats/min
A) Less than 60 beats/min
Rationale: In sinus bradycardia, the heart rate is less than 60 beats/min. The rhythm is regular, and the ECG shows normal P waves, PR interval, and QRS complex.
A patient’s ECG shows a heart rate of 58 beats/min, regular rhythm, and normal P waves that precede each QRS complex. The PR interval and QRS complex are also normal. Which ECG characteristic confirms the diagnosis of sinus bradycardia?
A) The heart rate of less than 60 beats/min
B) The irregular rhythm
C) The absence of P waves
D) The prolonged QRS complex
A) The heart rate of less than 60 beats/min
Rationale: The heart rate of less than 60 beats/min, combined with a regular rhythm, normal P waves, PR interval, and QRS complex, confirms the diagnosis of sinus bradycardia.
Which of the following symptoms may indicate that a patient with sinus bradycardia is experiencing inadequate perfusion?
A) Hypertension
B) Pale, cool skin
C) Elevated heart rate
D) Warm, flushed skin
B) Pale, cool skin
Rationale: Manifestations of symptomatic bradycardia that indicate inadequate perfusion include pale, cool skin, hypotension, weakness, angina, dizziness, confusion, and shortness of breath.
A 72-year-old patient with a heart rate of 50 beats/min presents with weakness and shortness of breath. What is the most likely clinical significance of these symptoms in the context of sinus bradycardia?
A) The patient has adapted to a lower heart rate
B) The patient is experiencing improved cardiac output
C) The patient is symptomatic and experiencing inadequate perfusion
D) The patient has unrelated
C) The patient is symptomatic and experiencing inadequate perfusion
Rationale: The symptoms of weakness and shortness of breath in a patient with sinus bradycardia indicate symptomatic bradycardia, suggesting that the patient is experiencing inadequate perfusion.
Which clinical manifestation is commonly associated with symptomatic sinus bradycardia?
A) Fever
B) Hypotension
C) Hyperglycemia
D) Hypertension
B) Hypotension
Rationale: Hypotension is a common manifestation of symptomatic sinus bradycardia, indicating inadequate perfusion and reduced cardiac output.
A patient with sinus bradycardia reports dizziness and confusion. What is the primary concern regarding these symptoms?
A) The patient may have an electrolyte imbalance
B) The patient may be experiencing improved cardiac function
C) The patient may be experiencing
D) The patient may be experiencing inadequate cerebral perfusion
D) The patient may be experiencing inadequate cerebral perfusion
Rationale: Dizziness and confusion in a patient with sinus bradycardia suggest inadequate cerebral perfusion, which is a significant concern and requires prompt evaluation and management.
A patient with symptomatic bradycardia is not responding to IV atropine. What is the next appropriate step in management?
A) Administer a beta-blocker
B) Initiate transcutaneous pacing
C) Discontinue all medications
D) Provide supplemental oxygen
B) Initiate transcutaneous pacing
Rationale: If IV atropine is ineffective in treating symptomatic bradycardia, the next steps include transcutaneous pacing, or infusion of dopamine or epinephrine. These interventions help increase the heart rate and improve perfusion.
A 65-year-old male presents with a heart rate of 50 beats/min and symptoms of dizziness and weakness. He is currently on a beta-blocker for hypertension. What is the most likely cause of his bradycardia, and what should be the initial treatment approach?
A) Hypothermia; provide warming measures
B) Medication-induced; hold or reduce the beta-blocker
C) Myocardial infarction; initiate thrombolytic therapy
D) Electrolyte imbalance; correct the imbalance
B) Medication-induced; hold or reduce the beta-blocker
Rationale: The patient’s bradycardia is likely due to the beta-blocker medication. The initial treatment approach should be to hold, stop, or reduce the beta-blocker and assess the patient’s response.
Which medication is commonly used as a first-line treatment for symptomatic bradycardia?
A) IV atropine
B) IV dopamine
C) IV epinephrine
D) IV lidocaine
A) IV atropine
Rationale: IV atropine, an anticholinergic drug, is commonly used as a first-line treatment for symptomatic bradycardia to increase the heart rate.
A patient with refractory symptomatic bradycardia is not responding to IV atropine or transcutaneous pacing. What additional treatment options are available for this patient?
A) Administer a calcium channel blocker
B) Initiate diuretic therapy
C) Perform emergency pericardiocentesis
D) Infuse dopamine or epinephrine
D) Infuse dopamine or epinephrine
Rationale: If symptomatic bradycardia does not respond to IV atropine or transcutaneous pacing, additional treatment options include infusing dopamine or epinephrine to increase the heart rate and improve cardiac output.
Which characteristic is typical of a premature atrial contraction (PAC) on an ECG?
A) Prolonged QRS complex
B) Distorted P wave
C) Prolonged QT interval
D) Inverted T wave
B) Distorted P wave
Rationale: A premature atrial contraction (PAC) results in a distorted P wave due to the ectopic signal starting in the atrium and traveling across the atria by an abnormal pathway.
A 55-year-old female presents with palpitations and an irregular heartbeat. Her ECG shows an early atrial contraction with a distorted P wave before the next expected sinus beat. What is the most likely diagnosis?
A) Atrial fibrillation
B) Sinus bradycardia
C) Premature atrial contraction (PAC)
D) Ventricular tachycardia
C) Premature atrial contraction (PAC)
Rationale: The patient’s ECG shows an early atrial contraction with a distorted P wave, which is characteristic of a PAC. This occurs when an ectopic focus in the atrium fires sooner than the next expected sinus beat.
What may happen to the ectopic signal at the AV node during a premature atrial contraction (PAC)?
A) It is always conducted normally
B) It may be stopped, delayed, or conducted normally
C) It is conducted through an abnormal pathway
D) It causes a prolonged PR interval
B) It may be stopped, delayed, or conducted normally
Rationale: At the AV node, the ectopic signal during a PAC may be stopped (non-conducted PAC), delayed (lengthened PR interval), or conducted normally through the ventricles.
A patient with a history of PACs is undergoing an ECG. The results show a lengthened PR interval following an early atrial contraction. What does this finding indicate?
A) The ectopic signal is non-conducted
B) The ectopic signal is delayed
C) The ectopic signal is conducted normally
D) The ectopic signal is originating from the ventricles
B) The ectopic signal is delayed
Rationale: A lengthened PR interval following an early atrial contraction indicates that the ectopic signal is delayed at the AV node.
Which of the following best describes the origin of a premature atrial contraction (PAC)?
A) An ectopic focus in the atrium
B) The SA node firing prematurely
C) An ectopic focus in the ventricles
D) The AV node firing prematurely
A) An ectopic focus in the atrium
Rationale: A premature atrial contraction (PAC) originates from an ectopic focus in the atrium, causing an early atrial contraction before the next expected sinus beat.
Which of the following can result in a premature atrial contraction (PAC) in a normal heart?
A) Emotional stress
B) Hypothyroidism
C) Hypotension
D) Hyperglycemia
A) Emotional stress
Rationale: In a normal heart, a PAC can result from emotional stress, fatigue, caffeine, tobacco, or alcohol use.
A 45-year-old male with no history of heart disease reports feeling palpitations and a sensation of skipped heartbeats after drinking several cups of coffee. What is the most likely cause of his symptoms?
A) Supraventricular tachycardia (SVT)
B) Premature atrial contraction (PAC)
C) Ventricular fibrillation
D) Sinus bradycardia
B) Premature atrial contraction (PAC)
Rationale: The patient’s symptoms of palpitations and a sensation of skipped heartbeats after consuming caffeine are indicative of PACs, which can result from caffeine use.
Which of the following conditions can lead to a premature atrial contraction (PAC)?
A) Hypoxia
B) Hyperthyroidism
C) Chronic obstructive pulmonary disease (COPD)
D) All of the above
D) All of the above
Rationale: PACs can result from hypoxia, electrolyte imbalances, hyperthyroidism, COPD, and heart disease, including CAD and valvular disease.
A patient presents with an irregular heart rhythm. The ECG shows a P wave that is hidden in the preceding T wave and a PR interval that varies in duration. What is the most likely diagnosis?
A) Atrial fibrillation
B) Premature atrial contraction (PAC)
C) Ventricular tachycardia
D) Sinus bradycardia
B) Premature atrial contraction (PAC)
Rationale: The ECG findings of an irregular heart rhythm, a P wave hidden in the preceding T wave, and a variable PR interval are characteristic of a PAC.
Which ECG characteristic is typically normal in a premature atrial contraction (PAC)?
A) PR interval
B) QRS complex
C) P wave shape
D) Heart rate
B) QRS complex
Rationale: In a PAC, the QRS complex is usually normal, although the rhythm is irregular, and the P wave may have a different shape or be hidden in the preceding T wave.
A 60-year-old patient with a history of coronary artery disease (CAD) reports frequent palpitations. His ECG shows frequent PACs. What is the clinical significance of this finding?
A) No clinical significance
B) Likely to resolve without intervention
C) May indicate enhanced automaticity of the atria or a reentry mechanism
D) Indicative of ventricular fibrillation
C) May indicate enhanced automaticity of the atria or a reentry mechanism
Rationale: In a person with heart disease, frequent PACs may indicate enhanced automaticity of the atria or a reentry mechanism, and they may warn of or start more serious dysrhythmias, such as SVT.
What may happen to the PR interval during a premature atrial contraction (PAC)?
A) It is always prolonged
B) It remains within normal limits
C) It may be shorter or longer than the PR interval from the SA node
D) It is always shortened
C) It may be shorter or longer than the PR interval from the SA node
Rationale: The PR interval during a PAC may be shorter or longer than the PR interval coming from the SA node but is within normal limits.
A 55-year-old female with a history of hyperthyroidism presents with palpitations and irregular heartbeats. Her ECG shows PACs. What is the most likely underlying cause of her PACs?
A) Hyperthyroidism
B) Hypoxia
C) Chronic obstructive pulmonary disease (COPD)
D) Valvular heart disease
A) Hyperthyroidism
Rationale: Hyperthyroidism is one of the clinical associations that can result in PACs, along with hypoxia, electrolyte imbalances, COPD, and heart disease.
How is the heart rate affected in the presence of premature atrial contractions (PACs)?
A) The heart rate remains constant
B) The heart rate decreases significantly
C) The heart rate increases significantly
D) The heart rate varies with the underlying rate and frequency of the PACs
D) The heart rate varies with the underlying rate and frequency of the PACs
Rationale: The heart rate in the presence of PACs varies with the underlying rate and frequency of the PACs.
A patient with a healthy heart reports a sensation that their heart “skipped a beat.” What is the likely clinical significance of this finding?
A) It indicates ventricular tachycardia
B) It is a sign of supraventricular tachycardia (SVT)
C) It is likely an isolated PAC with no significant clinical impact
D) It indicates sinus bradycardia
C) It is likely an isolated PAC with no significant clinical impact
Rationale: In a person with a healthy heart, isolated PACs are not significant and may present as a sensation that the heart “skipped a beat.”
What is the typical appearance of the P wave in a premature atrial contraction (PAC) on an ECG?
A) It has the same shape as a P wave originating in the SA node
B) It is always absent
C) It is always inverted
D) It has a different shape from a P wave originating in the SA node
D) It has a different shape from a P wave originating in the SA node
Rationale: In a PAC, the P wave has a different shape from that of a P wave originating in the SA node and may be hidden in the preceding T wave.
Where does paroxysmal supraventricular tachycardia (PSVT) originate?
A) In the ventricles
B) In the SA node
C) Above the bifurcation of the bundle of His
D) In the AV node
C) Above the bifurcation of the bundle of His
Rationale: PSVT is a dysrhythmia starting in an ectopic focus anywhere above the bifurcation of the bundle of His.
A 50-year-old patient presents with palpitations and rapid heart rate. The ECG shows a rapid rhythm originating above the bundle of His. The patient reports that the symptoms started and ended abruptly. What is the most likely diagnosis?
A) Atrial fibrillation
B) Paroxysmal supraventricular tachycardia (PSVT)
C) Sinus tachycardia
D) Ventricular tachycardia
B) Paroxysmal supraventricular tachycardia (PSVT)
Rationale: The patient’s symptoms and ECG findings are characteristic of PSVT, which has an abrupt onset and ending and originates above the bundle of His.
What phenomenon is responsible for PSVT?
A) Reentrant phenomenon
B) Decreased automaticity
C) Ventricular fibrillation
D) Bundle branch block
A) Reentrant phenomenon
Rationale: PSVT occurs because of a reentrant phenomenon, which involves reexcitation of the atria when there is a one-way block.
A patient with a history of PACs reports an episode of rapid heart rate that started abruptly. The ECG confirms PSVT. What typically triggers a run of repeated premature beats in PSVT?
A) Ventricular tachycardia
B) Sinus bradycardia
C) Atrial fibrillation
D) Premature atrial contraction (PAC)
D) Premature atrial contraction (PAC)
Rationale: A PAC usually triggers a run of repeated premature beats, leading to PSVT.
What does the term “paroxysmal” indicate in PSVT?
A) Gradual onset and ending
B) Abrupt onset and ending
C) Absence of symptoms
D) Irregular rhythm
B) Abrupt onset and ending
Rationale: The term “paroxysmal” refers to an abrupt onset and ending of the dysrhythmia.
A patient experiences a brief period of asystole following the termination of a rapid heart rate episode. The ECG shows that some impulses were not conducted to the ventricles. What condition is likely present?
A) Atrial fibrillation
B) PSVT with AV block
C) Sinus tachycardia
D) Ventricular fibrillation
B) PSVT with AV block
Rationale: A brief period of asystole may follow the termination of PSVT, and AV block can prevent some impulses from being conducted to the ventricles.
What syndrome is associated with PSVT due to extra conduction or accessory pathways?
A) Wolff-Parkinson-White syndrome
B) Brugada syndrome
C) Long QT syndrome
D) Sick sinus syndrome
A) Wolff-Parkinson-White syndrome
Rationale: PSVT can occur with Wolff-Parkinson-White (WPW) syndrome or “preexcitation” due to extra conduction or accessory pathways.
A patient with WPW syndrome experiences episodes of rapid heart rate that start and end abruptly. The ECG shows a reentrant phenomenon. What is the most likely type of dysrhythmia?
A) Ventricular tachycardia
B) Sinus bradycardia
C) Atrial fibrillation
D) Paroxysmal supraventricular tachycardia
D) Paroxysmal supraventricular tachycardia
Rationale: The patient’s symptoms, history of WPW syndrome, and ECG findings are indicative of PSVT, which is associated with extra conduction or accessory pathways in WPW syndrome.
Which of the following best describes paroxysmal supraventricular tachycardia (PSVT)?
A) A dysrhythmia starting in the ventricles
B) A dysrhythmia starting in an ectopic focus above the bifurcation of the bundle of His
C) A reentrant phenomenon occurring in the ventricles
D) A dysrhythmia starting in the SA node
B) A dysrhythmia starting in an ectopic focus above the bifurcation of the bundle of His
Rationale: PSVT is a dysrhythmia that starts in an ectopic focus anywhere above the bifurcation of the bundle of His.
A 60-year-old male presents with palpitations and a rapid heart rate. The ECG reveals a regular rhythm with a heart rate of 200 beats/min and a P wave that is hidden in the preceding T wave. What is the most likely diagnosis?
A) Atrial fibrillation
B) Sinus tachycardia
C) Paroxysmal supraventricular tachycardia
D) Ventricular tachycardia
C) Paroxysmal supraventricular tachycardia
Rationale: The patient’s symptoms and ECG findings are indicative of PSVT, characterized by a regular rhythm, a rapid heart rate, and P waves that may be hidden in the preceding T wave.
What triggers a run of repeated premature beats in PSVT?
A) Premature atrial contraction
B) Ventricular tachycardia
C) Sinus bradycardia
D) Atrial fibrillation
A) Premature atrial contraction
Rationale: In PSVT, a PAC usually triggers a run of repeated premature beats, leading to the reentrant phenomenon.
A patient experiences an abrupt onset of rapid heart rate that stops suddenly, followed by a brief period of asystole. What type of dysrhythmia is most likely present?
A) Atrial fibrillation
B) Sinus bradycardia
C) Paroxysmal supraventricular tachycardia
D) Ventricular fibrillation
C) Paroxysmal supraventricular tachycardia
Rationale: PSVT is characterized by an abrupt onset and ending of the rapid heart rate, often followed by a brief period of asystole.
What phenomenon causes PSVT?
A) One-way block and reexcitation of the atria
B) Decreased automaticity
C) Bundle branch block
D) Sinus node dysfunction
A) One-way block and reexcitation of the atria
Rationale: PSVT occurs because of a reentrant phenomenon, which involves reexcitation of the atria due to a one-way block.
A 45-year-old patient with a history of WPW syndrome experiences an episode of rapid heart rate. The ECG reveals a reentrant phenomenon. What is the most likely dysrhythmia?
A) Ventricular tachycardia
B) Sinus bradycardia
C) Atrial fibrillation
D) Paroxysmal supraventricular tachycardia
D) Paroxysmal supraventricular tachycardia
Rationale: The patient’s history of WPW syndrome and ECG findings are indicative of PSVT, which can occur with WPW syndrome due to extra conduction or accessory pathways.
Which condition is associated with PSVT?
A) Rheumatic heart disease
B) Hyperthyroidism
C) Hypoglycemia
D) Cardiomyopathy
A) Rheumatic heart disease
Rationale: PSVT is associated with rheumatic heart disease, digitalis toxicity, CAD, and cor pulmonale.
A patient presents with a heart rate of 190 beats/min, hypotension, and palpitations. The ECG shows a regular rhythm with P waves that are difficult to distinguish. What is the clinical significance of this condition?
A) The condition is likely benign and self-limiting
B) The condition may lead to decreased cardiac output and requires intervention
C) The patient is experiencing ventricular tachycardia
D) The condition indicates sinus bradycardia
B) The condition may lead to decreased cardiac output and requires intervention
Rationale: A prolonged episode of PSVT with a heart rate greater than 180 beats/min can cause decreased cardiac output due to reduced stroke volume, requiring intervention.
Which symptom is commonly associated with PSVT?
A) Hypertension
B) Hyperglycemia
C) Dyspnea
D) Fever
C) Dyspnea
Rationale: Common manifestations of PSVT include hypotension, palpitations, dyspnea, and angina.
A patient with PSVT is treated with IV adenosine. What is the primary reason for using this medication?
A) To decrease heart rate gradually
B) To convert PSVT to a normal sinus rhythm
C) To increase cardiac output
D) To provide long-term management
B) To convert PSVT to a normal sinus rhythm
Rationale: IV adenosine is the drug of choice to convert PSVT to a normal sinus rhythm. It has a short half-life and is well tolerated.
What is a common vagal maneuver used to treat PSVT?
A) Deep breathing exercises
B) Valsalva maneuver
C) Walking
D) Yoga
B) Valsalva maneuver
Rationale: Common vagal maneuvers used to treat PSVT include the Valsalva maneuver and coughing.
A patient with PSVT does not respond to vagal maneuvers and remains hemodynamically unstable. What is the next appropriate intervention?
A) IV β-blockers
B) Transcutaneous pacing
C) Synchronized cardioversion
D) IV adenosine
C) Synchronized cardioversion
Rationale: If a patient with PSVT becomes hemodynamically unstable and does not respond to vagal maneuvers, synchronized cardioversion is the next appropriate intervention.
Which heart rate range is typical for PSVT?
A) 60 to 100 beats/min
B) 80 to 120 beats/min
C) 100 to 150 beats/min
D) 151 to 220 beats/min
D) 151 to 220 beats/min
Rationale: In PSVT, the heart rate is typically between 151 to 220 beats/min.
A patient with recurring PSVT and WPW syndrome is scheduled for a procedure to treat their condition. What is the likely procedure?
A) Radiofrequency catheter ablation
B) Coronary artery bypass grafting
C) Permanent pacemaker insertion
D) Atrial septal defect repair
A) Radiofrequency catheter ablation
Rationale: Treatment of recurring PSVT and PSVT in patients with WPW syndrome includes radiofrequency catheter ablation of the accessory pathway.
What is the effect of a prolonged episode of PSVT with a heart rate greater than 180 beats/min?
A) Increased cardiac output
B) Decreased stroke volume and cardiac output
C) Decreased risk of complications
D) No clinical significance
B) Decreased stroke volume and cardiac output
Rationale: A prolonged episode of PSVT with a heart rate greater than 180 beats/min can cause decreased cardiac output due to reduced stroke volume.
Which characteristic on an ECG identifies atrial flutter?
A) Irregular QRS complex
B) Sawtooth-shaped flutter waves
C) Prolonged PR interval
D) Inverted T waves
B) Sawtooth-shaped flutter waves
Rationale: Atrial flutter is identified by recurring, regular, sawtooth-shaped flutter waves originating from a single ectopic focus in the atrium.
A 68-year-old male with a history of CAD presents with palpitations and an irregular heart rhythm. His ECG shows sawtooth-shaped flutter waves with an atrial rate of 300 beats/min. What is the most likely diagnosis?
A) Atrial fibrillation
B) Sinus tachycardia
C) Atrial flutter
D) Ventricular tachycardia
C) Atrial flutter
Rationale: The patient’s symptoms and ECG findings of sawtooth-shaped flutter waves and an atrial rate of 300 beats/min are indicative of atrial flutter.
Which condition is not typically associated with atrial flutter?
A) Rheumatic fever
B) Chronic lung disease
C) Hyperthyroidism
D) Mitral valve disorders
A) Rheumatic fever
Rationale: Atrial flutter is associated with conditions such as CAD, hypertension, mitral valve disorders, pulmonary embolus, chronic lung disease, cor pulmonale, cardiomyopathy, hyperthyroidism, and the use of certain drugs.
A patient with atrial flutter has a ventricular rate of 150 beats/min. What is the most likely conduction ratio?
A) 1:1
B) 2:1
C) 3:1
D) 4:1
B) 2:1
Rationale: In atrial flutter, a 2:1 conduction ratio typically results in a ventricular rate of around 150 beats/min.
Which of the following ECG characteristics is typical of atrial flutter?
A) Variable PR interval that is not measurable
B) Irregular ventricular rhythm that is not measurable
C) Wide QRS complex that is not measurable
D) Inverted P waves that is not measurable
A) Variable PR interval that is not measurable
Rationale: In atrial flutter, the PR interval is variable and not measurable, and the QRS complex is usually normal.
A 70-year-old female with chronic lung disease is diagnosed with atrial flutter. What is a significant clinical concern for this patient?
A) Increased cardiac output and risk of heart failure
B) Reduced risk of stroke and risk of heart failure
C) Decreased cardiac output and risk of heart failure
D) Lower heart rate and risk of heart failure
C) Decreased cardiac output and risk of heart failure
Rationale: The high ventricular rates and loss of the atrial “kick” in atrial flutter can decrease cardiac output and cause serious problems, such as heart failure.
What is the primary reason for the increased risk of stroke in patients with atrial flutter?
A) High ventricular rate
B) Rapid atrial depolarization
C) Increased blood pressure
D) Formation of thrombi in the atria due to blood stasis
D) Formation of thrombi in the atria due to blood stasis
Rationale: Patients with atrial flutter have an increased risk for stroke because thrombi can form in the atria from the stasis of blood.
A patient with atrial flutter is prescribed warfarin. What is the primary goal of this treatment?
A) To increase ventricular rate
B) To prevent stroke
C) To maintain sinus rhythm
D) To decrease blood pressure
B) To prevent stroke
Rationale: The primary goal of prescribing warfarin for a patient with atrial flutter is to prevent stroke by reducing the risk of thrombus formation.
What is the primary goal in the treatment of atrial flutter?
A) Slow ventricular response by increasing AV block
B) Increase ventricular rate by increasing AV block
C) Decrease PR interval by increasing AV block
D) Increase heart rateby increasing AV block
A) Slow ventricular response by increasing AV block
Rationale: The primary goal in the treatment of atrial flutter is to slow the ventricular response by increasing AV block.
A patient with atrial flutter is clinically unstable. What is an emergency treatment option to convert atrial flutter to sinus rhythm?
A) IV adenosine
B) Vagal maneuvers
C) Electrical cardioversion
D) Anticoagulant therapy
C) Electrical cardioversion
Rationale: Electrical cardioversion may be used to convert atrial flutter to sinus rhythm in an emergency when the patient is clinically unstable.
Which drug can be used to convert atrial flutter to sinus rhythm?
A) Diltiazem
B) Verapamil
C) Ibutilide
D) Metoprolol
C) Ibutilide
Rationale: Ibutilide (Corvert) is an antidysrhythmic drug that can convert atrial flutter to sinus rhythm.
A patient with recurrent atrial flutter undergoes radiofrequency catheter ablation. What is the goal of this procedure?
A) Increase ventricular rate and restore normal sinus rhythm
B) Ablate the ectopic foci and restore normal sinus rhythm
C) Perform synchronized cardioversion and restore normal sinus rhythm
D) Administer anticoagulation therapy and restore normal sinus rhythm
B) Ablate the ectopic foci and restore normal sinus rhythm
Rationale: The goal of radiofrequency catheter ablation is to ablate (destroy) the ectopic foci in the right atrium, restoring normal sinus rhythm.
What is the typical atrial rate range in atrial flutter?
A) 100 to 150 beats/min
B) 150 to 200 beats/min
C) 200 to 350 beats/min
D) 350 to 400 beats/min
C) 200 to 350 beats/min
Rationale: In atrial flutter, the atrial rate is typically between 200 to 350 beats/min.
A patient with atrial flutter has a ventricular rate of 150 beats/min and experiences palpitations and dyspnea. What medication class can be used to control the ventricular rate?
A) β-blockers
B) Anticoagulants
C) Diuretics
D) ACE inhibitors
A) β-blockers
Rationale: Drugs used to control the ventricular rate in atrial flutter include β-blockers and calcium channel blockers.
Which of the following drugs can maintain sinus rhythm in a patient with atrial flutter?
A) Amiodarone
B) Digoxin
C) Verapamil
D) Diltiazem
A) Amiodarone
Rationale: Amiodarone is an antidysrhythmic drug that can maintain sinus rhythm in a patient with atrial flutter.
A patient with atrial flutter is undergoing an EPS and radiofrequency catheter ablation. What is the expected outcome of this procedure?
A) Increased ventricular rate
B) Reduced atrial rate
C) Temporary relief of symptoms
D) Restoration of normal sinus rhythm
D) Restoration of normal sinus rhythm
Rationale: The expected outcome of radiofrequency catheter ablation is the restoration of normal sinus rhythm by destroying the ectopic foci.
Which clinical manifestation is commonly associated with atrial flutter?
A) Hypertension
B) Increased cardiac output
C) Bradycardia
D) Heart failure
D) Heart failure
Rationale: Atrial flutter can lead to decreased cardiac output and cause serious problems such as heart failure, especially in patients with underlying heart disease.
Which characteristic is most indicative of atrial fibrillation on an ECG?
A) Regular P waves
B) Sawtooth-shaped flutter waves
C) Chaotic, fibrillatory waves replacing P waves
D) Prolonged PR interval
C) Chaotic, fibrillatory waves replacing P waves
Rationale: Atrial fibrillation is characterized by chaotic, fibrillatory waves replacing the P waves on an ECG.
A 75-year-old male presents with palpitations and an irregular heart rhythm. The ECG shows chaotic, fibrillatory waves with an atrial rate of 400 beats/min and a ventricular rate of 110 beats/min. What is the most likely diagnosis?
A) Atrial flutter with a rapid ventricular response
B) Sinus tachycardia with a rapid ventricular response
C) Atrial fibrillation with a rapid ventricular response
D) Ventricular tachycardia with a rapid ventricular response
C) Atrial fibrillation with a rapid ventricular response
Rationale: The patient’s symptoms and ECG findings indicate atrial fibrillation with a rapid ventricular response, characterized by chaotic, fibrillatory waves and a ventricular rate greater than 100 beats/min.
Which condition is commonly associated with atrial fibrillation?
A) Hypertensive heart disease
B) Chronic obstructive pulmonary disease (COPD)
C) Rheumatic fever
D) Hyperglycemia
A) Hypertensive heart disease
Rationale: Atrial fibrillation is commonly associated with conditions such as CAD, valvular heart disease, cardiomyopathy, hypertensive heart disease, HF, and pericarditis.
A patient with a history of thyrotoxicosis experiences an acute onset of palpitations and irregular heartbeats. The ECG shows an atrial rate of 450 beats/min and fibrillatory waves. What is the likely diagnosis?
A) Ventricular fibrillation
B) Sinus bradycardia
C) Atrial fibrillation
D) Ventricular tachycardia
C) Atrial fibrillation
Rationale: The patient’s history of thyrotoxicosis and ECG findings of an atrial rate of 450 beats/min and fibrillatory waves are indicative of atrial fibrillation.
What is the clinical significance of atrial fibrillation with a rapid ventricular response?
A) Increased cardiac output
B) Decreased cardiac output
C) Lower risk of stroke
D) Increased atrial contraction
B) Decreased cardiac output
Rationale: Atrial fibrillation with a rapid ventricular response can decrease cardiac output due to ineffective atrial contractions and a rapid ventricular rate.
A 70-year-old female with a history of atrial fibrillation is at increased risk for stroke. What is the primary reason for this increased risk?
A) High ventricular rate due to blood stasis
B) Increased atrial contraction due to blood stasis
C) Elevated blood pressure due to blood stasis
D) Thrombi formation in the atria due to blood stasis
D) Thrombi formation in the atria due to blood stasis
Rationale: Atrial fibrillation increases the risk for stroke because thrombi can form in the atria due to blood stasis, and an embolized clot can move to the brain.
Which ECG finding is not typically associated with atrial fibrillation?
A) Measurable PR interval
B) Irregular ventricular rhythm
C) Normal QRS complex
D) Chaotic, fibrillatory waves
A) Measurable PR interval
Rationale: During atrial fibrillation, the PR interval is not measurable. The QRS complex is usually normal, and the ventricular rhythm is irregular.
A patient with atrial fibrillation has a ventricular rate of 90 beats/min. How is this condition described in terms of ventricular response?
A) Uncontrolled ventricular response
B) Rapid ventricular response
C) Bradycardic response
D) Controlled ventricular response
D) Controlled ventricular response
Rationale: When the ventricular rate is between 60 and 100 beats/min, atrial fibrillation is described as having a controlled ventricular response.
What is the atrial rate range typically seen in atrial fibrillation?
A) 100 to 150 beats/min
B) 150 to 250 beats/min
C) 250 to 300 beats/min
D) 350 to 600 beats/min
D) 350 to 600 beats/min
Rationale: During atrial fibrillation, the atrial rate may be as high as 350 to 600 beats/min.
A patient with atrial fibrillation presents with an embolic stroke. What is the most likely cause of this stroke?
A) Elevated blood pressure from the atria
B) Decreased heart rate from the atria
C) Hypercoagulability from the atria
D) Embolized clot from the atria
D) Embolized clot from the atria
Rationale: An embolic stroke in a patient with atrial fibrillation is most likely caused by an embolized clot that formed in the atria due to blood stasis.
Which type of atrial fibrillation is characterized by episodes that begin and end spontaneously?
A) Persistent atrial fibrillation
B) Paroxysmal atrial fibrillation
C) Chronic atrial fibrillation
D) Permanent atrial fibrillation
B) Paroxysmal atrial fibrillation
Rationale: Paroxysmal atrial fibrillation is characterized by episodes that begin and end spontaneously.
A 65-year-old male with CAD presents with new-onset atrial fibrillation. What is a common clinical manifestation associated with this condition?
A) Increased cardiac output
B) Regular heart rhythm
C) Palpitations and irregular heartbeats
D) Lower risk of embolic events
C) Palpitations and irregular heartbeats
Rationale: Common clinical manifestations of atrial fibrillation include palpitations, irregular heartbeats, and a decreased cardiac output.
Which patient population is at an increased risk for developing atrial fibrillation?
A) Young athletes
B) Patients with chronic lung disease
C) Elderly patients
D) Patients with type 1 diabetes
C) Elderly patients
Rationale: The prevalence of atrial fibrillation increases with age, making elderly patients more susceptible to developing this dysrhythmia.
A patient with atrial fibrillation and a history of hypertension is being managed for both conditions. Which underlying condition is commonly associated with atrial fibrillation?
A) Hyperthyroidism
B) Hypotension
C) Hypertensive heart disease
D) Chronic liver disease
C) Hypertensive heart disease
Rationale: Atrial fibrillation is commonly associated with underlying conditions such as hypertensive heart disease, CAD, valvular heart disease, and cardiomyopathy.
During an episode of atrial fibrillation, what typically replaces the P waves on an ECG?
A) Chaotic, fibrillatory waves
B) Sawtooth-shaped flutter waves
C) Wide QRS complexes
D) Prolonged PR intervals
A) Chaotic, fibrillatory waves
Rationale: During atrial fibrillation, chaotic, fibrillatory waves replace the P waves on an ECG.
A patient experiences an episode of atrial fibrillation with a ventricular rate of 120 beats/min. What term describes this ventricular response?
A) Bradycardic response
B) Controlled ventricular response
C) Rapid ventricular response
D) Normal ventricular response
C) Rapid ventricular response
Rationale: Atrial fibrillation with a ventricular rate greater than 100 beats/min is described as atrial fibrillation with a rapid (or uncontrolled) ventricular response.
Which characteristic is most indicative of atrial fibrillation on an ECG?
A) Regular P waves
B) Chaotic, fibrillatory waves replacing P waves
C) Sawtooth-shaped flutter waves
D) Prolonged PR interval
B) Chaotic, fibrillatory waves replacing P waves
Rationale: Atrial fibrillation is characterized by chaotic, fibrillatory waves replacing the P waves on an ECG.
A 75-year-old male presents with palpitations and an irregular heart rhythm. The ECG shows chaotic, fibrillatory waves with an atrial rate of 400 beats/min and a ventricular rate of 110 beats/min. What is the most likely diagnosis?
A) Atrial flutter with a rapid ventricular response
B) Sinus tachycardia with a rapid ventricular response
C) Ventricular tachycardia with a rapid ventricular response
D) Atrial fibrillation with a rapid ventricular response
D) Atrial fibrillation with a rapid ventricular response
Rationale: The patient’s symptoms and ECG findings indicate atrial fibrillation with a rapid ventricular response, characterized by chaotic, fibrillatory waves and a ventricular rate greater than 100 beats/min.
Which condition is commonly associated with atrial fibrillation?
A) Hypertensive heart disease
B) Chronic obstructive pulmonary disease (COPD)
C) Rheumatic fever
D) Hyperglycemia
A) Hypertensive heart disease
Rationale: Atrial fibrillation is commonly associated with conditions such as CAD, valvular heart disease, cardiomyopathy, hypertensive heart disease, HF, and pericarditis.
A patient with a history of thyrotoxicosis experiences an acute onset of palpitations and irregular heartbeats. The ECG shows an atrial rate of 450 beats/min and fibrillatory waves. What is the likely diagnosis?
A) Ventricular fibrillation
B) Sinus bradycardia
C) Atrial fibrillation
D) Ventricular tachycardia
C) Atrial fibrillation
Rationale: The patient’s history of thyrotoxicosis and ECG findings of an atrial rate of 450 beats/min and fibrillatory waves are indicative of atrial fibrillation.
What is the clinical significance of atrial fibrillation with a rapid ventricular response?
A) Increased cardiac output
B) Decreased cardiac output
C) Lower risk of stroke
D) Increased atrial contraction
B) Decreased cardiac output
Rationale: Atrial fibrillation with a rapid ventricular response can decrease cardiac output due to ineffective atrial contractions and a rapid ventricular rate.
Which ECG finding is not typically associated with atrial fibrillation?
A) Measurable PR interval
B) Irregular ventricular rhythm
C) Normal QRS complex
D) Chaotic, fibrillatory waves
A) Measurable PR interval
Rationale: During atrial fibrillation, the PR interval is not measurable. The QRS complex is usually normal, and the ventricular rhythm is irregular.
A patient with atrial fibrillation has a ventricular rate of 90 beats/min. How is this condition described in terms of ventricular response?
A) Uncontrolled ventricular response
B) Rapid ventricular response
C) Bradycardic response
D) Controlled ventricular response
D) Controlled ventricular response
Rationale: When the ventricular rate is between 60 and 100 beats/min, atrial fibrillation is described as having a controlled ventricular response.
What is the primary goal in the treatment of atrial fibrillation?
A) Increase ventricular rate response to less than 100 beats/min
B) Prevent stroke
C) Decrease ventricular response to less than 100 beats/min
D) Increase blood pressure
C) Decrease ventricular response to less than 100 beats/min
Rationale: The primary goal in the treatment of atrial fibrillation is to decrease the ventricular response to less than 100 beats/min, along with preventing stroke and converting to sinus rhythm if possible.
A patient with atrial fibrillation has a ventricular rate of 120 beats/min. Which medication would be appropriate for rate control?
A) Calcium channel blocker (e.g., diltiazem)
B) Diuretic (e.g., furosemide)
C) Anticoagulant (e.g., warfarin)
D) Nitrate (e.g., nitroglycerin)
A) Calcium channel blocker (e.g., diltiazem)
Rationale: Drugs used for rate control in atrial fibrillation include calcium channel blockers (e.g., diltiazem), β-blockers (e.g., metoprolol), amiodarone, and digoxin.
Which antidysrhythmic drug is commonly used for conversion to and maintenance of sinus rhythm in atrial fibrillation?
A) Metoprolol
B) Digoxin
C) Amiodarone
D) Diltiazem
C) Amiodarone
Rationale: Amiodarone is the most common antidysrhythmic drug used for conversion to and maintenance of sinus rhythm in atrial fibrillation.
A patient with atrial fibrillation has been in this dysrhythmia for over 48 hours. What is the recommended anticoagulation therapy before attempting electrical cardioversion?
A) 1 week of anticoagulation therapy with dabigatran
B) 2 weeks of anticoagulation therapy with rivaroxaban
C) 3 to 4 weeks of anticoagulation therapy with warfarin
D) No anticoagulation therapy needed
C) 3 to 4 weeks of anticoagulation therapy with warfarin
Rationale: If a patient has been in atrial fibrillation for longer than 48 hours, anticoagulation therapy with warfarin is needed for 3 to 4 weeks before attempting electrical cardioversion.
What diagnostic test may be performed to rule out clots in the atria before cardioversion?
A) Transesophageal echocardiogram (TEE)
B) Electrocardiogram (ECG)
C) Echocardiogram (ECHO)
D) Stress test
A) Transesophageal echocardiogram (TEE)
Rationale: A transesophageal echocardiogram (TEE) may be performed to rule out clots in the atria before cardioversion in a patient with atrial fibrillation.
A patient with atrial fibrillation is on long-term warfarin therapy. What monitoring is required to ensure therapeutic levels of the medication?
A) Complete blood count (CBC)
B) International normalized ratio (INR)
C) Electrolyte panel
D) Liver function tests
B) International normalized ratio (INR)
Rationale: Patients on long-term warfarin therapy require monitoring of therapeutic levels using the international normalized ratio (INR).
Which anticoagulant does not require routine laboratory testing?
A) Dabigatran
B) Warfarin
C) Heparin
D) Aspirin
A) Dabigatran
Rationale: Alternatives to warfarin, such as dabigatran (Pradaxa), do not require routine laboratory testing for therapeutic levels.
A patient with atrial fibrillation is refractory to drugs and electrical conversion. What further treatment option involves the destruction of the AV node and insertion of a permanent ventricular pacemaker?
A) Radiofrequency catheter ablation
B) AV nodal ablation
C) Electrical cardioversion
D) Maze procedure
B) AV nodal ablation
Rationale: AV nodal ablation involves the destruction of the AV node and insertion of a permanent ventricular pacemaker for patients with atrial fibrillation refractory to drugs and electrical conversion.
Which procedure interrupts the ectopic foci causing atrial fibrillation by making incisions in both atria and using cryoablation?
A) AV nodal ablation
B) Electrical cardioversion
C) Radiofrequency catheter ablation
D) Maze procedure
D) Maze procedure
Rationale: The Maze procedure stops atrial fibrillation by making incisions in both atria and using cryoablation to interrupt the ectopic foci causing the dysrhythmia.
A patient with atrial fibrillation is being considered for a Maze procedure. What is the expected outcome of this procedure?
A) Decreased ventricular rate
B) Increased atrial rate
C) Temporary reduction in symptoms
D) Restoration of normal sinus rhythm
D) Restoration of normal sinus rhythm
Rationale: The expected outcome of the Maze procedure is the restoration of normal sinus rhythm by interrupting the ectopic foci causing atrial fibrillation.
Which of the following is not a goal of atrial fibrillation treatment?
A) Decrease ventricular response to less than 100 beats/min
B) Prevent stroke
C) Increase blood pressure
D) Convert to sinus rhythm
C) Increase blood pressure
Rationale: The goals of atrial fibrillation treatment are to decrease the ventricular response to less than 100 beats/min, prevent stroke, and convert to sinus rhythm, if possible.
A patient with reduced exercise tolerance on rate control drugs for atrial fibrillation is being considered for conversion to sinus rhythm. What is a commonly used method for this conversion?
A) Electrical cardioversion
B) Anticoagulation therapy
C) Beta-blockers
D) Calcium channel blockers
A) Electrical cardioversion
Rationale: Electrical cardioversion may be used to convert atrial fibrillation to normal sinus rhythm, especially in patients with reduced exercise tolerance on rate control drugs.
Which medication is used to control the ventricular rate in atrial fibrillation by blocking calcium channels?
A) Amiodarone
B) Digoxin
C) Diltiazem
D) Metoprolol
C) Diltiazem
Rationale: Diltiazem is a calcium channel blocker used to control the ventricular rate in atrial fibrillation.
What is the primary purpose of left atrial appendage (LAA) occlusion in patients with atrial fibrillation?
A) To increase atrial contraction and decrease the incidence of strokes
B) To prevent blood clot formation and decrease the incidence of strokes
C) To enhance ventricular response and decrease the incidence of strokes
D) To improve blood flow to the left atrium and decrease the incidence of strokes
B) To prevent blood clot formation and decrease the incidence of strokes
Rationale: LAA occlusion is a treatment strategy to prevent blood clot formation in patients with atrial fibrillation, thereby decreasing the incidence of strokes.
A 68-year-old male with atrial fibrillation cannot take oral anticoagulants. What alternative treatment can be considered to prevent blood clot formation in this patient?
A) Use of a special stapler to remove the LAA
B) Administration of beta-blockers
C) Increase in physical activity
D) Dietary changes
A) Use of a special stapler to remove the LAA
Rationale: For patients who cannot use oral anticoagulants, LAA occlusion can be considered as an alternative treatment. This can be done using a special stapler to remove the LAA or an occlusion device to shut off blood flow to the LAA.
Which device can be used to occlude the left atrial appendage (LAA)?
A) Amplatzer Amulet Occluder
B) Pacemaker
C) Cardioverter
D) Stent
A) Amplatzer Amulet Occluder
Rationale: The Amplatzer Amulet Occluder is a device used to occlude the LAA by preventing blood from flowing into and out of the LAA.
A patient with atrial fibrillation undergoes LAA occlusion with an Amplatzer Amulet Occluder. What is the mechanism of action of this device?
A) It stimulates atrial contraction
B) It prevents the formation of clots by occluding the LAA
C) It enhances ventricular response
D) It promotes blood flow to the LAA
B) It prevents the formation of clots by occluding the LAA
Rationale: The Amplatzer Amulet Occluder is positioned around the LAA and closed like a clamp, preventing blood from flowing into and out of the LAA, thereby preventing clot formation.
Why might a patient with atrial fibrillation undergo left atrial appendage (LAA) occlusion instead of taking oral anticoagulants?
A) To increase atrial fibrillation episodes
B) To avoid side effects of oral anticoagulants
C) To promote blood clot formation
D) To improve heart rate variability
B) To avoid side effects of oral anticoagulants
Rationale: LAA occlusion is an alternative treatment for patients with atrial fibrillation who cannot use oral anticoagulants due to contraindications or side effects.
Where do junctional dysrhythmias originate?
A) AV junction
B) SA node
C) Bundle branches
D) Purkinje fibers
A) AV junction
Rationale: Junctional dysrhythmias start in the area of the AV node to the bundle of His, known as the AV junction.
A 70-year-old patient presents with palpitations and an abnormal ECG. The ECG shows inverted P waves occurring just after the QRS complex. What is the most likely diagnosis?
A) Sinus tachycardia
B) Atrial fibrillation
C) Ventricular tachycardia
D) Junctional dysrhythmia
D) Junctional dysrhythmia
Rationale: The patient’s ECG shows inverted P waves occurring just after the QRS complex, which is characteristic of a junctional dysrhythmia.
What causes the impulse to move in a retrograde (backward) fashion in junctional dysrhythmias?
A) Blockage of the impulse in the AV node
B) Increased sympathetic stimulation
C) Failure of the SA node to fire or signal block
D) Hyperkalemia
C) Failure of the SA node to fire or signal block
Rationale: Junctional dysrhythmias result because the SA node does not fire, or the signal is blocked, causing the AV node to become the pacemaker of the heart and the impulse to move in a retrograde fashion.
A patient with CAD presents with a heart rate of 50 beats/min and an ECG showing a regular rhythm with an inverted P wave hidden in the QRS complex. What type of junctional dysrhythmia is most likely present?
A) Accelerated junctional rhythm
B) Junctional escape rhythm
C) Junctional tachycardia
D) Premature ventricular contraction
B) Junctional escape rhythm
Rationale: The patient’s heart rate of 40 to 60 beats/min, regular rhythm, and inverted P wave hidden in the QRS complex indicate a junctional escape rhythm.
Which condition is commonly associated with junctional dysrhythmias?
A) Asthma
B) Hyperthyroidism
C) CAD
D) Diabetes mellitus
C) CAD
Rationale: Junctional dysrhythmias are often associated with CAD, HF, cardiomyopathy, electrolyte imbalances, inferior MI, and rheumatic heart disease.
A 65-year-old patient with a history of inferior MI presents with an accelerated heart rate and an ECG showing an inverted P wave before the QRS complex. What type of junctional dysrhythmia is this?
A) Junctional escape rhythm
B) Sinus tachycardia
C) Accelerated junctional rhythm
D) Atrial flutter
C) Accelerated junctional rhythm
Rationale: The patient’s heart rate of 61 to 100 beats/min and ECG findings of an inverted P wave before the QRS complex suggest an accelerated junctional rhythm.
What is the typical heart rate range in junctional tachycardia?
A) 40 to 60 beats/min
B) 61 to 100 beats/min
C) 101 to 180 beats/min
D) Greater than 180 beats/min
C) 101 to 180 beats/min
Rationale: In junctional tachycardia, the heart rate is typically between 101 to 180 beats/min.
A patient with an accelerated junctional rhythm caused by digoxin toxicity presents with an irregular heart rate and symptoms of nausea and vomiting. What is the appropriate treatment?
A) Discontinuation of digoxin
B) Administration of atropine
C) Increasing the dose of digoxin
D) Administration of epinephrine
A) Discontinuation of digoxin
Rationale: If an accelerated junctional rhythm or junctional tachycardia is caused by drug toxicity (e.g., digoxin), the appropriate treatment is to stop the drug.
What is the clinical significance of junctional escape rhythms?
A) They indicate a normal sinus rhythm
B) They serve as a safety mechanism when the SA node is not effective
C) They require immediate suppression
D) They always indicate digitalis toxicity
B) They serve as a safety mechanism when the SA node is not effective
Rationale: Junctional escape rhythms serve as a safety mechanism when the SA node has not been effective and should not be suppressed.
A patient with junctional tachycardia has a heart rate of 150 beats/min and presents with hypotension and dizziness. What is the primary concern for this patient?
A) Increased cardiac output
B) Increased oxygenation
C) Reduced cardiac output and hemodynamic instability
D) Improved exercise tolerance
C) Reduced cardiac output and hemodynamic instability
Rationale: Junctional tachycardia indicates a more serious problem and may reduce cardiac output, causing the patient to become hemodynamically unstable (e.g., hypotensive).
Which ECG characteristic is typical of a junctional escape rhythm?
A) Inverted P wave hidden in the QRS complex
B) Prolonged PR interval
C) Wide QRS complex
D) Irregular rhythm
A) Inverted P wave hidden in the QRS complex
Rationale: In junctional escape rhythm, the P wave is abnormal in shape and inverted. It may be hidden in the QRS complex.
A patient with symptoms of dizziness and fatigue has an ECG showing a regular rhythm with a heart rate of 55 beats/min and a PR interval of less than 0.12 seconds. What is the most likely diagnosis?
A) Sinus bradycardia
B) Atrial fibrillation
C) Ventricular tachycardia
D) Junctional escape rhythm
D) Junctional escape rhythm
Rationale: The patient’s heart rate of 40 to 60 beats/min, regular rhythm, and PR interval of less than 0.12 seconds suggest a junctional escape rhythm.
Which treatment is indicated for symptomatic junctional escape rhythm?
A) Calcium channel blockers
B) Atropine
C) β-blockers
D) Amiodarone
B) Atropine
Rationale: If a patient has symptoms with a junctional escape rhythm, atropine can be used to increase the heart rate.
A patient with junctional tachycardia caused by digitalis toxicity is prescribed amiodarone for rate control. What is the rationale behind this treatment?
A) To increase heart rate
B) To control the ventricular rate and prevent tachycardia
C) To promote sympathetic stimulation
D) To increase AV node conduction
B) To control the ventricular rate and prevent tachycardia
Rationale: In the absence of digitalis toxicity, amiodarone is used for rate control to prevent tachycardia and manage the heart rate.
What is the role of β-blockers in the treatment of junctional dysrhythmias?
A) To decrease sympathetic stimulation and control heart rate
B) To increase heart rate
C) To promote AV node conduction
D) To increase cardiac output
A) To decrease sympathetic stimulation and control heart rate
Rationale: β-blockers are used to decrease sympathetic stimulation and control the heart rate in patients with junctional dysrhythmias.
A patient with a history of HF presents with an accelerated junctional rhythm and symptoms of palpitations and fatigue. Which underlying condition might have contributed to the development of this dysrhythmia?
A) Rheumatic heart disease
B) Hyperglycemia
C) Asthma
D) Chronic kidney disease
A) Rheumatic heart disease
Rationale: Junctional dysrhythmias are often associated with conditions such as HF, CAD, cardiomyopathy, electrolyte imbalances, inferior MI, and rheumatic heart disease.
What is the PR interval typically in junctional dysrhythmias when the P wave precedes the QRS complex?
A) Greater than 0.20 seconds
B) Between 0.12 and 0.20 seconds
C) Less than 0.12 seconds
D) Variable and not measurable
C) Less than 0.12 seconds
Rationale: The PR interval is less than 0.12 seconds when the P wave precedes the QRS complex in junctional dysrhythmias.
A 62-year-old patient with a history of hypertension and coronary artery disease (CAD) is admitted for chest pain and palpitations. Telemetry monitoring shows frequent premature ventricular contractions (PVCs), including couplets and an episode of ventricular bigeminy. The patient reports feeling lightheaded and weak. Which of the following interventions should the nurse anticipate?
A. Administer IV amiodarone
B. Perform carotid sinus massage
C. Prepare the patient for synchronized cardioversion
D. Encourage the patient to take deep breaths and bear down
A. Administer IV amiodarone
Rationale: Frequent PVCs, especially in patterns like couplets and bigeminy, can progress to ventricular tachycardia (VT) or ventricular fibrillation (VF), particularly in a patient with CAD. IV amiodarone is used to stabilize the heart’s electrical activity and prevent further dysrhythmias.
A patient admitted with hypokalemia and dehydration is on continuous telemetry monitoring. The ECG shows multifocal PVCs occurring in runs of three, followed by normal sinus rhythm. What is the priority nursing intervention?
A. Check the patient’s magnesium level
B. Administer a beta-blocker
C. Encourage ambulation to reduce ectopic activity
D. Instruct the patient to increase caffeine intake to improve alertness
A. Check the patient’s magnesium level
Rationale: Electrolyte imbalances, particularly hypokalemia and hypomagnesemia, are major causes of PVCs. Checking the magnesium level is essential because hypomagnesemia often coexists with hypokalemia and increases the risk of ventricular arrhythmias.
A 72-year-old patient in the ICU is experiencing ventricular trigeminy on ECG. The patient is asymptomatic with a stable blood pressure of 128/76 mmHg. Which action should the nurse take first?
A. Assess for possible triggers, such as electrolyte imbalances or hypoxia
B. Prepare for immediate defibrillation
C. Administer epinephrine IV push
D. Initiate transcutaneous pacing
A. Assess for possible triggers, such as electrolyte imbalances or hypoxia
Rationale: PVCs in a stable patient should first be evaluated for underlying causes, such as hypoxia, electrolyte disturbances, or acid-base imbalances. Treating the underlying cause often resolves the dysrhythmia without requiring antiarrhythmic drugs.
A patient on telemetry reports palpitations and the ECG shows PVCs with an R-on-T phenomenon. The nurse recognizes this pattern increases the risk for which life-threatening dysrhythmia?
A. Atrial fibrillation
B. First-degree AV block
C. Ventricular fibrillation
D. Supraventricular tachycardia
C. Ventricular fibrillation
Rationale: R-on-T PVCs occur during the relative refractory period, when the heart is vulnerable to chaotic electrical activity. This increases the risk of ventricular fibrillation (VF), a life-threatening dysrhythmia that leads to cardiac arrest if not treated immediately.
A nurse is reviewing telemetry strips and notes a pattern of ventricular bigeminy in a patient recovering from myocardial infarction (MI). The patient denies chest pain or dizziness, and vital signs are stable. What is the best nursing intervention?
A. Prepare for emergency defibrillation
B. Immediately administer atropine IV push
C. Initiate CPR
D. Continue to monitor and notify the provider if symptoms develop
D. Continue to monitor and notify the provider if symptoms develop
Rationale: Asymptomatic PVCs in a stable patient post-MI require monitoring rather than immediate intervention. The nurse should continue assessing for changes in symptoms or worsening dysrhythmias that may require treatment.
A 45-year-old patient reports feeling their heart “skipping beats” and experiencing occasional dizziness. The ECG shows unifocal PVCs occurring every third beat (trigeminy). Which statement by the nurse is most appropriate?
A. “We will check your potassium and magnesium levels to see if they are contributing to the PVCs.”
B. “You need an immediate pacemaker insertion to prevent further complications.”
C. “This is a normal variation of heart rhythm that does not require treatment.”
D. “You should avoid all physical activity to prevent worsening of your condition.”
A. “We will check your potassium and magnesium levels to see if they are contributing to the PVCs.”
Rationale: Electrolyte imbalances (especially hypokalemia and hypomagnesemia) are common causes of PVCs. Checking these levels is an essential part of the assessment to determine if replacement therapy is needed.
A nurse is caring for a patient with frequent multifocal PVCs. The nurse reviews the patient’s morning labs and notes:
* Potassium: 3.2 mEq/L
* Magnesium: 1.5 mg/dL
* Calcium: 9.0 mg/dL
Which provider order should the nurse anticipate?
A. Give an IV calcium gluconate bolus
B. Start an epinephrine infusion
C. Administer IV potassium chloride and magnesium sulfate
D. Prepare for synchronized cardioversion
C. Administer IV potassium chloride and magnesium sulfate
Rationale: Hypokalemia (K+ < 3.5) and hypomagnesemia (Mg < 1.7) increase ventricular excitability, leading to multifocal PVCs. Replacing potassium and magnesium stabilizes the myocardium and reduces the risk of life-threatening dysrhythmias.
A patient with heart failure is experiencing frequent PVCs. The nurse educates the patient on lifestyle modifications. Which statement by the patient indicates the need for further teaching?
A. “I will reduce my caffeine intake to help prevent PVCs.”
B. “I should take my diuretics as prescribed to avoid electrolyte imbalances.”
C. “If I feel palpitations, I should immediately take an extra dose of my beta-blocker.”
D. “I need to manage my stress to help reduce the occurrence of PVCs.”
C. “If I feel palpitations, I should immediately take an extra dose of my beta-blocker.”
Rationale: Patients should never self-adjust their beta-blocker dosage. Taking an extra dose could cause bradycardia and hypotension, potentially worsening cardiac instability. The patient should report increased PVCs to their provider instead.
A 55-year-old male with a history of hypertension and coronary artery disease (CAD) is admitted for chest pain and palpitations. The ECG reveals frequent PVCs. Upon further assessment, the nurse learns that the patient drinks 4 cups of coffee daily and smokes 1 pack of cigarettes per day. Which of the following lifestyle modifications should the nurse emphasize first?
A. Avoid exercise, as it may worsen PVCs
B. Reduce caffeine and smoking to decrease PVC frequency
C. Start taking a daily aspirin to prevent myocardial infarction
D. Increase alcohol intake to help with stress management
B. Reduce caffeine and smoking to decrease PVC frequency
Rationale: Caffeine and nicotine are common triggers of PVCs because they increase myocardial excitability. Educating the patient about reducing or eliminating these stimulants is the first step in managing PVCs associated with lifestyle factors.
A 38-year-old female with a history of mitral valve prolapse reports feeling occasional skipped heartbeats. She recently started an intense workout routine and consumes energy drinks daily. The nurse reviews her ECG and observes frequent unifocal PVCs. What is the most appropriate nursing intervention?
A. Prepare for immediate synchronized cardioversion
B. Administer IV epinephrine to improve cardiac output
C. Advise the patient to stop consuming energy drinks
D. Instruct the patient to increase exercise intensity
C. Advise the patient to stop consuming energy drinks
Rationale: Energy drinks contain high levels of caffeine and stimulants (e.g., taurine, guarana), which can increase myocardial irritability and lead to PVCs. Eliminating these drinks can reduce ectopic activity and prevent worsening dysrhythmias.
A patient with heart failure (HF) is experiencing frequent PVCs. The provider orders a serum electrolyte panel. Which of the following electrolyte imbalances is most concerning in this patient?
A. Potassium 2.9 mEq/L
B. Sodium 140 mEq/L
C. Chloride 102 mEq/L
D. Calcium 9.2 mg/dL
A. Potassium 2.9 mEq/L
Rationale: Hypokalemia (K+ < 3.5 mEq/L) is a significant cause of PVCs because it increases cardiac excitability and the risk for life-threatening ventricular dysrhythmias, particularly in patients with heart failure. Potassium replacement is crucial to stabilize myocardial electrical activity.
A patient recovering from a myocardial infarction (MI) is experiencing occasional PVCs. The nurse reviews the patient’s medication list. Which medication may be contributing to the occurrence of PVCs?
A. Isoproterenol
B. Metoprolol
C. Lisinopril
D. Atorvastatin
A. Isoproterenol
Rationale: Isoproterenol is a beta-adrenergic agonist that increases heart rate and myocardial excitability, making it a potential cause of PVCs. This medication should be used cautiously in patients with a history of MI to avoid inducing ventricular dysrhythmias.
A patient with pneumonia and a fever of 102.5°F (39.2°C) reports feeling palpitations. The ECG reveals frequent PVCs. What is the best initial nursing action?
A. Prepare for immediate defibrillation
B. Start IV amiodarone
C. Encourage the patient to perform the Valsalva maneuver
D. Administer acetaminophen to reduce fever
D. Administer acetaminophen to reduce fever
Rationale: Fever increases sympathetic nervous system activity, which can lead to increased myocardial excitability and PVCs. Reducing the fever with antipyretics (e.g., acetaminophen) can help decrease ectopic activity and stabilize the heart rhythm.
A patient’s ECG shows an irregular rhythm with widened QRS complexes occurring prematurely and without preceding P waves. The nurse notes that the T waves are large and in the opposite direction of the QRS complexes. Based on this information, which dysrhythmia is the patient experiencing?
A. Atrial Fibrillation
B. Premature Ventricular Contractions (PVCs)
C. First-Degree AV Block
D. Supraventricular Tachycardia (SVT)
B. Premature Ventricular Contractions (PVCs)
Rationale: PVCs are identified on ECG by wide and distorted QRS complexes that occur prematurely without a preceding P wave. The T wave is typically large and opposite in direction to the QRS complex due to abnormal ventricular repolarization. This distinguishes PVCs from atrial fibrillation (irregular atrial activity with fibrillatory waves), first-degree AV block (prolonged PR interval), and SVT (narrow QRS with rapid HR).
A nurse is analyzing a patient’s ECG strip and observes that some QRS complexes are wide and distorted, lasting more than 0.12 seconds. The nurse notes that the PR interval is not measurable for these beats, and the rhythm appears irregular. Which characteristic best confirms that these are PVCs?
A. T waves are large and opposite in direction to the QRS complex
B. Each QRS complex is preceded by a clearly visible P wave
C. The QRS complex is consistently narrow and uniform
D. The PR interval is prolonged but measurable
A. T waves are large and opposite in direction to the QRS complex
Rationale: PVCs originate from an ectopic ventricular focus, causing wide and distorted QRS complexes with large, inverted T waves due to abnormal repolarization. In contrast, a normal sinus beat has a narrow QRS complex, a measurable PR interval, and a P wave preceding each QRS.
A patient is admitted with palpitations and dizziness. The ECG shows an irregular rhythm with wide QRS complexes occurring prematurely without preceding P waves. Which finding would help differentiate unifocal from multifocal PVCs?
A. Unifocal PVCs always occur in a trigeminy pattern, while multifocal PVCs occur randomly
B. Unifocal PVCs are more dangerous than multifocal PVCs
C. Unifocal PVCs have QRS complexes that are identical in shape, while multifocal PVCs have varying shapes
D. Unifocal PVCs occur only in patients with CAD, while multifocal PVCs occur with electrolyte imbalances
C. Unifocal PVCs have QRS complexes that are identical in shape, while multifocal PVCs have varying shapes
Rationale: Unifocal PVCs arise from a single ectopic focus, producing PVCs with identical morphology on the ECG. Multifocal PVCs originate from multiple ectopic sites, resulting in PVCs with different shapes. Multifocal PVCs are generally more concerning because they indicate increased ventricular irritability and a higher risk for progression to ventricular tachycardia or fibrillation.
A patient’s 12-lead ECG reveals retrograde conduction with P waves appearing after premature wide QRS complexes. The PR interval is absent before these abnormal beats, and the rhythm is irregular. The nurse correctly identifies this as:
A. Atrial flutter
B. Premature ventricular contractions
C. Second-degree AV block, Mobitz I
D. Idioventricular rhythm
B. Premature ventricular contractions
Rationale: PVCs may cause retrograde conduction, where the P wave appears after the QRS complex instead of preceding it. This is due to abnormal depolarization and conduction within the ventricles. PVCs are distinguished from atrial flutter (which has sawtooth flutter waves), AV blocks (which have measurable PR intervals), and idioventricular rhythm (which is a slow, regular rhythm with absent P waves).
A nurse is evaluating an ECG strip of a patient experiencing frequent PVCs. The strip shows a normal sinus rhythm with occasional premature, wide QRS complexes that last longer than 0.12 seconds. The nurse also notes that these PVCs occur every third beat. What is the correct term for this pattern?
A. Ventricular bigeminy
B. Ventricular trigeminy
C. Ventricular couplet
D. Ventricular tachycardia
B. Ventricular trigeminy
Rationale:
PVCs are classified based on their frequency and pattern:
* Ventricular bigeminy: Every other beat is a PVC.
* Ventricular trigeminy: Every third beat is a PVC.
* Couplet: Two consecutive PVCs.
* Ventricular tachycardia: Three or more consecutive PVCs.
Since the PVCs occur every third beat, this is defined as ventricular trigeminy.
A nurse is assessing a patient with frequent PVCs. The patient has a history of coronary artery disease (CAD) and reports episodes of chest pain and dizziness. What is the nurse’s priority action?
A. Assess the patient’s apical-radial pulse deficit
B. Prepare for immediate defibrillation
C. Administer atropine IV push
D. Encourage the patient to bear down to stimulate the vagus nerve
A. Assess the patient’s apical-radial pulse deficit
Rationale: PVCs may reduce cardiac output (CO) because some contractions do not generate enough force to create a peripheral pulse. This can lead to angina, dizziness, or worsening heart failure in patients with CAD. Assessing the apical-radial pulse deficit helps determine if PVCs are compromising perfusion.
A patient with an acute MI is on continuous telemetry monitoring. The nurse observes frequent multifocal PVCs, and the patient reports feeling lightheaded. Which of the following interpretations is most accurate?
A. Multifocal PVCs indicate a stable rhythm with no need for intervention
B. The patient is at risk for developing ventricular tachycardia (VT) or ventricular fibrillation (VF)
C. Lightheadedness is unrelated to the PVCs and is likely due to dehydration
D. The PVCs are a normal response to myocardial infarction (MI) and do not affect cardiac output
B. The patient is at risk for developing ventricular tachycardia (VT) or ventricular fibrillation (VF)
Rationale: Multifocal PVCs indicate increased ventricular irritability, especially in the setting of acute MI. This increases the risk for ventricular tachycardia (VT) or ventricular fibrillation (VF), both of which can be life-threatening.
* (A) Multifocal PVCs are not stable rhythms—they require monitoring and possible intervention.
* (C) Lightheadedness can result from reduced CO due to frequent PVCs, not necessarily dehydration.
* (D) PVCs are not a normal compensatory response; they reflect ventricular dysfunction and may worsen cardiac ischemia.
A nurse is evaluating a patient’s telemetry strip and observes frequent PVCs. The patient has heart failure (HF) and reports increased fatigue and shortness of breath. What is the nurse’s priority action?
A. Assess for signs of worsening heart failure
B. Increase the patient’s fluid intake
C. Instruct the patient to perform deep breathing exercises
D. Prepare to administer atropine to increase heart rate
A. Assess for signs of worsening heart failure
Rationale: PVCs in a patient with heart failure (HF) can further reduce cardiac output, leading to worsening symptoms like fatigue, dyspnea, and poor perfusion. The nurse should assess for signs of fluid overload, worsening pulmonary congestion, or hypotension, as PVCs can contribute to hemodynamic instability.
A nurse is caring for a patient post-MI and notes frequent PVCs on the ECG. The provider orders serum electrolytes. Which electrolyte imbalance is most likely contributing to the PVCs?
A. Hyperkalemia
B. Hypermagnesemia
C. Hypernatremia
D. Hypokalemia
D. Hypokalemia
Rationale: Hypokalemia increases ventricular irritability, making PVCs more likely, particularly in post-MI patients who are already at risk for dysrhythmias.
A nurse is monitoring a patient’s telemetry and notices frequent PVCs occurring in a pattern of ventricular bigeminy. The patient has a history of coronary artery disease (CAD) and reports new-onset chest discomfort. Which intervention should the nurse anticipate first?
A. Administer atropine IV push
B. Instruct the patient to bear down (Valsalva maneuver)
C. Obtain a 12-lead ECG
D. Prepare for synchronized cardioversion
C. Obtain a 12-lead ECG
Rationale: Frequent PVCs in a pattern of bigeminy in a patient with CAD and new-onset chest discomfort may indicate worsening myocardial ischemia or an impending myocardial infarction (MI). A 12-lead ECG is essential to assess for ST-segment changes and determine the need for further interventions.
A patient with heart failure (HF) is experiencing frequent multifocal PVCs. The patient is hypotensive and diaphoretic. The nurse should expect which immediate intervention?
A. Administer IV amiodarone
B. Defibrillate the patient
C. Encourage coughing and deep breathing
D. Administer atropine IV push
A. Administer IV amiodarone
Rationale: Frequent multifocal PVCs in a hypotensive, diaphoretic heart failure patient indicate severe ventricular irritability, increasing the risk of ventricular tachycardia (VT) or ventricular fibrillation (VF). Amiodarone is an antiarrhythmic drug used to stabilize ventricular ectopy and reduce the risk of life-threatening dysrhythmias.
A patient reports palpitations and occasional dizziness. The nurse reviews the ECG and notes frequent R-on-T PVCs. What is the nurse’s priority action?
A. Monitor the patient closely and document findings
B. Notify the provider immediately
C. Perform carotid sinus massage
D. Administer a calcium channel blocker
B. Notify the provider immediately
Rationale: R-on-T PVCs are dangerous because they can trigger ventricular tachycardia (VT) or ventricular fibrillation (VF). This phenomenon occurs when a PVC falls on the T wave of the preceding beat, a vulnerable period in the cardiac cycle. Immediate intervention is necessary to prevent a lethal arrhythmia.
A nurse is reviewing the lab results of a patient experiencing frequent PVCs. Which electrolyte imbalance is the most concerning and requires immediate correction?
A. Serum potassium of 2.9 mEq/L
B. Serum sodium of 150 mEq/L
C. Serum calcium of 10.4 mg/dL
D. Serum magnesium of 2.2 mg/dL
A. Serum potassium of 2.9 mEq/L
Rationale: Hypokalemia (K+ < 3.5 mEq/L) increases ventricular irritability, predisposing the patient to PVCs, ventricular tachycardia (VT), and ventricular fibrillation (VF). Potassium plays a crucial role in cardiac repolarization, and low levels delay repolarization, leading to ectopic ventricular beats. Immediate potassium replacement is necessary to reduce the risk of life-threatening arrhythmias.
A patient is brought into the emergency department after experiencing syncope and palpitations. The ECG reveals monomorphic VT with regular QRS complexes. The patient is hemodynamically unstable with a blood pressure of 80/50 mmHg.
What is the initial intervention the nurse should anticipate?
A. Administer amiodarone IV
B. Perform synchronized cardioversion
C. Give lidocaine IV
D. Administer atropine IV
B. Perform synchronized cardioversion
Rationale: Synchronized cardioversion is the preferred treatment for monomorphic VT when the patient is hemodynamically unstable. It is performed to restore normal rhythm while ensuring synchronized timing to avoid inducing VF.
- Amiodarone (A) may be used for VT but is not the first-line treatment in the presence of hemodynamic instability.
- Lidocaine (C) is an antiarrhythmic used in VT but also not the first intervention in an unstable patient.
- Atropine (D) is used for bradyarrhythmias, not tachyarrhythmias like VT.
A patient has polymorphic VT with a prolonged QT interval on the ECG. What is the most likely associated condition?
A. Torsades de pointes
B. Hypokalemia
C. Myocardial infarction
D. Atrial fibrillation
A. Torsades de pointes
Rationale: Torsades de pointes is a form of polymorphic VT associated with a prolonged QT interval, which causes a twisting of the points pattern on the ECG.
- Hypokalemia (A) can lead to other arrhythmias but is not directly associated with torsades.
- Myocardial infarction (C) typically leads to monomorphic VT, not polymorphic VT with a prolonged QT.
- Atrial fibrillation (D) is a supraventricular arrhythmia, not a type of VT.
Which factors are risk factors for developing ventricular tachycardia (VT)? (SATA)
A. Myocardial infarction
B. Prolonged QT interval
C. Electrolyte imbalances
D. Bradycardia
E. Use of digoxin
A. Myocardial infarction
B. Prolonged QT interval
C. Electrolyte imbalances
E. Use of digoxin
Rationale:
* Myocardial infarction (A) and electrolyte imbalances (C) (such as hypokalemia or hypomagnesemia) are known triggers for VT.
* Prolonged QT interval (B) predisposes patients to polymorphic VT, particularly torsades de pointes.
* Digoxin (E) toxicity can lead to ventricular arrhythmias, including VT.
* Bradycardia (D) is generally not a direct risk factor for VT; rather, VT is more commonly associated with tachycardia or other conditions such as ischemia or structural heart disease.
A patient with a history of CAD and recent MI develops a nonsustained VT episode lasting 20 seconds. The nurse notices a drop in blood pressure and lightheadedness. What is the most appropriate action?
A. Monitor the patient and document the episode
B. Administer IV amiodarone
C. Prepare for immediate synchronized cardioversion
D. Administer 100% oxygen and obtain an ECG
D. Administer 100% oxygen and obtain an ECG
Rationale: For a nonsustained VT episode that causes symptoms, the first action should be to administer oxygen to improve oxygenation and obtain an ECG to evaluate the rhythm. This helps assess the severity and decide on further interventions.
* Amiodarone (B) may be appropriate later but is not the first step without further assessment.
* Synchronized cardioversion (C) is needed for sustained VT causing hemodynamic instability, but nonsustained VT requires more monitoring and assessment.
* Monitoring and documenting (A) is important, but the symptoms of lightheadedness and drop in BP suggest a need for further intervention.
What is the defining characteristic of monomorphic VT on the ECG?
A. Gradual changes in QRS shape, size, and direction
B. Frequent P waves preceding each QRS complex
C. A short QT interval with fast rates
D. Wide QRS complexes that are consistent in size, shape, and direction
D. Wide QRS complexes that are consistent in size, shape, and direction
Rationale: Monomorphic VT is characterized by wide, uniform QRS complexes that are consistent in shape, size, and direction.
* Polymorphic VT (A) is associated with gradual changes in the QRS shape and direction.
* Frequent P waves (C) are not seen in VT because it originates in the ventricles, not the atria.
* A short QT interval (D) is not a defining characteristic of VT; instead, a prolonged QT can be associated with torsades de pointes.
A patient is experiencing sustained VT with a heart rate of 150 bpm and is hypotensive. What is the most immediate intervention?
A. Administer a fluid bolus
B. Prepare for synchronized cardioversion
C. Administer IV atropine
D. Perform vagal maneuvers
B. Prepare for synchronized cardioversion
Rationale: In a patient with sustained VT and hemodynamic instability (hypotension), synchronized cardioversion is the most immediate intervention to restore normal rhythm and improve the patient’s hemodynamics.
* Fluid boluses (A) are useful for hypotension but are not the first-line treatment for VT.
* Atropine (C) is used for bradycardia, not for tachyarrhythmias like VT.
* Vagal maneuvers (D) are used for supraventricular tachycardias, not for ventricular arrhythmias like VT.
Which of the following are clinical manifestations of sustained ventricular tachycardia (VT)? (SATA)
A. Chest pain
B. Palpitations
C. Shortness of breath
D. Loss of consciousness
E. Bradycardia
A. Chest pain
B. Palpitations
C. Shortness of breath
D. Loss of consciousness
Rationale:
* Chest pain (A), palpitations (B), shortness of breath (C), and loss of consciousness (D) are common symptoms associated with sustained VT, which can cause reduced cardiac output.
* Bradycardia (E) is typically not seen in VT, as it is a tachyarrhythmia, not a bradyarrhythmia.
A patient with a history of long QT syndrome presents with palpitations and an ECG showing ventricular tachycardia (VT). The nurse knows that the patient’s long QT syndrome is most likely the underlying cause of the arrhythmia.
Which of the following interventions should be priority to reduce the risk of further complications?
A. Administer IV potassium
B. Initiate a lidocaine infusion
C. Avoid medications that prolong the QT interval
D. Prepare for immediate defibrillation
C. Avoid medications that prolong the QT interval
Rationale: Long QT syndrome increases the risk of developing torsades de pointes or other forms of polymorphic VT. The most effective way to manage this risk is to avoid QT-prolonging medications.
- Administering IV potassium (A) may be appropriate for some arrhythmias but does not directly address the underlying long QT syndrome.
- Lidocaine infusion (B) is used to treat VT but is not the first-line treatment in a patient with long QT syndrome.
- Immediate defibrillation (D) is indicated for ventricular fibrillation (VF), not for treating VT without signs of hemodynamic instability.
A patient with no history of heart disease presents with ventricular tachycardia (VT) on the ECG after experiencing a severe head injury. Which of the following conditions is most likely contributing to the development of VT in this patient?
A. Electrolyte imbalance
B. Drug toxicity
C. Central nervous system disorder
D. Coronary artery disease
C. Central nervous system disorder
Rationale: Central nervous system disorders, such as head trauma, can trigger ventricular arrhythmias, including VT, even in the absence of heart disease. The sympathetic nervous system can become overstimulated due to brain injury, leading to arrhythmias.
- Electrolyte imbalances (A) and drug toxicity (B) are common causes of VT but are not the most likely causes in this scenario based on the history of head injury.
- Coronary artery disease (D) is not indicated in a patient with no heart disease.
A patient with cardiomyopathy is being monitored for arrhythmias. The nurse notices ventricular tachycardia (VT) on the ECG. Which of the following is the most likely clinical association with the development of VT in this patient?
A. MI and CAD
B. Electrolyte imbalances
C. Long QT syndrome
D. Cardiomyopathy
D. Cardiomyopathy
Rationale: Cardiomyopathy is strongly associated with ventricular tachycardia (VT) due to structural heart changes that can lead to electrical disturbances in the ventricles.
- MI and CAD (A) may also cause VT but are not as directly related to cardiomyopathy as the primary cause.
- Electrolyte imbalances (B) and long QT syndrome (C) are potential causes of VT but are not the main association with cardiomyopathy.
A nurse is reviewing an ECG of a patient with ventricular tachycardia (VT). The ventricular rate is noted to be 180 beats per minute, and the rhythm is regular. The P waves appear to be buried in the QRS complex, and the PR interval is not measurable. Which of the following is the most likely explanation for these findings?
A. Complete AV block with retrograde conduction
B. Sinus tachycardia with an associated bundle branch block
C. Ventricular tachycardia with AV dissociation
D. Atrial fibrillation with ventricular response
C. Ventricular tachycardia with AV dissociation
Rationale: In ventricular tachycardia (VT), AV dissociation occurs when the atria and ventricles beat independently. The P waves may be buried in the QRS complex, and the PR interval is not measurable. This indicates that the atria and ventricles are not synchronized, which is characteristic of VT.
- Complete AV block with retrograde conduction (A) is a possible cause of AV dissociation, but the wide QRS and regular rhythm more strongly indicate VT.
- Sinus tachycardia with a bundle branch block (B) would not have the characteristic AV dissociation seen in VT.
- Atrial fibrillation with ventricular response (D) typically shows irregular rhythms and no clear P waves, unlike VT.
A nurse is monitoring a patient with ventricular tachycardia. The ECG shows a ventricular rate of 220 beats per minute and wide, distorted QRS complexes greater than 0.12 seconds in duration. The T wave is observed to be in the opposite direction of the QRS complex.
What is the most likely reason for the wide QRS complex?
A. Ventricular depolarization occurring asynchronously
B. Impulse originating from the AV node
C. Premature atrial contraction
D. Prolonged PR interval
A. Ventricular depolarization occurring asynchronously
Rationale: A wide QRS complex greater than 0.12 seconds is a hallmark of ventricular tachycardia (VT), where the ventricles depolarize asynchronously. This results in a distorted and wide QRS that is characteristic of VT.
- Impulse originating from the AV node (A) would not cause a wide QRS complex as seen in VT.
- A premature atrial contraction (C) would not cause a wide QRS complex; it typically produces a normal QRS.
- A prolonged PR interval (D) would affect the P wave to QRS relationship but not the QRS width.
A nurse observes an ECG of a patient with ventricular tachycardia (VT). The T waves are noted to be in the opposite direction of the QRS complexes. What does this finding most likely indicate?
A. Left bundle branch block
B. Presence of myocardial ischemia
C. Electrical discordance in VT
D. Hyperkalemia
C. Electrical discordance in VT
Rationale: In ventricular tachycardia (VT), the T waves are typically in the opposite direction of the QRS complex, which is indicative of electrical discordance. This is a common finding in VT due to the asynchronous depolarization of the ventricles.
- Left bundle branch block (A) would cause specific changes to the QRS morphology but not the T wave direction.
- Myocardial ischemia (B) can cause changes in ST segments but does not typically cause the opposite T wave direction seen in VT.
- Hyperkalemia (D) causes peaked T waves, not the opposite T wave direction seen in VT.
A nurse is reviewing an ECG that shows a ventricular tachycardia (VT) with a ventricular rate of 150 to 250 beats per minute and P waves that are buried in the QRS complex. The PR interval is not measurable, and the QRS complex is noted to be wide and distorted. What is the significance of these findings?
A. The patient is at risk for developing ventricular fibrillation
B. The patient is experiencing a stable VT
C. The rhythm is indicative of an atrial arrhythmia
D. The patient is experiencing sinus tachycardia
A. The patient is at risk for developing ventricular fibrillation
Rationale: Ventricular tachycardia (VT) with wide QRS complexes, buried P waves, and an unstable rhythm places the patient at high risk for progressing to ventricular fibrillation (VF), which is a life-threatening arrhythmia.
- Stable VT (A) would typically be characterized by a regular rhythm and no immediate risk for VF.
- Atrial arrhythmias (C) would show a different pattern, typically with irregular rhythms and P waves not buried in the QRS.
- Sinus tachycardia (D) would present with normal P waves and narrow QRS complexes, not the characteristics of VT.
The nurse is monitoring a patient with ventricular tachycardia (VT) on an ECG. The P waves are independent of the QRS complexes, and the PR interval is not measurable. What does this finding indicate?
A. There is a lack of communication between the atria and ventricles
B. Atrial and ventricular rhythms are synchronized
C. The heart rate is within normal limits
D. The QRS complex is originating from the SA node
A. There is a lack of communication between the atria and ventricles
Rationale: The lack of a measurable PR interval and P waves occurring independently of the QRS complexes indicate AV dissociation, which is characteristic of ventricular tachycardia (VT). This means that the atria and ventricles are not synchronized, and each is firing independently.
- Atrial and ventricular rhythms synchronized (A) would not show AV dissociation or buried P waves.
- Heart rate within normal limits (C) does not apply in VT, where the rate is typically 150-250 beats per minute.
- QRS originating from the SA node (D) would not result in the findings seen in VT, where the QRS is wide and distorted.
A nurse is interpreting an ECG showing ventricular tachycardia. The ventricular rate is noted to be 250 beats per minute, and the QRS complex is greater than 0.12 seconds in duration. The T wave is observed to be in the opposite direction of the QRS complex.
Which of the following interventions should be prioritized for this patient?
A. Immediate defibrillation
B. Administer IV amiodarone
C. Provide supplemental oxygen
D. Monitor for signs of bradycardia
A. Immediate defibrillation
Rationale: In ventricular tachycardia (VT), especially with ventricular rates exceeding 200 beats per minute, there is a high risk of progressing to ventricular fibrillation (VF). Immediate defibrillation is the priority intervention to prevent VF and restore a normal rhythm.
- IV amiodarone (B) may be used to manage VT in certain situations but is not the first-line intervention when VT is life-threatening.
- Supplemental oxygen (C) can be supportive but does not directly address the emergency of VT.
- Monitoring for bradycardia (D) is not applicable to VT, as this rhythm is tachycardic.
A nurse is caring for a patient with sustained ventricular tachycardia (VT). The patient is experiencing hypotension, pulmonary edema, and decreased cerebral blood flow. What is the most likely cause of these symptoms?
A. Decreased ventricular diastolic filling times and loss of atrial contraction
B. Increased sympathetic nervous system activation
C. Increased preload and afterload
D. Enhanced myocardial oxygen supply
A. Decreased ventricular diastolic filling times and loss of atrial contraction
Rationale: In sustained VT, the rapid heart rate reduces the ventricular diastolic filling time and causes a loss of atrial contraction, resulting in severe decreases in cardiac output (CO). This leads to hypotension, pulmonary edema, and decreased cerebral blood flow.
- Increased sympathetic nervous system activation (B) may occur in response to hypotension, but it is not the primary cause of the symptoms.
- Increased preload and afterload (C) are not the causes of the symptoms seen in sustained VT.
- Enhanced myocardial oxygen supply (D) is not related to the pathophysiology of sustained VT.
A patient with ventricular tachycardia (VT) has a pulse but is symptomatic with hypotension. What is the most appropriate initial treatment for this patient?
A. Immediate defibrillation
B. Antiarrhythmic medications such as amiodarone
C. Administering oxygen and fluids
D. Inserting a temporary pacemaker
B. Antiarrhythmic medications such as amiodarone
Rationale: For stable VT with a pulse and symptoms such as hypotension, the initial treatment is typically antiarrhythmic medications such as amiodarone to control the rhythm.
- Immediate defibrillation (A) is used for pulseless VT or ventricular fibrillation (VF), not for stable VT with a pulse.
- Administering oxygen and fluids (C) can be supportive but is not the primary treatment for managing VT.
- Inserting a temporary pacemaker (D) is typically not the first-line intervention for stable VT with a pulse.
A nurse is monitoring a patient who has experienced an episode of ventricular tachycardia (VT) that resolved on its own but is now at risk for recurrent episodes. What is the most appropriate action to reduce the risk of further episodes?
A. Initiate prophylactic antiarrhythmic therapy
B. Encourage the patient to increase physical activity
C. Monitor the patient for signs of bradycardia
D. Increase the patient’s fluid intake
A. Initiate prophylactic antiarrhythmic therapy
Rationale: To reduce the risk of recurrent episodes of VT, prophylactic antiarrhythmic therapy is recommended. This treatment helps prevent the recurrence of VT and ventricular fibrillation (VF).
- Increasing physical activity (B) could potentially exacerbate the arrhythmia and increase the risk of further episodes.
- Monitoring for bradycardia (C) is not appropriate, as VT is a tachycardic arrhythmia.
- Increasing fluid intake (D) may help with hypotension, but it does not directly address the need for antiarrhythmic therapy to prevent recurrent VT.
A patient with ventricular tachycardia (VT) is pulseless and requires immediate intervention. Which of the following actions should the nurse take first?
A. Administer intravenous amiodarone
B. Start chest compressions and prepare for defibrillation
C. Perform synchronized cardioversion
D. Insert a temporary pacemaker
B. Start chest compressions and prepare for defibrillation
Rationale: For a pulseless VT patient, the first action is to start chest compressions and prepare for defibrillation. This is the priority intervention to attempt to restore a normal rhythm and prevent cardiopulmonary arrest.
- Administering intravenous amiodarone (A) is appropriate for stable VT, but in the case of pulseless VT, defibrillation is the first line of treatment.
- Synchronized cardioversion (C) is used for unstable rhythms such as atrial fibrillation or SVT, but defibrillation is needed for pulseless VT.
- Inserting a temporary pacemaker (D) is not the initial treatment for pulseless VT, as defibrillation is the most critical intervention.
A nurse is caring for a patient with monomorphic ventricular tachycardia (VT) who is clinically stable with a pulse and preserved left ventricular function. Which of the following medications is appropriate for this patient?
A. IV procainamide
B. IV magnesium
C. IV isoproterenol
D. IV amiodarone
A. IV procainamide
Rationale: For monomorphic VT in a clinically stable patient with a pulse and preserved left ventricular function, IV procainamide is an appropriate treatment.
- IV magnesium (B) is used for polymorphic VT with a prolonged QT interval, not for monomorphic VT.
- IV isoproterenol (C) is used for polymorphic VT with a prolonged QT interval, not for monomorphic VT.
- IV amiodarone (D) can also be used for monomorphic VT, but procainamide is the preferred option for this specific case.
A patient with polymorphic ventricular tachycardia (VT) has a prolonged baseline QT interval. Which of the following treatments should the nurse administer?
A. IV procainamide
B. IV lidocaine
C. IV amiodarone
D. IV magnesium
D. IV magnesium
Rationale: For polymorphic VT with a prolonged baseline QT interval, the recommended treatment is IV magnesium.
- IV procainamide (A) and IV lidocaine (C) are used for monomorphic VT, not for polymorphic VT with a prolonged QT interval.
- IV amiodarone (D) may be used for VT but is not specifically recommended for polymorphic VT with a prolonged QT interval.
A patient is experiencing polymorphic VT and has a normal baseline QT interval. Which of the following treatments would be most appropriate?
A. IV magnesium
B. IV isoproterenol
C. IV procainamide
D. Antitachycardia pacing
C. IV procainamide
Rationale: For polymorphic VT with a normal baseline QT interval, IV procainamide is an appropriate treatment.
- IV magnesium (A) is used for polymorphic VT with a prolonged QT interval, not for those with a normal QT interval.
- IV isoproterenol (C) and antitachycardia pacing (D) are used for polymorphic VT with a prolonged QT interval, not for those with a normal QT interval.
A patient with ventricular tachycardia (VT) without a pulse is unresponsive to defibrillation. What is the next appropriate action?
A. Administer IV lidocaine
B. Administer vasopressors such as epinephrine
C. Administer IV procainamide
D. Initiate synchronized cardioversion
B. Administer vasopressors such as epinephrine
Rationale: For pulseless VT, after defibrillation fails, the next step is to administer vasopressors such as epinephrine. This helps to improve circulation during CPR.
- IV lidocaine (A) and IV procainamide (C) are used for VT with a pulse, not for pulseless VT.
- Synchronized cardioversion (D) is used for unstable rhythms with a pulse, not for pulseless VT.
A patient with VT is unresponsive and has no pulse. What is the priority intervention?
A. Administer antidysrhythmics
B. Start IV fluids and electrolytes
C. Perform synchronized cardioversion
D. Initiate CPR and prepare for defibrillation
D. Initiate CPR and prepare for defibrillation
Rationale: For a pulseless VT patient, the priority intervention is to initiate CPR and prepare for defibrillation, which is the first line of treatment.
- Administering antidysrhythmics (A) and starting IV fluids (C) are appropriate after defibrillation if necessary, but defibrillation comes first.
- Synchronized cardioversion (D) is used for unstable rhythms with a pulse, not for pulseless VT.
A nurse is caring for a patient with ventricular tachycardia (VT). The patient’s baseline QT interval is normal. The physician orders IV lidocaine for treatment. Which of the following actions should the nurse take?
A. Administer the medication as ordered, as it is effective for VT
B. Monitor the patient for signs of QT prolongation
C. Verify the physician’s order because lidocaine is not appropriate for VT
D. Monitor the patient for signs of bradycardia
A. Administer the medication as ordered, as it is effective for VT
Rationale: IV lidocaine is appropriate for treating VT with a normal baseline QT interval and can be safely administered as ordered.
- Monitoring for signs of QT prolongation (B) is not necessary in this case, as lidocaine does not cause QT prolongation.
- Verifying the physician’s order (C) is unnecessary, as lidocaine is a standard treatment for VT.
- Monitoring for signs of bradycardia (D) is not the primary concern for lidocaine, as it typically causes tachycardia, not bradycardia
A patient with polymorphic VT has a prolonged baseline QT interval. Which of the following interventions is most appropriate for this condition?
A. Administering IV procainamide
B. Administering IV magnesium
C. Initiating synchronized cardioversion
D. Administering lidocaine
B. Administering IV magnesium
Rationale: For polymorphic VT with a prolonged baseline QT interval, the treatment of choice is IV magnesium, which helps stabilize the rhythm.
- IV procainamide (A) and lidocaine (D) are not appropriate for polymorphic VT with a prolonged QT interval.
- Synchronized cardioversion (C) may be used for unstable rhythms, but the first-line treatment for polymorphic VT with a prolonged QT is IV magnesium.
A patient with ventricular tachycardia (VT) requires treatment. The patient has no pulse, and initial defibrillation is unsuccessful. What is the next step in managing the patient?
A. Administer IV atropine
B. Administer IV amiodarone
C. Administer epinephrine and continue CPR
D. Perform synchronized cardioversion
C. Administer epinephrine and continue CPR
Rationale: For a pulseless VT patient who is unresponsive to defibrillation, the next step is to administer epinephrine and continue CPR to improve circulation and prepare for the next defibrillation attempt.
- IV atropine (A) is used for bradycardia, not pulseless VT.
- IV amiodarone (B) may be used for VT with a pulse, but it is not the immediate next step for pulseless VT.
- Synchronized cardioversion (D) is used for unstable rhythms with a pulse, not for pulseless VT.
A nurse is assessing a patient with ventricular fibrillation (VF). The ECG shows irregular waveforms of varying shapes and amplitudes. What is the primary physiological consequence of VF?
A. Decreased ventricular diastolic filling
B. No effective contraction of the ventricles
C. Increased cardiac output (CO)
D. Normal electrical conduction through the heart
B. No effective contraction of the ventricles
Rationale: In ventricular fibrillation (VF), the ventricles quiver with no effective contraction, leading to the absence of cardiac output (CO).
* Decreased ventricular diastolic filling (A) may occur in other dysrhythmias but is not the primary cause in VF.
* Increased CO (C) does not occur in VF; CO is essentially absent.
* Normal electrical conduction (D) is disrupted in VF, as it is characterized by irregular, erratic electrical activity.
A patient is diagnosed with ventricular fibrillation (VF). Which of the following is the most appropriate immediate intervention?
A. Administer vasopressors
B. Perform synchronized cardioversion
C. Initiate CPR and prepare for defibrillation
D. Administer antidysrhythmics
C. Initiate CPR and prepare for defibrillation
Rationale: The immediate intervention for VF is to initiate CPR and prepare for defibrillation, as it is a life-threatening arrhythmia.
- Administering vasopressors (A) is part of the treatment after defibrillation, not the first step.
- Synchronized cardioversion (B) is used for rhythms with a pulse, not for pulseless VF.
- Antidysrhythmics (D) are not the first-line treatment for VF; defibrillation is.
A patient with ventricular fibrillation (VF) requires defibrillation. What is the priority in the management of this condition?
A. Administer oxygen
B. Monitor for QT interval prolongation
C. Rapid defibrillation
D. Administer IV magnesium
C. Rapid defibrillation
Rationale: The priority in the management of VF is rapid defibrillation to restore a normal rhythm.
- Administering oxygen (A) is important but should not delay defibrillation.
- Monitoring for QT interval prolongation (B) is not a primary focus in VF; the priority is defibrillation.
- Administering IV magnesium (D) is used for certain dysrhythmias like polymorphic VT with prolonged QT, but not for VF.
Which of the following ECG findings is characteristic of ventricular fibrillation (VF)?
A. Regular, evenly spaced QRS complexes
B. Irregular waveforms of varying shapes and amplitudes
C. P waves and QRS complexes occurring at a fixed interval
D. A narrow, uniform QRS complex
B. Irregular waveforms of varying shapes and amplitudes
Rationale: Ventricular fibrillation (VF) is characterized by irregular waveforms of varying shapes and amplitudes, which represent chaotic electrical activity.
- Regular, evenly spaced QRS complexes (A) are not seen in VF.
- P waves and QRS complexes occurring at a fixed interval (C) would indicate a regular rhythm, not VF.
- A narrow, uniform QRS complex (D) is typical of normal rhythms, not VF.
A nurse is educating a patient’s family about ventricular fibrillation (VF). What is the most important piece of information to convey regarding the prognosis of VF?
A. VF can be controlled with medications alone
B. VF is a lethal dysrhythmia that requires immediate treatment
C. VF is a benign condition that resolves on its own
D. VF is most commonly caused by electrolyte imbalances
B. VF is a lethal dysrhythmia that requires immediate treatment
Rationale: Ventricular fibrillation (VF) is a lethal dysrhythmia that requires immediate treatment to prevent death.
- Medications alone (A) are not sufficient to treat VF; defibrillation is the first-line treatment.
- VF is not benign (C) and will not resolve without treatment.
- Electrolyte imbalances (D) can contribute to VF, but the primary cause is typically electrical instability in the ventricles, not always an imbalance.
A nurse is caring for a patient who recently experienced a myocardial infarction (MI) and is at risk for developing ventricular fibrillation (VF). Which of the following is a potential cause of VF in this patient?
A. Normal cardiac conduction
B. Decreased oxygen supply to the heart
C. Increased blood pressure
D. Hyperkalemia
D. Hyperkalemia
Rationale: Hyperkalemia is a recognized risk factor for ventricular fibrillation (VF) due to its effects on the electrical conductivity of the heart.
- Normal cardiac conduction (A) would not lead to VF.
- While decreased oxygen supply (C) to the heart can contribute to ischemia and arrhythmias, hyperkalemia (B) directly affects the electrical system and increases VF risk.
- Increased blood pressure (D) is not a typical direct cause of VF.
A patient with chronic heart failure (HF) is at increased risk for ventricular fibrillation (VF). Which of the following factors most likely contributes to the development of VF in this patient?
A. Impaired ventricular function leading to electrical instability
B. Increased blood flow to the myocardium
C. Low serum potassium levels
D. Effective cardiac pacing
A. Impaired ventricular function leading to electrical instability
Rationale: Impaired ventricular function in heart failure (HF) leads to electrical instability, increasing the risk of ventricular fibrillation (VF).
- Increased blood flow to the myocardium (B) is generally beneficial and does not contribute to VF.
- Low serum potassium (C) can increase the risk of arrhythmias, but it is not the primary factor for VF in HF.
- Effective cardiac pacing (D) helps to regulate the heart rhythm, reducing the risk of VF.
A patient undergoing cardiac catheterization develops ventricular fibrillation (VF) during the procedure. Which of the following is the most likely cause of VF in this scenario?
A. Stimulation of the ventricle by the catheter
B. Improved myocardial oxygenation
C. Administration of anticoagulants
D. Increased sympathetic nervous system activity
A. Stimulation of the ventricle by the catheter
Rationale: During cardiac catheterization, stimulation of the ventricle by the catheter is a known cause of ventricular fibrillation (VF).
- Improved myocardial oxygenation (B) would reduce the likelihood of arrhythmias, not increase it.
- Administration of anticoagulants (C) can increase bleeding risk but is not a primary cause of VF.
- Increased sympathetic nervous system activity (D) can contribute to arrhythmias but is less directly linked to VF in this context.
A nurse is monitoring a patient who has just received thrombolytic therapy for a myocardial infarction (MI). The nurse understands that ventricular fibrillation (VF) may develop in this patient due to:
A. The reperfusion of coronary arteries
B. Decreased myocardial oxygen demand
C. Reduction in ventricular filling time
D. Increased heart rate from the thrombolytic medication
A. The reperfusion of coronary arteries
Rationale: Reperfusion of coronary arteries after thrombolytic therapy can lead to ventricular fibrillation (VF) due to the sudden changes in myocardial electrical stability.
- Decreased myocardial oxygen demand (B) typically occurs as a result of successful reperfusion, but it does not directly cause VF.
- Reduction in ventricular filling time (C) is related to ventricular dysfunction but not directly linked to VF.
- Increased heart rate (D) can be a side effect of certain medications but is not a primary cause of VF in this scenario.
A patient is found to be unresponsive, pulseless, and apneic. The ECG shows an irregular and chaotic rhythm with no visible P waves, and the QRS complex is not measurable. What is the most likely diagnosis for this patient?
A. Atrial fibrillation
B. Ventricular fibrillation
C. Sinus arrest
D. Normal sinus rhythm
B. Ventricular fibrillation
Rationale: The described ECG findings are characteristic of ventricular fibrillation (VF), which is marked by an irregular and chaotic rhythm, with no measurable P wave, PR interval, or QRS interval.
- Atrial fibrillation (A) has a chaotic rhythm but still has visible P waves, though they are not clearly defined.
- Sinus arrest (C) involves a pause in the normal rhythm, but it does not present with the chaotic rhythm described here.
- Normal sinus rhythm (D) would have a regular rhythm with measurable P waves, PR intervals, and QRS intervals.
A nurse is assessing a patient experiencing ventricular fibrillation (VF). Which of the following is the immediate priority for treatment?
A. Administer IV amiodarone
B. Administer oxygen via non-rebreather mask
C. Begin chest compressions
D. Prepare for synchronized cardioversion
C. Begin chest compressions
Rationale: For ventricular fibrillation (VF), immediate chest compressions are the priority, as they help maintain circulation and provide some perfusion to vital organs until defibrillation can be performed.
- Administering IV amiodarone (A) and oxygen therapy (C) are important but come after chest compressions in the emergency treatment sequence.
- Synchronized cardioversion (D) is used for certain arrhythmias but is not the initial treatment for VF.
In the treatment of ventricular fibrillation (VF), what is the primary role of defibrillation?
A. To restore the heart’s electrical activity to normal sinus rhythm
B. To deliver a controlled shock that temporarily stops the heart
C. To stimulate the atria to initiate coordinated electrical activity
D. To increase the heart rate in bradycardic patients
A. To restore the heart’s electrical activity to normal sinus rhythm
Rationale: Defibrillation aims to deliver an electric shock to the heart to restore normal electrical activity and stop the chaotic rhythm seen in ventricular fibrillation (VF).
- Defibrillation (B) does not temporarily stop the heart; it is intended to reset the electrical activity, not just halt it.
- Stimulating the atria (C) is not the goal in VF treatment, as the ventricular rhythm must be corrected.
- Increasing heart rate (D) applies to conditions of bradycardia, not VF.
A patient is diagnosed with ventricular fibrillation (VF). Which of the following clinical signs would most likely be present in this patient?
A. Regular pulse with a rate of 70 beats per minute
B. Slow and irregular pulse with low blood pressure
C. Loud heart sounds with a bounding pulse
D. Absence of a pulse, unresponsiveness, and apnea
D. Absence of a pulse, unresponsiveness, and apnea
Rationale: Ventricular fibrillation (VF) leads to complete loss of effective cardiac output, resulting in no pulse, unresponsiveness, and apnea.
- Regular pulse and normal blood pressure (A) are not consistent with VF, which causes cardiac arrest.
- Slow and irregular pulse (C) is seen in arrhythmias like bradycardia or heart block, but VF leads to pulselessness.
- Bounding pulse (D) would indicate good circulation, which is not present in VF.
During a code blue scenario, a patient is in ventricular fibrillation (VF). The nurse prepares to administer epinephrine. What is the primary reason for giving this drug in VF treatment?
A. To increase the heart rate and restore rhythm
B. To stimulate the pacemaker cells in the atria
C. To promote vasoconstriction and improve coronary perfusion
D. To correct electrolyte imbalances in the heart
C. To promote vasoconstriction and improve coronary perfusion
Rationale: Epinephrine is administered in ventricular fibrillation (VF) to promote vasoconstriction, which helps improve coronary perfusion during CPR, increasing the chances of successful defibrillation.
- Increasing heart rate (A) is not the primary effect of epinephrine; its role is improving perfusion.
- Stimulating pacemaker cells (B) is not the purpose of epinephrine in VF.
- Correcting electrolyte imbalances (D) is not the primary action of epinephrine.
A nurse is caring for a patient in ventricular fibrillation (VF). Which of the following interventions is most important to implement immediately?
A. Administering intravenous fluids to improve circulation
B. Performing chest compressions while awaiting defibrillation
C. Giving aspirin to prevent blood clots
D. Checking the patient’s blood pressure
B. Performing chest compressions while awaiting defibrillation
Rationale: Chest compressions are the first and most important step in ventricular fibrillation (VF) treatment to maintain circulation and perfusion until defibrillation can be performed.
- Administering fluids (A) may be helpful in other cases but is not a priority in VF treatment.
- Giving aspirin (C) is not a priority in VF, as the immediate goal is to restore the heart’s rhythm.
- Checking blood pressure (D) is unnecessary when the patient is pulseless.
Which of the following is a characteristic finding on an ECG during ventricular fibrillation (VF)?
A. Wide, regular QRS complexes
B. Rapid, irregular ventricular rate with no identifiable P waves
C. Slow, regular rhythm with measurable PR intervals
D. Normal sinus rhythm with a regular heart rate
B. Rapid, irregular ventricular rate with no identifiable P waves
Rationale: During ventricular fibrillation (VF), the ECG shows a rapid, irregular ventricular rate with no identifiable P waves, PR interval, or QRS complex.
- Wide, regular QRS complexes (A) are seen in other arrhythmias but not in VF.
- Slow, regular rhythm (C) is not typical of VF, which is chaotic and irregular.
- Normal sinus rhythm (D) is a healthy rhythm, not associated with VF.
A patient has just been diagnosed with ventricular fibrillation (VF). Which of the following is the first step in the ACLS protocol for treating VF?
A. Administer a defibrillating shock immediately
B. Start high-dose epinephrine therapy
C. Assess the patient’s airway
D. Begin CPR with chest compressions
D. Begin CPR with chest compressions
Rationale: The first step in the ACLS protocol for ventricular fibrillation (VF) is to begin CPR with chest compressions, which helps maintain circulation until defibrillation can be performed.
- Defibrillation (A) is performed after CPR has been initiated.
- Epinephrine (B) is given after chest compressions, not immediately.
- Assessing the airway (D) is important but should be done simultaneously with chest compressions.
A patient is found unresponsive, pulseless, and apneic. The ECG shows the absence of any ventricular electrical activity, with occasional P waves present. What is the most likely diagnosis?
A. Ventricular fibrillation
B. Atrial fibrillation
C. Sinus bradycardia
D. Asystole
D. Asystole
Rationale: The description of no ventricular electrical activity with occasional P waves and the absence of ventricular contraction is characteristic of asystole. In this condition, there is no electrical activity in the ventricles, and the patient is unresponsive, pulseless, and apneic.
- Ventricular fibrillation (A) presents with chaotic, irregular electrical activity in the ventricles, not as a flatline with P waves.
- Atrial fibrillation (C) shows irregularly irregular rhythms and rapid atrial activity, but not a flatline pattern.
- Sinus bradycardia (D) involves a slow, regular rhythm, which would not lead to a flatline appearance.
A patient presents with asystole. Which of the following is the most important action for the healthcare provider to take?
A. Administer a defibrillating shock
B. Start chest compressions immediately
C. Administer high-dose epinephrine
D. Perform synchronized cardioversion
B. Start chest compressions immediately
Rationale: In the case of asystole, chest compressions should be started immediately as part of the ACLS protocol to provide circulation until other interventions can be initiated.
- Defibrillation (A) is not indicated in asystole, as there is no electrical activity to reset.
- High-dose epinephrine (C) is an appropriate treatment after chest compressions have been initiated, but it is not the first step.
- Synchronized cardioversion (D) is used for certain arrhythmias but not for asystole.
A nurse is assessing a patient who is unresponsive and pulseless. The ECG shows a flatline with occasional P waves. Which of the following should the nurse do next?
A. Assess the rhythm in multiple leads
B. Administer intravenous amiodarone
C. Continue monitoring for 5 minutes
D. Prepare for synchronized cardioversion
A. Assess the rhythm in multiple leads
Rationale: Before diagnosing asystole, the nurse should assess the rhythm in more than one lead, as ventricular fibrillation (VF) can occasionally masquerade as asystole in a single lead.
- Administering amiodarone (B) is appropriate for treating ventricular arrhythmias, not asystole.
- Monitoring (C) for 5 minutes is too long to wait in an emergency situation like this. Immediate action is needed.
- Synchronized cardioversion (D) is used for certain arrhythmias but not for asystole.
Which of the following is a key characteristic of asystole on an ECG?
A. Chaotic, irregular rhythm with no identifiable P waves
B. A flatline with occasional P waves
C. Regular, slow rhythm with measurable PR intervals
D. Rapid, irregular rhythm with wide QRS complexes
B. A flatline with occasional P waves
Rationale: Asystole is characterized by a flatline on the ECG, with occasional P waves sometimes seen, but there is no ventricular electrical activity or contraction.
- Chaotic and irregular rhythm (A) describes ventricular fibrillation, not asystole.
- Regular, slow rhythm (C) is seen in sinus bradycardia, not asystole.
- Rapid, irregular rhythm with wide QRS complexes (D) is characteristic of ventricular tachycardia, not asystole.
A patient with asystole is unresponsive, pulseless, and apneic. The prognosis for this patient is typically:
A. Excellent if treated within the first few minutes
B. Good if defibrillation is applied immediately
C. Very poor, with minimal chance of recovery
D. Moderate, depending on the underlying cause
C. Very poor, with minimal chance of recovery
Rationale: The prognosis for a patient in asystole is generally very poor, as it is a lethal dysrhythmia with minimal chances of recovery if not treated immediately and effectively.
- Excellent prognosis (A) is unlikely in asystole, especially without rapid intervention.
- Defibrillation (C) is not effective in asystole, as there is no electrical activity to reset.
- Moderate prognosis (D) may apply to other conditions but not to asystole, where the prognosis is extremely poor.
A patient with a history of advanced heart disease is found unresponsive, pulseless, and apneic. The ECG shows asystole. Which of the following is most likely the cause of this patient’s condition?
A. Myocardial infarction
B. End-stage heart failure
C. Atrial fibrillation
D. Ventricular tachycardia
B. End-stage heart failure
Rationale: Asystole is often a result of advanced heart disease, severe conduction system issues, or end-stage heart failure (HF). This patient’s history of heart disease makes end-stage HF a likely cause.
- Myocardial infarction (A) can lead to arrhythmias, but asystole is more commonly associated with end-stage heart disease or prolonged arrest.
- Atrial fibrillation (C) and ventricular tachycardia (D) are distinct arrhythmias that would not typically lead directly to asystole.
A patient in asystole is receiving CPR and ACLS measures. Which of the following treatments is considered first-line for this condition?
A. Defibrillation
B. Epinephrine
C. Amiodarone
D. Lidocaine
B. Epinephrine
Rationale: For asystole, epinephrine is the first-line drug used during CPR and ACLS measures to stimulate cardiac activity.
- Defibrillation (A) is not indicated in asystole because there is no electrical activity to reset.
- Amiodarone (C) and Lidocaine (D) are used for arrhythmias like ventricular fibrillation or ventricular tachycardia, not asystole.
A patient with asystole is unresponsive and pulseless. Which of the following is the most appropriate next step in treatment?
A. Intubation and ventilation
B. Synchronized cardioversion
C. Chest compressions with CPR
D. Immediate defibrillation
C. Chest compressions with CPR
Rationale: For asystole, chest compressions with CPR should be started immediately to maintain circulation. This is the most crucial first step.
- Intubation (A) may be necessary later, but chest compressions are the priority.
- Synchronized cardioversion (B) is used for certain arrhythmias, not asystole.
- Defibrillation (D) is ineffective in asystole since there is no electrical activity to reset.
A patient with asystole is being resuscitated. The healthcare team is attempting to identify the underlying cause. Which of the following is a common cause of asystole that should be ruled out?
A. Hyperkalemia
B. Hypertension
C. Atrial fibrillation
D. Left-sided heart failure
A. Hyperkalemia
Rationale: Hyperkalemia is a known cause of asystole and should be ruled out as part of the underlying cause.
- Hypertension (B), atrial fibrillation (C), and left-sided heart failure (D) are not direct causes of asystole, though they can contribute to heart failure or other conditions that increase the risk of asystole.
A patient in asystole is unresponsive and pulseless. The healthcare team has already initiated CPR and started epinephrine. What is the next step in management?
A. Prepare for synchronized cardioversion
B. Administer sodium bicarbonate
C. Assess the rhythm in more than one lead
D. Continue chest compressions and assess for a pulse
D. Continue chest compressions and assess for a pulse
Rationale: The most important action is to continue chest compressions and assess for a pulse regularly. This provides the necessary circulation and ensures proper perfusion during the resuscitation effort.
- Synchronized cardioversion (A) is not used for asystole.
- Sodium bicarbonate (B) is not typically used in the initial treatment of asystole.
- Assessing the rhythm in more than one lead (C) should have been done early in the process to confirm the diagnosis of asystole.
A patient is in pulseless electrical activity (PEA), with organized electrical activity on the ECG but no pulse. What is the most important intervention in the management of this condition?
A. Immediate defibrillation
B. Administration of sodium bicarbonate
C. Correcting the underlying cause
D. Insertion of a temporary pacemaker
C. Correcting the underlying cause
Rationale: The most critical step in managing PEA is identifying and correcting the underlying cause. This can significantly impact prognosis.
- Immediate defibrillation (A) is not effective in PEA since there is organized electrical activity, not a shockable rhythm.
- Sodium bicarbonate (B) is not indicated unless acidosis is present.
- Inserting a temporary pacemaker (D) is not a first-line treatment for PEA.
Which of the following is a common cause of pulseless electrical activity (PEA)?
A. Atrial fibrillation
B. Coronary artery disease
C. Hyperkalemia
D. Ventricular fibrillation
C. Hyperkalemia
Rationale: Hyperkalemia is a common cause of PEA. Other causes include hypovolemia, hypoxia, metabolic acidosis, and more.
- Atrial fibrillation (A) and ventricular fibrillation (D) are not causes of PEA but are other types of dysrhythmias.
- Coronary artery disease (B) can lead to PEA if it results in conditions like myocardial infarction (thrombosis).
A patient is experiencing pulseless electrical activity (PEA). After starting CPR, which drug should be administered to the patient?
A. Lidocaine
B. Amiodarone
C. Magnesium sulfate
D. Epinephrine
D. Epinephrine
Rationale: The first-line drug for PEA is epinephrine, which should be administered as part of CPR to stimulate cardiac activity and improve prognosis.
- Lidocaine (A) and amiodarone (C) are used for other arrhythmias, but not for PEA.
- Magnesium sulfate (D) is used for certain arrhythmias, such as torsades de pointes, but not for PEA.
A patient in pulseless electrical activity (PEA) has no pulse despite organized electrical activity on the ECG. The most common cause of PEA in this patient is hypovolemia. Which of the following is the initial treatment for this condition?
A. Immediate defibrillation
B. Administration of fluid resuscitation
C. Administration of magnesium sulfate
D. Antitachycardia pacing
B. Administration of fluid resuscitation
Rationale: In hypovolemia-induced PEA, the priority treatment is fluid resuscitation to restore circulating blood volume.
- Immediate defibrillation (A) is not effective in PEA.
- Magnesium sulfate (C) is not the first treatment for hypovolemia.
- Antitachycardia pacing (D) is not used for PEA.
A patient is in pulseless electrical activity (PEA) after a suspected drug overdose. Which of the following should be prioritized in the treatment plan?
A. Drug therapy and intubation
B. Rapid defibrillation
C. Pacemaker insertion
D. Correction of electrolyte imbalances
A. Drug therapy and intubation
Rationale: For PEA caused by drug overdose, drug therapy (e.g., epinephrine) and intubation are necessary to manage the patient.
- Rapid defibrillation (A) is not appropriate for PEA, as it is not a shockable rhythm.
- Pacemaker insertion (C) is not indicated for PEA.
- Correcting electrolyte imbalances (D) is important but is part of managing the underlying cause.
A patient with pulseless electrical activity (PEA) is undergoing resuscitation. What is the primary goal of treatment in this scenario?
A. Restoring electrical activity
B. Preventing further arrhythmias
C. Restoring mechanical cardiac function
D. Correcting the cause of PEA
C. Restoring mechanical cardiac function
Rationale: The primary goal in PEA is to restore mechanical cardiac function, as there is electrical activity but no effective cardiac contraction.
- Restoring electrical activity (A) is not the main goal; mechanical function must be restored for effective circulation.
- Preventing further arrhythmias (B) and correcting the cause (D) are important but secondary to restoring cardiac output.
A patient in pulseless electrical activity (PEA) is found to have a tension pneumothorax. Which intervention should be prioritized to improve the patient’s condition?
A. Intubation and mechanical ventilation
B. Administration of sodium bicarbonate
C. Insertion of a chest tube
D. Immediate defibrillation
C. Insertion of a chest tube
Rationale: For a tension pneumothorax, the priority intervention is to insert a chest tube to relieve the pressure and restore normal circulation.
- Intubation (A) and mechanical ventilation may be needed but are secondary to treating the tension pneumothorax.
- Sodium bicarbonate (C) and defibrillation (D) are not indicated for tension pneumothorax.
A patient in pulseless electrical activity (PEA) is undergoing resuscitation. Which of the following underlying conditions must be ruled out to improve the patient’s prognosis?
A. Hyperkalemia
B. Hypertension
C. Atrial fibrillation
D. Coronary artery disease
A. Hyperkalemia
Rationale: Hyperkalemia is one of the common causes of PEA, and it must be identified and corrected to improve the prognosis.
- Hypertension (B), atrial fibrillation (C), and coronary artery disease (D) may predispose to other arrhythmias but are not the most common causes of PEA.
A 72-year-old patient with a history of heart failure is admitted with atrial fibrillation. The provider prescribes amiodarone. The nurse prepares to administer the first dose. Which priority assessment should the nurse complete before giving the medication?
A. Assess for signs of hyperthyroidism
B. Obtain baseline ECG and vital signs
C. Evaluate the patient’s ability to swallow pills
D. Assess the patient’s history of alcohol use
B. Obtain baseline ECG and vital signs
Rationale: Before administering any antidysrhythmic medication, the nurse must obtain a baseline ECG to assess for preexisting conduction abnormalities and monitor for changes after administration. Vital signs, especially heart rate and blood pressure, are essential to evaluate the patient’s stability and response to treatment. Amiodarone has significant effects on cardiac conduction, so baseline data are crucial.
A nurse is educating a patient prescribed an oral antidysrhythmic drug. Which statements indicate appropriate understanding of the teaching? (SATA)
A. “I should take this medication on an empty stomach to improve absorption.”
B. “I will check my pulse daily and report any irregularities to my healthcare provider.”
C. “I need to avoid drinking grapefruit juice while taking this medication.”
D. “If I develop a dry cough or shortness of breath, I should notify my provider immediately.”
E. “I can continue to smoke as long as I monitor my heart rate.”
B. “I will check my pulse daily and report any irregularities to my healthcare provider.”
C. “I need to avoid drinking grapefruit juice while taking this medication.”
D. “If I develop a dry cough or shortness of breath, I should notify my provider immediately.”
Rationale:
* B: Patients taking antidysrhythmic medications should monitor their pulse daily, as these drugs can alter heart rate and rhythm.
* C: Grapefruit juice can interfere with the metabolism of certain antidysrhythmics, such as amiodarone, leading to toxicity.
* D: Antidysrhythmic drugs, particularly amiodarone, can cause pulmonary toxicity, which may present as a dry cough or shortness of breath. This requires immediate medical attention.
* A and E are incorrect because most antidysrhythmic drugs are better tolerated with food to prevent GI upset, and smoking can exacerbate cardiac dysrhythmias.
A patient in the ICU is receiving IV lidocaine for ventricular tachycardia. The nurse notices new-onset confusion and slurred speech. What is the nurse’s best action?
A. Stop the infusion and notify the provider
B. Continue the infusion but monitor the ECG closely
C. Administer a bolus of lidocaine to achieve a therapeutic effect
D. Encourage the patient to rest and reassess in 30 minutes
A. Stop the infusion and notify the provider
Rationale: Lidocaine toxicity can present as confusion, slurred speech, dizziness, and even seizures. The nurse should immediately stop the infusion and notify the provider to prevent further complications. Continuing the infusion or delaying intervention could worsen neurotoxicity and cardiac effects.
A patient receiving amiodarone reports persistent fatigue, weight gain, and cold intolerance. The nurse suspects which adverse effect of the medication?
A. Hepatic dysfunction
B. Thyroid dysfunction
C. Pulmonary fibrosis
D. Optic neuropathy
B. Thyroid dysfunction
Rationale: Amiodarone contains iodine and can cause hypo- or hyperthyroidism. Symptoms such as fatigue, weight gain, and cold intolerance suggest hypothyroidism, a common adverse effect. The nurse should notify the provider and expect thyroid function tests.
A nurse is monitoring a patient who just received IV adenosine for supraventricular tachycardia (SVT). Which of the following actions should the nurse take? (SATA)
A. Monitor the ECG for asystole and bradycardia
B. Have emergency equipment readily available
C. Administer the drug slowly over 1-2 minutes
D. Assess for flushing, dizziness, or chest discomfort
E. Repeat the dose immediately if the first attempt fails
A. Monitor the ECG for asystole and bradycardia
B. Have emergency equipment readily available
D. Assess for flushing, dizziness, or chest discomfort
Rationale:
* A: Adenosine causes a brief period of asystole before converting the rhythm. This is expected but should be closely monitored.
* B: Emergency equipment should always be available in case the patient does not recover from the brief asystole.
* D: Side effects of adenosine include flushing, dizziness, and chest discomfort, which should be assessed and documented.
* C is incorrect because adenosine should be administered as a rapid IV push followed by a saline flush.
* E is incorrect because repeated doses should be given only if ordered and with a proper time interval.
A patient with new-onset atrial fibrillation is started on diltiazem IV. Which interprofessional team members should be involved in the patient’s care?
A. Pharmacist, to evaluate drug interactions
B. Respiratory therapist, to assist with oxygen therapy
C. Dietitian, to provide a low-sodium diet
D. Physical therapist, to assess mobility and endurance
A. Pharmacist, to evaluate drug interactions
Rationale: Diltiazem, a calcium channel blocker, can interact with other medications like beta-blockers and digoxin, increasing the risk of bradycardia and hypotension. The pharmacist plays a crucial role in assessing drug interactions. While other team members may be involved in the patient’s care, the pharmacist is most relevant in managing antidysrhythmic therapy.
A patient on long-term sotalol therapy is admitted with a potassium level of 2.8 mEq/L. What is the nurse’s priority intervention?
A. Continue sotalol as prescribed
B. Administer potassium replacement
C. Increase the patient’s fluid intake
D. Place the patient in a high-Fowler’s position
B. Administer potassium replacement
Rationale: Hypokalemia increases the risk of life-threatening dysrhythmias, especially in patients on sotalol, a beta-blocker with class III antidysrhythmic properties. Correcting potassium is crucial to preventing arrhythmias like torsades de pointes.
A nurse is caring for a patient receiving continuous IV amiodarone. The patient’s ECG shows a prolonged QT interval. What should the nurse do first?
A. Stop the infusion and notify the provider
B. Administer atropine to increase the heart rate
C. Decrease the IV amiodarone infusion rate
D. Check the patient’s potassium and magnesium levels
D. Check the patient’s potassium and magnesium levels
Rationale: A prolonged QT interval increases the risk of torsades de pointes. Low potassium or magnesium levels can contribute to QT prolongation, so checking electrolytes is the priority. If abnormalities are present, they should be corrected to prevent life-threatening arrhythmias.
A nurse arrives at the scene where a patient has collapsed in a hospital hallway. The patient is unresponsive, pulseless, and the cardiac monitor shows ventricular fibrillation (VF). What is the nurse’s next immediate action?
A. Start CPR, then retrieve the defibrillator
B. Administer epinephrine before defibrillating
C. Deliver a defibrillation shock as soon as the defibrillator is available
D. Check for a carotid pulse for at least 30 seconds before proceeding
C. Deliver a defibrillation shock as soon as the defibrillator is available
Rationale: Defibrillation is the priority intervention for VF or pulseless VT. It is most effective when performed within 2 minutes of cardiac arrest. CPR should be initiated while the defibrillator is being retrieved, but once it is available, immediate defibrillation takes precedence. Epinephrine is given after the first shock and CPR cycle.
A nurse is teaching a new graduate about defibrillation. Which statements indicate the graduate nurse understands the procedure? (SATA)
A. “Defibrillation is the treatment of choice for pulseless ventricular tachycardia.”
B. “CPR should be paused while the defibrillator is charging.”
C. “The defibrillator should deliver an initial 360-joule shock when using a monophasic defibrillator.”
D. “A biphasic defibrillator requires a higher initial shock energy than a monophasic defibrillator.”
E. “After the first shock, I should immediately resume CPR, starting with chest compressions.”
A. “Defibrillation is the treatment of choice for pulseless ventricular tachycardia.”
B. “CPR should be paused while the defibrillator is charging.”
C. “The defibrillator should deliver an initial 360-joule shock when using a monophasic defibrillator.”
E. “After the first shock, I should immediately resume CPR, starting with chest compressions.”
Rationale:
* A: Defibrillation is the first-line treatment for pulseless VT and VF.
* B: CPR should be paused briefly while the defibrillator is charging to minimize interruptions.
* C: Monophasic defibrillators deliver an initial shock of 360 joules.
* E: After defibrillation, CPR should be resumed immediately to maintain circulation.
* D is incorrect because biphasic defibrillators require lower energy shocks (120-200 joules), leading to fewer post-shock dysrhythmias.
A patient in the ICU suddenly develops pulseless ventricular tachycardia. The nurse prepares for defibrillation. Which action should the nurse take first?
A. Call for help and retrieve the defibrillator
B. Confirm the absence of a pulse and initiate CPR
C. Administer epinephrine per ACLS protocol
D. Set the defibrillator to synchronized mode
B. Confirm the absence of a pulse and initiate CPR
Rationale: Before defibrillating, the nurse must confirm that the patient is truly pulseless. Once pulseless VT or VF is confirmed, CPR should begin immediately while preparing for defibrillation. Defibrillation should never be delayed for medication administration. Synchronized mode is used for cardioversion, not for pulseless rhythms.
A nurse is preparing to use an automated external defibrillator (AED) on an unconscious patient. Which actions should the nurse take? (SATA)
A. Ensure the patient’s chest is dry before placing the AED pads
B. Place the AED pads over medication patches if present
C. Confirm that no one is touching the patient before delivering a shock
D. Use pediatric AED pads on an adult if adult pads are unavailable
E. Allow the AED to analyze the rhythm before delivering a shock
A. Ensure the patient’s chest is dry before placing the AED pads
C. Confirm that no one is touching the patient before delivering a shock
E. Allow the AED to analyze the rhythm before delivering a shock
Rationale:
* A: The chest should be dry to ensure proper conduction of electrical energy.
* C: No one should touch the patient when the shock is delivered to prevent accidental injury.
* E: The AED automatically analyzes the rhythm before instructing the user to deliver a shock.
* B is incorrect because medication patches should be removed before placing AED pads.
* D is incorrect because pediatric pads should not be used on adults; if only pediatric pads are available, manual CPR should be performed until proper equipment is obtained.
A patient experiences sudden cardiac arrest in the emergency department. The nurse initiates CPR, and the defibrillator is brought to the bedside. Which interprofessional team member is responsible for airway management during the resuscitation?
A. The primary nurse
B. The respiratory therapist
C. The pharmacist
D. The case manager
B. The respiratory therapist
Rationale: During cardiac resuscitation, the respiratory therapist is responsible for managing the airway, including providing oxygenation and ventilations. The nurse focuses on defibrillation, medication administration, and CPR coordination.
A patient with ventricular fibrillation is defibrillated with a biphasic defibrillator at 150 joules. The patient remains unresponsive, and the monitor still shows VF. What is the nurse’s next action?
A. Increase the energy to 360 joules and deliver another shock
B. Resume CPR immediately and prepare to administer epinephrine
C. Perform a pulse check to confirm the rhythm is still VF
D. Switch to synchronized cardioversion mode
B. Resume CPR immediately and prepare to administer epinephrine
Rationale: If the first defibrillation attempt is unsuccessful, CPR should be resumed immediately to maintain circulation. Epinephrine is then given per ACLS protocol to increase the likelihood of successful defibrillation. A pulse check should not be performed until after CPR and rhythm assessment. Cardioversion is used for tachyarrhythmias with a pulse.
A patient is being defibrillated for ventricular fibrillation. After two unsuccessful shocks and ongoing CPR, the provider orders amiodarone IV. What is the expected dose for the first
administration?
A. 150 mg IV push
B. 300 mg IV push
C. 1 mg/min IV infusion
D. 5 mg/kg IV bolus
B. 300 mg IV push
Rationale: According to ACLS guidelines, the first dose of amiodarone in cardiac arrest due to VF or pulseless VT is 300 mg IV push. A second dose of 150 mg may be given if needed.
A nurse is assisting with defibrillation in a patient with pulseless VT. Which safety measures should the nurse take? (SATA)
A. Ensure the “all clear” command is given before delivering a shock
B. Place the conductive gel pads directly over the ECG electrodes
C. Verify that the defibrillator is set to “synchronized mode” before shocking
D. Position the defibrillator pads at least 1 inch away from pacemaker sites
E. Confirm that the defibrillator is set to the appropriate energy level
A. Ensure the “all clear” command is given before delivering a shock
D. Position the defibrillator pads at least 1 inch away from pacemaker sites
E. Confirm that the defibrillator is set to the appropriate energy level
Rationale:
* A: The “all clear” command ensures no one is in contact with the patient.
* D: Pads should be placed at least 1 inch from pacemakers to prevent device malfunction.
* E: Proper energy selection is crucial for effective defibrillation.
* B is incorrect because conductive gel pads should not be placed over ECG electrodes.
* C is incorrect because synchronized mode is used for cardioversion, not defibrillation.
A nurse is teaching a patient’s family about AED use. Which statement by a family member indicates further teaching is needed?
A. “If the AED says ‘no shock advised,’ I should continue CPR until help arrives.”
B. “I should place the pads on the chest as shown in the diagram on the AED.”
C. “I should check for a pulse after the AED advises a shock.”
D. “If the patient is in water, I should move them to a dry surface before using the AED.”
C. “I should check for a pulse after the AED advises a shock.”
Rationale: The AED automatically assesses the heart rhythm. Rescuers should not delay CPR to check for a pulse after a shock. Instead, CPR should be resumed immediately as per AED instructions.
A 58-year-old patient presents to the emergency department with palpitations, dizziness, and a heart rate of 180 bpm. The ECG reveals supraventricular tachycardia (SVT) with a rapid ventricular rate. The patient is hemodynamically stable. Which action should the nurse anticipate?
A. Administration of IV adenosine before considering cardioversion
B. Immediate defibrillation at 200 joules
C. Synchronized cardioversion at 360 joules
D. Immediate transcutaneous pacing
A. Administration of IV adenosine before considering cardioversion
Rationale: In a stable patient with SVT, the first-line treatment is vagal maneuvers followed by IV adenosine to terminate the rhythm. If the rhythm persists, synchronized cardioversion may be required. Defibrillation is only used for pulseless rhythms or ventricular fibrillation (VF). Transcutaneous pacing is used for bradycardia, not tachyarrhythmias.
A nurse is preparing a patient for synchronized cardioversion due to atrial flutter with a rapid ventricular response (RVR). Which actions should the nurse take? (SATA)
A. Ensure the synchronizer switch is turned on before delivering the shock
B. Start with an initial energy level of 50 to 100 joules with a biphasic defibrillator
C. Sedate the patient with IV fentanyl if hemodynamically stable
D. Place defibrillator pads over ECG electrodes to ensure accurate conduction
E. Be prepared to switch to defibrillation mode if the patient becomes pulseless
A. Ensure the synchronizer switch is turned on before delivering the shock
B. Start with an initial energy level of 50 to 100 joules with a biphasic defibrillator
C. Sedate the patient with IV fentanyl if hemodynamically stable
E. Be prepared to switch to defibrillation mode if the patient becomes pulseless
Rationale:
* A: The synchronizer switch must be turned on to ensure the shock is delivered on the R wave.
* B: The initial energy for cardioversion in atrial flutter or SVT is 50 to 100 joules (biphasic).
* C: If the patient is stable, sedation is administered before cardioversion.
* E: If the patient deteriorates into a pulseless rhythm (e.g., VF), the synchronizer must be turned off, and defibrillation should be performed.
* D is incorrect because defibrillator pads should not be placed over ECG electrodes.
A patient with atrial fibrillation and a rapid ventricular response is scheduled for synchronized cardioversion. The nurse should anticipate which initial energy setting when using a biphasic defibrillator?
A. 50 joules
B. 100 joules
C. 120-200 joules
D. 360 joules
C. 120-200 joules
Rationale: Atrial fibrillation requires a higher energy level for cardioversion than SVT or atrial flutter. The recommended starting energy for synchronized cardioversion of atrial fibrillation using a biphasic defibrillator is 120-200 joules.
A 72-year-old patient with a history of coronary artery disease and hypertension develops ventricular tachycardia with a pulse. The patient is diaphoretic and has a BP of 80/50 mmHg. Which intervention should the nurse anticipate?
A. Administer IV amiodarone and reassess the patient in 15 minutes
B. Defibrillate the patient at 200 joules
C. Perform vagal maneuvers to attempt rhythm conversion
D. Prepare for synchronized cardioversion at 100 joules
D. Prepare for synchronized cardioversion at 100 joules
Rationale: The patient has VT with a pulse but is hemodynamically unstable (hypotension, diaphoresis). In this case, synchronized cardioversion should be performed as quickly as possible. If the patient were stable, IV antiarrhythmic drugs like amiodarone would be considered first. Defibrillation is used only if the patient becomes pulseless.
A nurse is preparing to assist with synchronized cardioversion for a patient with SVT. Which safety measures should the nurse follow? (SATA)
A. Ensure the synchronizer mode is activated before delivering the shock
B. Confirm that all personnel are clear of the patient before delivering the shock
C. Start at 200 joules for all patients undergoing cardioversion
D. Continuously monitor the patient’s airway and oxygenation
E. If the patient becomes pulseless, turn off the synchronizer and defibrillate
A. Ensure the synchronizer mode is activated before delivering the shock
B. Confirm that all personnel are clear of the patient before delivering the shock
D. Continuously monitor the patient’s airway and oxygenation
E. If the patient becomes pulseless, turn off the synchronizer and defibrillate
Rationale:
* A: The synchronizer switch must be turned on to prevent an R-on-T phenomenon, which can cause VF.
* B: Ensuring that no one is touching the patient prevents accidental electrical injury.
* D: Airway monitoring is critical, especially if sedation is used.
* E: If the patient loses a pulse or the rhythm changes to VF, defibrillation is required.
* C is incorrect because initial energy settings depend on the rhythm (50-100 joules for SVT/VT with a pulse, higher for AFib).
A nurse is assisting with synchronized cardioversion for a patient in SVT. After the shock is delivered, the ECG monitor shows asystole. What is the nurse’s priority action?
A. Deliver another synchronized shock at a higher energy level
B. Reassess the patient’s pulse and blood pressure before intervening
C. Administer adenosine to try and restore a normal rhythm
D. Initiate CPR and prepare to administer epinephrine
D. Initiate CPR and prepare to administer epinephrine
Rationale: Asystole following cardioversion indicates that the heart is not generating electrical activity. The immediate priority is to initiate CPR and administer epinephrine as per ACLS protocols. Additional shocks are not indicated for asystole.
A patient with atrial flutter undergoing elective synchronized cardioversion is receiving IV midazolam for sedation. What is the nurse’s priority assessment during the procedure?
A. Blood glucose levels
B. Peripheral pulses
C. Bowel sounds and GI motility
D. Airway patency and respiratory status
D. Airway patency and respiratory status
Rationale: Sedation with midazolam or fentanyl can cause respiratory depression. Maintaining a patent airway and monitoring respiratory status is the highest priority during cardioversion.
A 65-year-old male with a history of heart failure and an ejection fraction of 25% recently had an implantable cardioverter-defibrillator (ICD) placed. He expresses anxiety about the possibility of receiving a shock. Which statement by the nurse is most appropriate?
A. “The ICD will only shock you if your heart completely stops.”
B. “You will receive a shock if your heart rate drops too low.”
C. “The shock may feel like a sudden jolt, but it is brief and lifesaving.”
D. “If you receive a shock, you should immediately go to the emergency department.”
C. “The shock may feel like a sudden jolt, but it is brief and lifesaving.”
Rationale: Patients with ICDs often fear the sensation of the shock. It is important to reassure them that the shock, while potentially startling, is brief and intended to restore a normal heart rhythm. The ICD does not shock for bradycardia, and patients do not need to seek immediate medical attention unless they experience multiple shocks or symptoms.
A nurse is providing discharge instructions to a patient who recently had an ICD implanted. Which statements should be included in the teaching? (SATA)
A. “Avoid lifting your arm above your shoulder on the side of the ICD placement for at least 6 weeks.”
B. “You should notify your healthcare provider if you receive more than one shock within a short period.”
C. “It is safe to undergo an MRI at any time after the ICD placement.”
D. “Avoid standing near strong electromagnetic fields, such as airport security scanners.”
E. “You should wear a medical alert bracelet indicating you have an ICD.”
A. “Avoid lifting your arm above your shoulder on the side of the ICD placement for at least 6 weeks.”
B. “You should notify your healthcare provider if you receive more than one shock within a short period.”
D. “Avoid standing near strong electromagnetic fields, such as airport security scanners.”
E. “You should wear a medical alert bracelet indicating you have an ICD.”
Rationale:
* A: Patients should avoid excessive arm movement on the ICD side to prevent lead displacement.
* B: Multiple shocks in a short time could indicate an issue that requires medical evaluation.
* D: Strong electromagnetic fields can interfere with ICD function.
* E: A medical alert bracelet helps emergency responders identify the ICD.
* C is incorrect because most ICDs are not MRI-compatible unless specifically designed for MRI use.
A patient with an ICD experiences syncope and is found to be in ventricular fibrillation. The ICD does not successfully restore a normal rhythm after delivering a shock. What is the nurse’s next priority action?
A. Administer IV amiodarone and monitor the rhythm.
B. Call for emergency help and begin CPR immediately.
C. Increase the ICD shock threshold and reassess.
D. Reprogram the ICD to deliver another shock.
B. Call for emergency help and begin CPR immediately.
Rationale: If the ICD fails to restore a normal rhythm, CPR should be initiated immediately while awaiting emergency response. ICDs are programmed and maintained by specialists; nurses cannot adjust shock thresholds or reprogram the device.
Which patients would most likely be candidates for an implantable cardioverter-defibrillator (ICD)? (SATA)
A. A 55-year-old who survived sudden cardiac death due to ventricular fibrillation
B. A 72-year-old with bradycardia requiring permanent pacing
C. A 60-year-old with spontaneous sustained ventricular tachycardia
D. A 45-year-old with hypertrophic cardiomyopathy at high risk for ventricular dysrhythmias
E. A 68-year-old with atrial fibrillation and a rapid ventricular response
A. A 55-year-old who survived sudden cardiac death due to ventricular fibrillation
C. A 60-year-old with spontaneous sustained ventricular tachycardia
D. A 45-year-old with hypertrophic cardiomyopathy at high risk for ventricular dysrhythmias
Rationale:
* A: ICDs are indicated for patients who have survived sudden cardiac death due to VF.
* C: ICDs are recommended for sustained VT to prevent life-threatening arrhythmias.
* D: ICDs are used for high-risk patients with conditions like hypertrophic cardiomyopathy.
* B and E are incorrect because bradycardia is treated with pacemakers, and atrial fibrillation is usually managed with medications, cardioversion, or ablation.
A patient with an ICD reports feeling a single shock but no other symptoms. What is the nurse’s best response?
A. “You should come to the hospital immediately for evaluation.”
B. “Check your pulse, and if it is normal and you feel fine, continue monitoring for additional symptoms.”
C. “You should call 911 because your ICD may have malfunctioned.”
D. “This is normal and does not require any action.”
B. “Check your pulse, and if it is normal and you feel fine, continue monitoring for additional symptoms.”
Rationale: A single shock without ongoing symptoms may indicate the ICD appropriately treated a transient arrhythmia. However, multiple shocks or associated symptoms should prompt medical evaluation.
Which statement by a patient with a newly implanted ICD requires further teaching?
A. “I should avoid lifting heavy objects for several weeks.”
B. “I can use my cell phone on the opposite side of my ICD.”
C. “I need to avoid microwave ovens at home.”
D. “I should notify my dentist about my ICD before any procedure.”
C. “I need to avoid microwave ovens at home.”
Rationale: Modern microwave ovens do not interfere with ICDs. However, patients should still avoid strong electromagnetic fields, such as MRI machines and industrial magnets.
A nurse is educating a patient on what to do if they receive multiple ICD shocks. Which instructions should be included? (SATA)
A. “Lie down and rest after each shock to allow your heart to stabilize.”
B. “If you receive more than one shock in a short period, seek medical attention immediately.”
C. “If you feel unwell after a shock, call 911.”
D. “If you feel fine after a single shock, you do not need to take any further action.”
E. “You should manually reset your ICD after a shock.”
B. “If you receive more than one shock in a short period, seek medical attention immediately.”
C. “If you feel unwell after a shock, call 911.”
Rationale:
* B: Multiple shocks could indicate an ongoing life-threatening arrhythmia requiring immediate medical evaluation.
* C: If a patient feels unwell after a shock, emergency services should be contacted.
* A, D, and E are incorrect because resting alone is not sufficient, and ICDs cannot be manually reset by patients.
A patient with an ICD is scheduled for an elective surgical procedure. What action should the nurse take before the surgery?
A. Ensure the ICD is turned off before surgery.
B. Inform the surgical team that the ICD should not be deactivated.
C. No action is required because ICDs do not interfere with surgery.
D. Advise the patient to have the ICD removed before the procedure.
A. Ensure the ICD is turned off before surgery.
Rationale: Electrocautery used during surgery can interfere with the ICD. The device should be temporarily turned off by a cardiology specialist to prevent inappropriate shocks.
A nurse is caring for a patient who had an ICD implanted 48 hours ago. Which assessment finding requires immediate intervention?
A. Mild discomfort at the incision site
B. The patient reports a sensation of a “fluttering” heartbeat
C. Swelling and redness around the incision site with a temperature of 101°F (38.3°C)
D. The patient expresses concern about body image changes
C. Swelling and redness around the incision site with a temperature of 101°F (38.3°C)
Rationale: Signs of infection, such as redness, swelling, and fever, require immediate intervention to prevent complications, including sepsis or device-related infections. Mild discomfort is expected, and body image concerns should be addressed with support and education.
A patient with an ICD asks about safety precautions when traveling. Which statements should the nurse include in the teaching? (SATA)
A. “Carry a medical alert ID indicating you have an ICD.”
B. “Inform airport security that you have an ICD before passing through metal detectors.”
C. “Avoid placing your cell phone in a pocket near the ICD.”
D. “You should avoid air travel because altitude changes can interfere with your ICD.”
E. “When traveling, bring a copy of your ICD identification card and your healthcare provider’s contact information.”
A. “Carry a medical alert ID indicating you have an ICD.”
B. “Inform airport security that you have an ICD before passing through metal detectors.”
C. “Avoid placing your cell phone in a pocket near the ICD.”
E. “When traveling, bring a copy of your ICD identification card and your healthcare provider’s contact information.”
Rationale:
* A & E: Carrying identification and healthcare provider information ensures appropriate care if an emergency arises.
* B: Airport security scanners contain electromagnetic fields that may interfere with ICDs; patients should notify security personnel.
* C: Cell phones should be kept at least 6 inches away from the ICD to avoid interference.
* D is incorrect because air travel is safe for patients with ICDs.
A patient with an ICD reports feeling anxious about returning to normal activities. What is the best response by the nurse?
A. “You should avoid all physical activity to prevent triggering the ICD.”
B. “It’s best to limit activity and stay indoors as much as possible.”
C. “You may gradually return to normal activities, but avoid contact sports and heavy lifting for now.”
D. “You should only engage in activity if your doctor is present.”
C. “You may gradually return to normal activities, but avoid contact sports and heavy lifting for now.”
Rationale: Patients can resume normal activities but should avoid activities that could displace leads (such as heavy lifting) or cause direct trauma to the ICD (such as contact sports).
A nurse is reviewing a patient’s ICD interrogation report and notes that the ICD delivered three shocks in the past week. What should the nurse do first?
A. Notify the healthcare provider immediately.
B. Reassure the patient that this is normal and requires no action.
C. Suggest the patient deactivate the ICD until further evaluation is completed.
D. Ask the patient if they experienced symptoms such as dizziness, chest pain, or syncope.
D. Ask the patient if they experienced symptoms such as dizziness, chest pain, or syncope.
Rationale: The presence of symptoms can indicate that the shocks were appropriate for life-threatening dysrhythmias. If symptoms are present, urgent evaluation is needed. While notifying the provider is important, gathering more data first helps guide clinical decision-making.
A patient with an ICD is experiencing recurrent inappropriate shocks. What interventions should the healthcare team consider? (SATA)
A. Reassessing ICD settings and lead placement
B. Checking for electrolyte imbalances such as hypokalemia
C. Recommending psychological counseling for anxiety related to the shocks
D. Advising the patient to turn off the ICD temporarily
E. Evaluating for triggers such as strong electromagnetic interference
A. Reassessing ICD settings and lead placement
B. Checking for electrolyte imbalances such as hypokalemia
E. Evaluating for triggers such as strong electromagnetic interference
Rationale:
* A: Improper lead placement or incorrect device programming may cause inappropriate shocks.
* B: Electrolyte imbalances can cause arrhythmias that trigger unnecessary ICD shocks.
* E: Environmental factors, such as electromagnetic interference, may falsely trigger the ICD.
* C is incorrect because while anxiety management is important, it does not address the cause of inappropriate shocks.
* D is incorrect because patients should never turn off their ICD on their own.
A 58-year-old patient with an ICD is admitted to the emergency department after experiencing three shocks in the past hour. The patient is alert but reports dizziness and palpitations. What is the nurse’s priority intervention?
A. Place the patient on continuous ECG monitoring.
B. Instruct the patient to sit in a reclined position and rest.
C. Obtain a blood glucose level and check for hypoglycemia.
D. Administer an anti-anxiety medication to reduce stress.
A. Place the patient on continuous ECG monitoring.
Rationale: Continuous ECG monitoring allows for assessment of the underlying rhythm and determines if the ICD shocks were appropriate. While resting, glucose monitoring, and anxiety management may be beneficial, they are not the priority.
A nurse is teaching a patient with an ICD about post-procedure care. Which statement by the patient indicates a need for further teaching?
A. “I should avoid lifting anything heavier than 10 pounds for the next few weeks.”
B. “I can place my cell phone in my chest pocket next to the ICD for easy access.”
C. “I will notify my healthcare provider if I experience dizziness or feel lightheaded.”
D. “I should avoid prolonged exposure to electromagnetic fields, like arc welding equipment.”
B. “I can place my cell phone in my chest pocket next to the ICD for easy access.”
Rationale: Cell phones should be kept at least 6 inches away from the ICD to prevent electromagnetic interference. The other statements reflect appropriate post-procedure care.
A nurse is assessing a patient with a newly implanted permanent pacemaker. Which finding requires immediate intervention?
A. The patient reports mild discomfort at the incision site.
B. The ECG shows paced beats with appropriate capture.
C. The patient’s heart rate is 40 beats per minute despite the pacemaker being set at 60 beats per minute.
D. The patient reports occasional dizziness when standing up too quickly.
C. The patient’s heart rate is 40 beats per minute despite the pacemaker being set at 60 beats per minute.
Rationale: A heart rate significantly below the pacemaker’s set rate indicates pacemaker malfunction, such as failure to capture or failure to pace. This requires immediate intervention to assess the device and prevent hemodynamic instability. Mild discomfort at the incision site is expected, occasional dizziness may be related to orthostatic hypotension, and paced beats with capture indicate normal pacemaker function.
A nurse is providing discharge instructions to a patient with a new permanent pacemaker. Which statements indicate the patient understands the teaching? (SATA)
A. “I will carry a pacemaker identification card with me at all times.”
B. “I should avoid MRI scans unless my pacemaker is MRI-compatible.”
C. “I can safely use my microwave at home.”
D. “I will avoid lifting my affected arm above my shoulder until cleared by my doctor.”
E. “I should place my cell phone directly over my pacemaker when making calls.”
A. “I will carry a pacemaker identification card with me at all times.”
B. “I should avoid MRI scans unless my pacemaker is MRI-compatible.”
C. “I can safely use my microwave at home.”
D. “I will avoid lifting my affected arm above my shoulder until cleared by my doctor.”
Rationale:
* A & B: Patients should carry identification in case of emergencies and avoid MRI if the pacemaker is not MRI-compatible.
* C: Modern pacemakers are shielded, making microwaves safe to use.
* D: Raising the affected arm above the shoulder too soon can dislodge leads.
* E is incorrect because cell phones should be kept at least 6 inches away from the pacemaker to avoid electromagnetic interference.
A 72-year-old patient with a pacemaker comes to the clinic complaining of intermittent dizziness. The nurse reviews the patient’s ECG and notes that there are no paced beats visible. What is the priority nursing action?
A. Check the pacemaker battery function.
B. Advise the patient to increase fluid intake.
C. Reassure the patient that dizziness is normal with a pacemaker.
D. Suggest the patient lie down when feeling dizzy.
A. Check the pacemaker battery function.
Rationale: Dizziness in a pacemaker-dependent patient with no paced beats suggests pacemaker malfunction, possibly due to battery depletion or lead displacement. Checking battery function is a priority to ensure proper pacing. Increasing fluids or lying down does not address the underlying issue, and dizziness is not a normal or expected symptom of a functioning pacemaker.
A nurse is monitoring a patient with a temporary transvenous pacemaker. The ECG shows pacing spikes without associated QRS complexes. What is the most appropriate initial action?
A. Increase the pacemaker’s milliamperage output.
B. Place the patient in Trendelenburg position.
C. Administer IV atropine.
D. Reposition the patient’s arm above their head.
A. Increase the pacemaker’s milliamperage output.
Rationale: Pacing spikes without QRS complexes indicate failure to capture, meaning the pacemaker’s impulse is not strong enough to depolarize the myocardium. Increasing the milliamperage enhances electrical stimulation to promote capture. Atropine is used for bradycardia but does not address pacemaker failure. Trendelenburg positioning and arm repositioning are not appropriate interventions for pacemaker capture failure.
A nurse is evaluating the function of a patient’s pacemaker. Which findings suggest pacemaker malfunction? (SATA)
A. The patient’s ECG shows pacing spikes but no QRS complexes.
B. The patient reports feeling lightheaded and fatigued.
C. The patient’s heart rate is consistently above the pacemaker’s set rate.
D. The ECG shows random pacing spikes not followed by a heartbeat.
E. The patient reports occasional palpitations but has a normal heart rate.
A. The patient’s ECG shows pacing spikes but no QRS complexes.
B. The patient reports feeling lightheaded and fatigued.
D. The ECG shows random pacing spikes not followed by a heartbeat.
Rationale:
* A & D: Pacing spikes without associated QRS complexes suggest failure to capture, a sign of pacemaker malfunction.
* B: Lightheadedness and fatigue may indicate inadequate cardiac output due to pacemaker failure.
* C is incorrect because the pacemaker only functions when the HR drops below its set threshold.
* E is incorrect because occasional palpitations without abnormal ECG findings do not necessarily indicate pacemaker malfunction.
A patient with a pacemaker is admitted to the hospital after experiencing an electric shock while working with power tools. Which assessment finding would be most concerning?
A. The patient’s heart rate remains at 72 beats per minute.
B. The patient reports mild soreness at the pacemaker site.
C. The ECG shows loss of sensing and erratic pacemaker firing.
D. The patient has no recollection of the event leading to hospitalization.
C. The ECG shows loss of sensing and erratic pacemaker firing.
Rationale: Electromagnetic interference (EMI) from power tools can disrupt pacemaker function, causing inappropriate pacing. Loss of sensing and erratic pacing indicate pacemaker malfunction, which requires immediate evaluation. A stable HR, mild soreness, and temporary memory loss are not as concerning as potential pacemaker dysfunction.
A nurse is caring for a patient who had a permanent pacemaker implanted 24 hours ago. Which action requires immediate intervention?
A. The patient is lying with their affected arm restrained to the bed.
B. The patient’s incision site is clean and dry.
C. The patient states, “I feel like something is pounding in my chest.”
D. The ECG shows a paced rhythm with consistent capture.
C. The patient states, “I feel like something is pounding in my chest.”
Rationale: A sensation of pounding in the chest could indicate pacemaker lead dislodgement or malfunction. This requires immediate evaluation. Restraining the affected arm is inappropriate, as gentle movement is encouraged to prevent stiffness. A clean incision site and consistent pacemaker function are expected findings.
A patient with a permanent pacemaker asks if they can participate in regular physical activities. What is the nurse’s best response?
A. “You should avoid all physical activity to prevent damage to the pacemaker.”
B. “You should limit activity to walking short distances only.”
C. “Strenuous activity is completely safe as long as you feel well.”
D. “You can resume most normal activities, but avoid contact sports and heavy lifting until cleared by your doctor.”
D. “You can resume most normal activities, but avoid contact sports and heavy lifting until cleared by your doctor.”
Rationale: Patients with pacemakers can return to regular activities, but contact sports and heavy lifting can displace leads or damage the device. Avoiding all activity is unnecessary, and unrestricted strenuous activity without clearance is unsafe.
A nurse is caring for a patient who just had a permanent pacemaker implanted. Which action should the nurse take to prevent pacemaker lead dislodgment?
A. Encourage the patient to perform range-of-motion exercises with the affected arm.
B. Teach the patient to avoid lifting the affected arm above the shoulder.
C. Instruct the patient to sleep on the side of the pacemaker.
D. Allow the patient to resume regular activities immediately.
B. Teach the patient to avoid lifting the affected arm above the shoulder.
Rationale: Lifting the affected arm above the shoulder too soon after pacemaker implantation can dislodge the pacing leads. The patient should avoid excessive movement of the arm for several weeks. Range-of-motion exercises should be limited initially, and sleeping on the pacemaker side may cause discomfort and increase the risk of lead dislodgment. Immediate resumption of normal activities is not recommended.
A nurse is providing discharge teaching to a patient with a newly implanted permanent pacemaker. Which statements by the patient indicate a need for further teaching? (SATA)
A. “I will avoid lifting heavy objects for several weeks.”
B. “I should not raise my arm above my shoulder until my doctor clears me.”
C. “I can have an MRI whenever needed.”
D. “I should avoid standing near strong magnets, such as in airport security.”
E. “I can use my cell phone, but I should keep it at least 6 inches from my pacemaker.”
C. “I can have an MRI whenever needed.”
D. “I should avoid standing near strong magnets, such as in airport security.”
Rationale:
* C: Not all pacemakers are MRI-compatible. The patient must check with their provider before undergoing an MRI.
* D: Strong electromagnetic fields, such as those in airport security, can interfere with pacemaker function. The patient should inform security personnel about their device.
* A, B, and E are correct statements and demonstrate proper understanding of post-pacemaker care.
A patient with a permanent pacemaker suddenly develops dizziness and syncope. The ECG shows a heart rate of 30 beats per minute with no pacemaker spikes visible. What is the nurse’s priority action?
A. Increase the pacemaker’s sensitivity setting.
B. Place the patient in Trendelenburg position.
C. Prepare to initiate transcutaneous pacing.
D. Administer IV atropine and observe for improvement.
C. Prepare to initiate transcutaneous pacing.
Rationale: A heart rate of 30 bpm without pacemaker spikes suggests pacemaker failure (failure to pace). Transcutaneous pacing is the priority intervention to maintain cardiac output while awaiting further evaluation. Atropine may be considered but is not the first-line treatment for pacemaker failure. Trendelenburg positioning does not address the underlying issue, and increasing the pacemaker’s sensitivity setting is not within the nurse’s scope of practice.
A patient with a leadless permanent pacemaker asks about its benefits compared to a traditional pacemaker. Which statement by the nurse is accurate?
A. “A leadless pacemaker eliminates the risk of lead-related complications.”
B. “A leadless pacemaker has a larger battery that lasts longer than traditional pacemakers.”
C. “A leadless pacemaker requires surgical removal when the battery depletes.”
D. “A leadless pacemaker is only suitable for patients who need dual-chamber pacing.”
A. “A leadless pacemaker eliminates the risk of lead-related complications.”
Rationale: Leadless pacemakers do not require transvenous leads, reducing complications such as lead dislodgment, fracture, or infection. They are typically smaller but have a similar battery lifespan to traditional pacemakers. Most leadless pacemakers remain in the heart after the battery depletes, with a new device implanted as needed. Leadless pacemakers are primarily used for single-chamber pacing, not dual-chamber pacing.
The nurse is educating a patient with a permanent pacemaker about signs of pacemaker malfunction. Which symptoms should the patient report immediately? (SATA)
A. Sudden dizziness or fainting.
B. Persistent hiccups.
C. Heart rate consistently below the programmed rate.
D. Mild discomfort at the pacemaker site.
E. Frequent palpitations.
A. Sudden dizziness or fainting.
B. Persistent hiccups.
C. Heart rate consistently below the programmed rate.
E. Frequent palpitations.
Rationale:
* A, B, C, and E: Dizziness, fainting, persistent hiccups (which may indicate diaphragmatic pacing), a low heart rate, and frequent palpitations could indicate pacemaker malfunction and require prompt evaluation.
* D is incorrect because mild discomfort is expected postoperatively, but worsening pain or signs of infection should be reported.
A patient with a permanent pacemaker is scheduled for surgery. What action should the nurse take to ensure pacemaker function during the procedure?
A. Place the patient in a lead-lined operating room to protect against electromagnetic interference.
B. Advise the patient to turn off the pacemaker using their remote device before the procedure.
C. Reprogram the pacemaker to deliver a higher pacing rate during the procedure.
D. Notify the surgical team so they can place a magnet over the pacemaker to switch it to asynchronous mode if needed.
D. Notify the surgical team so they can place a magnet over the pacemaker to switch it to asynchronous mode if needed.
Rationale: During surgery, electrocautery can interfere with pacemaker function. Placing a magnet over the device switches it to asynchronous mode, ensuring consistent pacing. Lead-lined rooms are unnecessary, and patients should never turn off their own pacemaker. Adjusting the pacing rate is a decision made by the cardiologist, not the nurse.
A patient with heart failure has an intraventricular conduction delay leading to dyssynchrony between the right and left ventricles. The cardiologist recommends cardiac resynchronization therapy (CRT).
Which primary benefit should the nurse explain to the patient?
A. “CRT will eliminate the need for future heart failure medications.”
B. “CRT will permanently cure your heart failure by strengthening the heart muscle.”
C. “CRT will prevent all future episodes of ventricular tachycardia and fibrillation.”
D. “CRT will improve ventricular contraction and overall heart function.”
D. “CRT will improve ventricular contraction and overall heart function.”
Rationale: CRT improves ventricular synchrony, enhancing systolic function and cardiac output. It does not eliminate the need for medications, cure heart failure, or prevent all dysrhythmias, though it may reduce their occurrence when combined with an ICD.
A nurse is caring for a patient who recently received a cardiac resynchronization therapy (CRT) device. Which expected improvements should the nurse monitor for in this patient? (SATA)
A. Increased exercise tolerance
B. Decreased ejection fraction
C. Reduced symptoms of heart failure, such as dyspnea
D. Improved cardiac output
E. Increased ventricular dyssynchrony
A. Increased exercise tolerance
C. Reduced symptoms of heart failure, such as dyspnea
D. Improved cardiac output
Rationale:
* A, C, and D: CRT improves cardiac function, leading to increased exercise tolerance, reduced heart failure symptoms, and enhanced cardiac output.
* B is incorrect because CRT aims to improve, not decrease, ejection fraction.
* E is incorrect because CRT reduces, not increases, ventricular dyssynchrony.
A patient with a CRT device combined with an implantable cardioverter-defibrillator (ICD) experiences multiple shocks from the device. What is the nurse’s priority intervention?
A. Reassure the patient that multiple shocks are normal and will stop on their own.
B. Place a magnet over the device to deactivate the shocks permanently.
C. Assess the patient for signs of worsening heart failure or dysrhythmias.
D. Advise the patient to avoid movement until the shocks stop.
C. Assess the patient for signs of worsening heart failure or dysrhythmias.
Rationale: Multiple ICD shocks indicate recurrent dysrhythmias or worsening heart failure, requiring immediate assessment. The nurse should evaluate the patient’s condition and notify the provider. Magnets are used to deactivate ICD shocks temporarily in emergency situations but should not be applied without medical direction.
A patient with severe left ventricular dysfunction is scheduled for CRT device placement. The nurse understands that which additional device is commonly combined with CRT in these patients?
A. External pacemaker
B. Automated external defibrillator (AED)
C. Implantable cardioverter-defibrillator (ICD)
D. Left ventricular assist device (LVAD)
C. Implantable cardioverter-defibrillator (ICD)
Rationale: Patients with severe left ventricular dysfunction are at high risk for life-threatening ventricular arrhythmias, so CRT is often combined with an ICD to provide both resynchronization and defibrillation if needed. An external pacemaker is not a long-term solution, an AED is for external use, and an LVAD is typically reserved for end-stage heart failure or transplant candidates.
A patient in the ICU requires temporary pacing due to symptomatic bradycardia unresponsive to atropine. The physician orders the placement of a transvenous pacemaker. What is the primary nursing intervention before the procedure?
A. Administer pain medication and sedation as prescribed.
B. Educate the patient that the pacemaker will remain permanent.
C. Place defibrillator pads on the patient’s chest in case of ventricular fibrillation.
D. Explain that the pacemaker will not require any external power source.
A. Administer pain medication and sedation as prescribed.
Rationale: Transvenous pacemaker insertion can be uncomfortable, so sedation and analgesia should be provided when appropriate. The device is temporary, not permanent (B is incorrect). Defibrillator pads are not routinely required for pacemaker placement (C is incorrect). A transvenous pacemaker requires an external power source (D is incorrect).
A nurse is caring for a patient with an epicardial pacemaker following open-heart surgery. Which nursing considerations are appropriate for this patient? (SATA)
A. Closely monitor for signs of infection at the insertion site.
B. Secure and insulate the pacemaker wires to prevent accidental dislodgment.
C. Perform CPR immediately if pacemaker wires become disconnected.
D. Avoid manipulating or pulling on the pacing leads.
E. Teach the patient that the pacemaker will remain in place permanently.
A. Closely monitor for signs of infection at the insertion site.
B. Secure and insulate the pacemaker wires to prevent accidental dislodgment.
D. Avoid manipulating or pulling on the pacing leads.
Rationale:
* A, B, and D: Infection risk, wire security, and avoiding lead displacement are critical for epicardial pacemaker care.
* C is incorrect: If wires are dislodged, CPR is not the immediate action unless cardiac arrest occurs.
* E is incorrect: Epicardial pacemakers are temporary, typically used postoperatively.
A patient with a transcutaneous pacemaker (TCP) expresses concern about discomfort from the pacing. What is the best response by the nurse?
A. “Unfortunately, there is no way to reduce the discomfort from the pacemaker.”
B. “The discomfort is temporary, and we will provide pain relief and sedation as needed.”
C. “Once the pacemaker is placed, you should not feel any discomfort at all.”
D. “We will discontinue the pacemaker immediately if you feel pain.”
B. “The discomfort is temporary, and we will provide pain relief and sedation as needed.”
Rationale: Transcutaneous pacing often causes muscle contractions, which can be uncomfortable. The nurse should reassure the patient while also providing pain management. Pain is expected, so stating that there is no way to reduce discomfort (A) or that the patient will not feel any pain (C) is inaccurate. Discontinuing pacing immediately is not an option if the patient requires it for life-saving purposes (D).
A patient with a transcutaneous pacemaker is experiencing extreme discomfort due to the electrical impulses. What is the nurse’s priority action?
A. Increase the pacing current to ensure proper ventricular capture.
B. Administer prescribed analgesics or sedatives to improve comfort.
C. Turn off the pacemaker and monitor the patient’s intrinsic rhythm.
D. Remove the electrode pads and reposition them to a less painful area.
B. Administer prescribed analgesics or sedatives to improve comfort.
Rationale: Transcutaneous pacing is often painful, so pain management is crucial. Increasing the pacing current (A) would worsen discomfort. Turning off the pacemaker (C) may compromise the patient’s cardiac output. Repositioning pads (D) may not help and could interfere with pacing effectiveness.
Which statement best explains why a transcutaneous pacemaker (TCP) is used as an emergency intervention?
A. “It provides immediate electrical stimulation to the heart until more definitive therapy is available.”
B. “It permanently corrects bradyarrhythmias by stimulating myocardial contraction.”
C. “It is the most comfortable and effective method of cardiac pacing for long-term use.”
D. “It reduces the risk of myocardial infarction by decreasing heart rate variability.”
A. “It provides immediate electrical stimulation to the heart until more definitive therapy is available.”
Rationale: A TCP is a temporary, emergency measure to sustain heart function until a transvenous or permanent pacemaker is placed. It does not permanently correct arrhythmias (B), is not ideal for long-term use due to discomfort (C), and does not directly reduce MI risk (D).
Which statements should the nurse include when educating a patient about a temporary transvenous pacemaker? (SATA)
A. “This pacemaker requires an external power source to function.”
B. “The leads of the pacemaker are inserted through a vein and into the heart.”
C. “This type of pacemaker is permanent and does not require further intervention.”
D. “You may experience mild discomfort at the insertion site.”
E. “Your heart will become completely dependent on this pacemaker for normal function.”
A. “This pacemaker requires an external power source to function.”
B. “The leads of the pacemaker are inserted through a vein and into the heart.”
D. “You may experience mild discomfort at the insertion site.”
Rationale:
* A and B: A transvenous pacemaker has leads inserted into the heart and relies on an external power source.
* D: Some discomfort at the insertion site is expected.
* C is incorrect: This pacemaker is temporary.
* E is incorrect: It may support the heart but does not necessarily mean complete dependence.
A patient with a transvenous pacemaker suddenly develops loss of capture (no paced QRS complexes). What is the nurse’s immediate action?
A. Increase the pacing output (mA) to improve capture.
B. Notify the provider and wait for further instructions.
C. Turn off the pacemaker and observe the patient’s intrinsic rhythm.
D. Place the patient in the Trendelenburg position.
A. Increase the pacing output (mA) to improve capture.
Rationale: Increasing the pacing output can improve capture. Waiting for instructions (B) could delay necessary intervention. Turning off the pacemaker (C) could lead to hemodynamic instability. Trendelenburg position (D) is not indicated.
A patient with an epicardial pacemaker post-cardiac surgery suddenly develops electrical interference and pacemaker malfunction. What is the nurse’s priority intervention?
A. Remove the pacemaker leads immediately to prevent further issues.
B. Disconnect the pacemaker from the external power source and observe.
C. Insulate the pacemaker wires and notify the healthcare provider.
D. Increase the pacing rate to override the interference.
C. Insulate the pacemaker wires and notify the healthcare provider.
Rationale: Epicardial wires must be secured and insulated to prevent electrical disturbances. Removing leads (A) is not within the nurse’s scope. Disconnecting the pacemaker (B) may cause bradycardia or asystole. Increasing the pacing rate (D) does not address the underlying problem.
A patient with a newly implanted permanent pacemaker is admitted for routine monitoring. While reviewing the ECG, the nurse observes pacemaker spikes that do not generate QRS complexes. What is the most likely cause of this finding?
A. Failure to pace
B. Failure to sense
C. Oversensing
D. Failure to capture
D. Failure to capture
Rationale: Failure to capture occurs when the pacemaker generates an electrical stimulus, but the myocardium does not respond with a contraction, leading to pacemaker spikes without QRS complexes. Common causes include battery failure, dislodged leads, or insufficient electrical charge.
Which findings indicate failure to sense in a patient with a pacemaker? (SATA)
A. Pacemaker spikes occurring randomly throughout the ECG strip
B. Pacemaker firing during the refractory period, increasing the risk of ventricular tachycardia
C. Pacemaker spikes preceding each QRS complex consistently
D. Pacemaker activity not adjusting in response to the patient’s intrinsic heart rhythm
E. ECG showing continuous asystole without pacemaker activity
A. Pacemaker spikes occurring randomly throughout the ECG strip
B. Pacemaker firing during the refractory period, increasing the risk of ventricular tachycardia
D. Pacemaker activity not adjusting in response to the patient’s intrinsic heart rhythm
Rationale:
* A & B: Failure to sense occurs when the pacemaker does not detect the heart’s intrinsic activity and fires inappropriately.
* D: The pacemaker should adjust based on intrinsic activity, and failure to sense prevents this.
* C is incorrect: Consistent pacing before QRS complexes suggests appropriate function.
* E is incorrect: Continuous asystole suggests failure to pace, not failure to sense.
A patient with a temporary transvenous pacemaker suddenly develops failure to pace. What is the nurse’s priority action?
A. Check the pacemaker battery and replace it if necessary
B. Increase the pacing voltage to improve conduction
C. Place the patient in a high Fowler’s position
D. Immediately prepare for pacemaker removal
A. Check the pacemaker battery and replace it if necessary
Rationale: Failure to pace occurs when the pacemaker does not deliver an electrical impulse. Common causes include battery failure, wire fractures, or lead displacement. The priority is to assess and replace the battery if needed before considering other interventions.
A patient with a permanent pacemaker is being discharged. Which statement by the patient indicates a need for further teaching?
A. “I will avoid lifting my left arm above my shoulder for several weeks.”
B. “I should check my pulse daily and report any unusual changes.”
C. “I should avoid MRI scans unless my doctor says my pacemaker is MRI-compatible.”
D. “I can use my microwave, but I should avoid standing too close while it’s on.”
D. “I can use my microwave, but I should avoid standing too close while it’s on.”
Rationale: Modern pacemakers are shielded against microwave interference, so there is no need to avoid standing near a microwave. The other statements are correct regarding activity restrictions, pulse monitoring, and MRI safety.
Which nursing interventions help prevent complications after permanent pacemaker insertion? (SATA)
A. Monitor the patient’s ECG rhythm for pacemaker malfunction
B. Encourage early ambulation without movement restrictions
C. Assess the insertion site for signs of infection or hematoma
D. Educate the patient to avoid vigorous arm and shoulder activity on the operative side
E. Delay antibiotic administration until signs of infection appear
A. Monitor the patient’s ECG rhythm for pacemaker malfunction
C. Assess the insertion site for signs of infection or hematoma
D. Educate the patient to avoid vigorous arm and shoulder activity on the operative side
Rationale:
* A: ECG monitoring helps detect pacemaker malfunctions.
* C & D: Monitoring for infection and limiting shoulder movement prevents complications.
* B is incorrect: Arm and shoulder activity should be limited initially to avoid lead dislodgement.
* E is incorrect: Prophylactic antibiotics are given before and after pacemaker insertion to prevent infection.
A patient with a newly inserted permanent pacemaker reports increasing pain and swelling at the insertion site. The nurse’s priority action is to:
A. Apply a warm compress to reduce swelling
B. Administer prescribed analgesics and document the pain level
C. Notify the provider immediately and assess for signs of infection
D. Encourage the patient to perform gentle range-of-motion exercises
C. Notify the provider immediately and assess for signs of infection
Rationale: Increasing pain and swelling at the insertion site may indicate infection or hematoma formation. Immediate assessment and possible intervention are needed to prevent complications.
Which complication is most closely associated with lead displacement after pacemaker insertion?
A. Pneumothorax
B. Failure to capture
C. Hypertension
D. Atrial fibrillation
B. Failure to capture
Rationale: Lead displacement can prevent the pacemaker from delivering an effective electrical stimulus, resulting in failure to capture. Pneumothorax (A) is a potential complication but is not directly caused by lead displacement.
A patient with a pacemaker shows intermittent loss of capture on ECG. What is the first nursing intervention?
A. Reposition the patient to improve lead contact
B. Increase the pacing threshold
C. Replace the pacemaker battery
D. Apply external pacing immediately
A. Reposition the patient to improve lead contact
Rationale: A change in position may help restore contact between the pacemaker lead and the myocardium. Increasing the threshold (B) or replacing the battery (C) should be done if repositioning fails. External pacing (D) is a last resort.
A patient with a new pacemaker asks when they can resume normal activities. The nurse should respond:
A. “You can resume all normal activities immediately.”
B. “You should avoid lifting your arm above your shoulder for at least 4 to 6 weeks.”
C. “You should avoid walking for long distances for the rest of your life.”
D. “You can drive as soon as you feel ready, regardless of your doctor’s advice.”
B. “You should avoid lifting your arm above your shoulder for at least 4 to 6 weeks.”
Rationale: Lifting the arm above the shoulder too soon increases the risk of lead displacement.
After a permanent pacemaker insertion, the nurse should immediately report which finding?
A. Small amount of bruising at the insertion site
B. Mild discomfort when moving the affected arm
C. Persistent hiccups
D. Heart rate of 68 bpm with normal pacemaker function
C. Persistent hiccups
Rationale: Persistent hiccups may indicate lead displacement stimulating the diaphragm. This requires immediate intervention.
Which statement by a patient with a pacemaker requires further teaching?
A. “I will carry a pacemaker identification card at all times.”
B. “I should avoid standing in antitheft devices at store entrances.”
C. “I will need regular follow-up visits to check pacemaker function.”
D. “I should avoid using cell phones and Wi-Fi devices.”
B. “I should avoid standing in antitheft devices at store entrances.”
Rationale: Cell phones and Wi-Fi do not interfere with modern pacemakers, but patients should avoid placing a cell phone directly over the pacemaker.
A patient with a pacemaker asks, “How will I know when my pacemaker battery is low?” The nurse should respond:
A. “Your pacemaker will stop working suddenly, so you need emergency backup pacing.”
B. “You will experience severe chest pain when the battery runs low.”
C. “There are no warning signs before the pacemaker battery stops.”
D. “Routine follow-up visits will detect when the battery is near the end of its life.”
D. “Routine follow-up visits will detect when the battery is near the end of its life.”
Rationale: Pacemaker batteries are monitored regularly, and elective replacement is scheduled before failure occurs.
A patient with atrial fibrillation is scheduled for radiofrequency catheter ablation therapy. The nurse explains that the purpose of this procedure is to:
A. Implant a pacemaker to regulate the heart rate
B. Burn or ablate areas of the heart’s conduction system causing dysrhythmias
C. Replace the sinoatrial (SA) node with an artificial pacemaker
D. Administer a controlled electrical shock to restore normal sinus rhythm
B. Burn or ablate areas of the heart’s conduction system causing dysrhythmias
Rationale: Radiofrequency catheter ablation therapy uses electrical energy to destroy abnormal conduction pathways responsible for tachydysrhythmias. It is a definitive treatment for certain atrial dysrhythmias, including atrial fibrillation and AV nodal reentrant tachycardia.
Which pre-procedure nursing interventions are appropriate for a patient undergoing radiofrequency catheter ablation therapy? (SATA)
A. Obtain informed consent before the procedure
B. Educate the patient that they will be under general anesthesia
C. Hold anticoagulant medications as prescribed before the procedure
D. Instruct the patient that they may feel a burning sensation during the ablation
E. Monitor the patient for signs of bleeding or hematoma formation at the catheter insertion site
A. Obtain informed consent before the procedure
C. Hold anticoagulant medications as prescribed before the procedure
D. Instruct the patient that they may feel a burning sensation during the ablation
E. Monitor the patient for signs of bleeding or hematoma formation at the catheter insertion site
Rationale:
* A: Informed consent is required for invasive procedures.
* C: Anticoagulants are often held before ablation to reduce bleeding risk.
* D: Patients may feel a burning sensation during the ablation as heat energy is applied.
* E: Bleeding or hematoma formation is a potential complication at the catheter insertion site.
A patient with syncope has continuous ECG monitoring. The rhythm strip shows: Atrial rate 74 beats/min and regular; ventricular rate 62 beats/min and irregular; P wave normal shape; PR interval lengthens progressively until a P wave is not conducted; QRS normal shape. Which intervention would the nurse prioritize?
a. Administer epinephrine 1 mg IV push.
b. Prepare the patient for synchronized cardioversion.
c. Observe for symptoms of hypotension and angina.
d. Apply transcutaneous pacemaker pads on the patient.
c. Observe for symptoms of hypotension and angina.
The ECG monitor of a patient in the cardiac care unit after an MI shows ventricular bigeminy with a rate of 50 beats/min. Which action would the nurse take?
a. Perform defibrillation.
b. Administer IV amiodarone.
c. Prepare for pacemaker insertion.
d. Assess the patient’s response.
b. Administer IV amiodarone.
In the patient with supraventricular tachycardia, which assessment indicates decreased cardiac output?
a. Hypertension and dyspnea
b. Chest pain and palpitations
c. Abdominal distention and tachypnea
d. Bounding pulses and a systolic murmur
b. Chest pain and palpitations
The nurse prepares a patient for elective synchronized cardioversion. Which information would the nurse consider in planning for the procedure?
a. Defibrillation delivers a lower dose of electrical energy.
b. Cardioversion is a treatment for atrial bradydysrhythmias.
c. Defibrillation delivers a shock during the QRS complex.
d. Cardioversion is painful for an awake patient.
d. Cardioversion is painful for an awake patient.
Which patient teaching points would the nurse include when providing discharge instructions to a patient with a new permanent pacemaker? (select all that apply)
a. Avoid or limit air travel.
b. Take and record a daily pulse rate.
c. Obtain and wear a Medic Alert ID device at all times.
d. Avoid lifting arm on the side of the pacemaker above shoulder.
b. Take and record a daily pulse rate.
c. Obtain and wear a Medic Alert ID device at all times.
d. Avoid lifting arm on the side of the pacemaker above shoulder.
Which information would the nurse teach the patient scheduled for a radiofrequency catheter ablation procedure?
a. Ventricular bradycardia may be induced and treated during the procedure.
b. A catheter will be placed in both femoral arteries to allow double-catheter intervention.
c. The procedure will destroy areas of the conduction system that are causing rapid heart rhythms.
d. General anesthetic will be given to prevent the awareness of any “sudden cardiac death” experiences.
c. The procedure will destroy areas of the conduction system that are causing rapid heart rhythms.
What would the nurse measure to determine whether there is a delay in electrical impulse conduction through the patient‘s ventricles?
a. P wave
b. Q wave
c. PR interval
d. QRS complex
d. QRS complex
Rationale: The QRS complex represents ventricular depolarization. The P wave represents the depolarization of the atria. The PR interval represents depolarization of the atria, atrioventricular node, bundle of His, bundle branches, and the Purkinje fibers. The Q wave is the first negative deflection following the P wave and would be narrow and short.
The nurse needs to measure the heart rate for a patient with an irregular heart rhythm. Which method will be accurate?
a. Count the number of large squares in the R-R interval and divide by 300.
b. Print a 1-minute electrocardiogram (ECG) strip and count the number of QRS complexes.
c. Use the 3-second markers to count the number of QRS complexes in 6 seconds and multiply by 10.
d. Calculate the number of small squares between one QRS complex and the next and divide into 1500.
b. Print a 1-minute electrocardiogram (ECG) strip and count the number of QRS complexes.
Rationale: The accurate way to measure the heart rate from an EKG of a patient with an irregular rhythm is to count the number of QRS complexes in 1 minute. The other methods are accurate for regular heart rhythms.
A patient has a junctional escape rhythm on the monitor. Which range of heart rate would the nurse expect?
a. 15 to 20
b. 20 to 40
c. 40 to 60
d. 60 to 100
c. 40 to 60
Rationale: If the sinoatrial (SA) node does not discharge, the atrioventricular (AV) node will automatically discharge at the normal rate of 40 to 60 beats/min. The slower rates are typical of the bundle of His and Purkinje system and may be seen with failure of both the SA and AV node to discharge. The normal SA node rate is 60 to 100 beats/min.
The nurse obtains a rhythm strip on a patient who has had a myocardial infarction and makes the following analysis: no visible P waves, PR interval not measurable, ventricular rate of 162, R-R interval regular, QRS complex wide and distorted, and QRS duration of 0.18 second. How would the nurse interpret this cardiac rhythm?
a. Atrial flutter
b. Sinus tachycardia
c. Ventricular fibrillation
d. Ventricular tachycardia
d. Ventricular tachycardia
Rationale: The absence of P waves, wide QRS, rate greater than 150 beats/min, and the regularity of the rhythm indicate ventricular tachycardia. Atrial flutter is usually regular, has a narrow QRS configuration, and has flutter waves present representing atrial activity. Sinus tachycardia has P waves. Ventricular fibrillation is irregular and does not have a consistent QRS duration.
A patient‘s heart monitor shows that every other beat is earlier than expected, has no visible P wave, and has a QRS complex that is wide and bizarre in shape. How will the nurse document the rhythm?
a. Ventricular couplets
b. Ventricular bigeminy
c. Ventricular R-on-T phenomenon
d. Multifocal premature ventricular contractions
b. Ventricular bigeminy
Rationale: Ventricular bigeminy describes a rhythm in which every other QRS complex is wide and bizarre looking. Pairs of wide QRS complexes are described as ventricular couplets. There is no indication that the premature ventricular contractions are multifocal or that the R-on-T phenomenon is occurring.
A patient has a sinus rhythm and a heart rate of 72 beats/min. The nurse determines that the PR interval is 0.24 seconds. Which action would the nurse take?
a. Notify the health care provider immediately.
b. Document the finding and monitor the patient.
c. Give atropine per agency dysrhythmia protocol.
d. Prepare the patient for temporary pacemaker insertion.
b. Document the finding and monitor the patient.
Rationale: First-degree atrioventricular block is asymptomatic and usually not serious. There is no treatment for first-degree AV block; treatment of associated conditions may be considered. Monitor patients for changes in heart rhythm (e.g., more serious AV block). The rate is normal, so there is no indication that atropine is needed. Immediate notification of the health care provider about an asymptomatic rhythm is not necessary.
A patient who was admitted with a myocardial infarction has a 45-second episode of ventricular tachycardia, then converts to sinus rhythm with a heart rate of 98 beats/min. Which action would the nurse take next?
a. Immediately notify the health care provider.
b. Document the rhythm and continue to monitor the patient.
c. Prepare for synchronized cardioversion per agency protocol.
d. Prepare to give IV amiodarone per agency dysrhythmia protocol.
d. Prepare to give IV amiodarone per agency dysrhythmia protocol.
Rationale: The burst of sustained ventricular tachycardia indicates that the patient has significant ventricular irritability, and antidysrhythmic medication administration is needed to prevent further episodes. The nurse should notify the health care provider after the medication is started. Cardioversion is not indicated given that the patient has returned to a sinus rhythm. Documentation and continued monitoring are not adequate responses to this situation.
After the nurse gives IV atropine to a patient with symptomatic type 1, second-degree atrioventricular (AV) block, which finding indicates that the drug has been effective?
a. Increase in the patient‘s heart rate
b. Increase in strength of peripheral pulses
c. Decrease in premature atrial contractions
d. Decrease in premature ventricular contractions
a. Increase in the patient‘s heart rate
Rationale: Atropine will increase the heart rate and conduction through the AV node. Because the drug increases electrical conduction, not cardiac contractility, the quality of the peripheral pulses is not used to evaluate the drug effectiveness. Premature atrial or ventricular contractions are not a feature of second degree AV block.
A patient with dilated cardiomyopathy has new onset atrial fibrillation that has been unresponsive to drug therapy for four days. Which topic would the nurse plan to include in patient teaching?
a. Anticoagulant therapy
b. Permanent pacemakers
c. Emergency cardioversion
d. IV adenosine (Adenocard)
a. Anticoagulant therapy
Rationale: Atrial fibrillation therapy that has persisted for more than 48 hours requires anticoagulant treatment for 3 weeks before attempting cardioversion. This is done to prevent embolization of clots from the atria. Cardioversion may be done after several weeks of anticoagulation therapy. Adenosine is not used to treat atrial fibrillation. Pacemakers are routinely used for patients with bradydysrhythmias. Information does not indicate that the patient has a slow heart rate.
Which information will the nurse include when teaching a patient with atrial flutter who is scheduled for a radiofrequency catheter ablation?
a. The procedure stimulates the growth of new pathways between the atria.
b. The procedure uses cold therapy to stop the formation of the flutter waves.
c. The procedure uses electrical energy to destroy areas of the conduction system.
d. The procedure prevents or minimizes the patient‘s risk for sudden cardiac death.
c. The procedure uses electrical energy to destroy areas of the conduction system.
Rationale: Radiofrequency catheter ablation therapy uses electrical energy to “burn” or ablate areas of the conduction system as definitive treatment of atrial flutter (i.e., restore normal sinus rhythm) and tachydysrhythmias. All other statements about the procedure are incorrect.
Which patient statement indicates that discharge teaching about the management of a new permanent pacemaker was effective?
a. “It will be several weeks before I can return to my usual activities.”
b. “I will avoid cooking with a microwave oven or being near one in use.”
c. “I will notify the airlines when I make a reservation that I have a pacemaker.”
d. “I won‘t lift the arm on the incision side until I see the health care provider.”
d. “I won‘t lift the arm on the incision side until I see the health care provider.”
Rationale: The patient is instructed to avoid lifting the arm on the pacemaker side above the shoulder to avoid displacing the pacemaker leads. The patient should notify airport security about the presence of a pacemaker before going through the metal detector, but there is no need to notify the airlines when making a reservation. Microwave oven use does not affect the pacemaker. The insertion procedure involves minor surgery that will have a short recovery period.
Which action by a nurse caring for a patient after an implantable cardioverter-defibrillator (ICD) insertion indicates a need for more teaching about the care of patients with ICDs?
a. The nurse administers amiodarone (Cordarone) to the patient.
b. The nurse helps the patient fill out the application for obtaining a Medic Alert device.
c. The nurse encourages the patient to do active range-of-motion exercises for all extremities.
d. The nurse teaches the patient that sexual activity can be resumed when the incision is healed.
c. The nurse encourages the patient to do active range-of-motion exercises for all extremities.
Rationale: The patient should avoid moving the arm on the ICD insertion site until healing has occurred to prevent displacement of the ICD leads. The other actions by the nurse are appropriate for this patient.
A patient with supraventricular tachycardia who is alert and has a blood pressure of 110/66 mm Hg is being prepared for cardioversion. Which action should the nurse expect to take?
a. Turn the synchronizer switch to the “off” position.
b. Give a sedative before cardioversion is implemented.
c. Set the defibrillator/cardioverter energy to 360 joules.
d. Provide assisted ventilations with a bag-valve-mask device.
b. Give a sedative before cardioversion is implemented.
Rationale: When a patient has a nonemergency cardioversion, sedation is used just before the procedure. The synchronizer switch is turned “on” for cardioversion. The initial level of joules for cardioversion is low (e.g., 50). Assisted ventilations are not indicated for this alert patient.
A 20-yr-old patient has a mandatory electrocardiogram (ECG) before participating on a college soccer team. The patient is found to have sinus bradycardia, rate 52 and blood pressure (BP) 114/54 mm Hg. The student denies any health problems. Which action would the nurse take?
a. Approve the student to participate on the soccer team.
b. Refer the student to a cardiologist for further testing.
c. Tell the student to stop playing immediately if any dyspnea occurs.
d. Obtain more detailed information about the student‘s family health history.
a. Approve the student to participate on the soccer team.
Rationale: In an aerobically trained individual, sinus bradycardia is normal. The student‘s normal BP and negative health history indicate that there is no need for a cardiology referral or for more detailed information about the family‘s health history. Dyspnea during an aerobic activity such as soccer is normal.
Which finding from a newly admitted adult patient‘s electrocardiogram (ECG) requires further investigation by the nurse?
a. Isoelectric ST segment
b. PR interval of 0.18 second
c. QT interval of 0.38 second
d. QRS interval of 0.14 second
d. QRS interval of 0.14 second
Rationale: Because the normal QRS interval is less than 0.12 seconds, the patient‘s QRS interval of 0.14 seconds indicates that the conduction through the ventricular conduction system is prolonged. The PR interval and QT interval are within normal range and ST segment should be isoelectric (flat).
Which laboratory result for a patient with multifocal premature ventricular contractions (PVCs) is most important for the nurse to communicate to the health care provider?
a. Blood glucose of 243 mg/dL
b. Serum chloride of 92 mEq/L
c. Serum sodium of 134 mEq/L
d. Serum potassium of 2.9 mEq/L
d. Serum potassium of 2.9 mEq/L
Rationale: Hypokalemia increases the risk for ventricular dysrhythmias such as PVCs, ventricular tachycardia, and ventricular fibrillation. The health care provider will need to prescribe a potassium infusion to correct this abnormality. Although the other laboratory values are also abnormal, they are not likely to be the cause of the patient‘s PVCs and do not require immediate correction.
A patient‘s heart monitor shows a pattern of undulations of varying contours and amplitude with no measurable ECG pattern. The patient is unconscious, apneic, and pulseless. Which action would the nurse take first?
a. Give epinephrine (Adrenalin) IV.
b. Perform immediate defibrillation.
c. Prepare for endotracheal intubation.
d. Ventilate with a bag-valve-mask device.
b. Perform immediate defibrillation.
Rationale: The patient‘s rhythm and assessment indicate ventricular fibrillation and cardiac arrest; the initial action should be to defibrillate. If a defibrillator is not immediately available or is unsuccessful in converting the patient to a better rhythm, begin chest compressions. The other actions may also be appropriate but should not be done first.
A patient‘s heart monitor shows sinus rhythm, rate 64. The PR interval is 0.18 seconds at 1:00 AM, 0.22 seconds at 2:30 PM, and 0.28 seconds at 4:00 PM. Which action would the nurse take first?
a. Place the transcutaneous pacemaker pads on the patient.
b. Give atropine sulfate 1 mg IV per agency dysrhythmia protocol.
c. Hold the scheduled metoprolol (Lopressor) and call the health care provider.
d. Document the patient‘s rhythm and PR measurements in the medical record.
c. Hold the scheduled metoprolol (Lopressor) and call the health care provider.
Rationale: The patient has progressive first-degree atrioventricular (AV) block, and the -blocker should be held until discussing the drug with the health care provider. Documentation is appropriate later. The patient with first-degree AV block usually is asymptomatic; if the patient became symptomatic, a pacemaker or atropine may be used.
A patient develops sinus bradycardia at a rate of 32 beats/min, has a blood pressure (BP) of 80/42 mm Hg, and reports feeling faint. Which action would the nurse take?
a. Reposition the patient on the left side.
b. Have the patient perform the Valsalva maneuver.
c. Give the scheduled dose of diltiazem (Cardizem).
d. Apply the transcutaneous pacemaker (TCP) pads.
d. Apply the transcutaneous pacemaker (TCP) pads.
Rationale: The patient is experiencing symptomatic bradycardia and treatment with TCP is appropriate. Calcium channel blockers will further decrease the heart rate and the diltiazem should be held. The Valsalva maneuver will further decrease the rate. Repositioning on the left side may decrease cardiac output and blood pressure further.
A 19-year-old student comes to the student health center at the end of the semester stating, “My heart is skipping beats.” An electrocardiogram (ECG) shows occasional unifocal premature ventricular contractions (PVCs). What action would the nurse take next?
a. Insert an IV catheter for emergency use.
b. Start supplemental O 2 at 2 to 3 L/min via nasal cannula.
c. Ask the patient about current stress level and caffeine use.
d. Have the patient taken to the nearest emergency department (ED).
c. Ask the patient about current stress level and caffeine use.
Rationale: In a patient with a normal heart, occasional PVCs are a benign finding. The timing of the PVCs suggests stress or caffeine as possible etiologic factors. The patient is hemodynamically stable, so there is no indication that the patient needs supplemental O 2 , an IV, or to be seen in the ED.
The nurse has received change-of-shift report about the following patients on the progressive care unit. Which patient would the nurse see first?
a. A patient with atrial fibrillation, rate 88 and irregular, who has a dose of warfarin (Coumadin) due
b. A patient with second-degree atrioventricular (AV) block, type 1, rate 60, who is dizzy when ambulating
c. A patient who is in a sinus rhythm, rate 98 and regular, recovering from an elective cardioversion 2 hours ago
d. A patient whose implantable cardioverter-defibrillator (ICD) fired twice today and has a dose of amiodarone due
d. A patient whose implantable cardioverter-defibrillator (ICD) fired twice today and has a dose of amiodarone due
Rationale: The frequent firing of the ICD indicates that the patient‘s ventricles are very irritable. The priority is to assess the patient and give the amiodarone. The other patients can be seen after the amiodarone is given.
A patient on the telemetry unit develops atrial flutter, rate 150, with associated dyspnea and chest pain. Which action in the agency dysrhythmia protocol would the nurse take first?
a. Obtain a 12-lead electrocardiogram (ECG).
b. Notify the health care provider of the change in rhythm.
c. Give supplemental O 2 at 2 to 3 L/min via nasal cannula.
d. Assess the patient‘s blood pressure and discomfort level.
c. Give supplemental O 2 at 2 to 3 L/min via nasal cannula.
Rationale: Because this patient has dyspnea and chest pain in association with the new rhythm, the nurse‘s initial actions should be to address the patient‘s airway, breathing, and circulation (ABC) by starting with O 2 administration. The other actions are also important and should be implemented rapidly.
A patient is apneic and has no palpable pulses. The heart monitor shows sinus tachycardia, rate 132. Which action would the nurse take next?
a. Start cardiopulmonary resuscitation (CPR).
b. Perform synchronized cardioversion.
c. Give atropine per agency dysrhythmia protocol.
d. Apply supplemental O 2 via non-rebreather mask.
a. Start cardiopulmonary resuscitation (CPR).
Rationale: The patient‘s manifestations indicate pulseless electrical activity, and the nurse would immediately start CPR. The other actions would not benefit this patient.
Which action will the nurse include in the plan of care for a patient who was admitted with syncopal episodes of unknown origin?
a. Teach the patient to avoid caffeine and other stimulants.
b. Instruct the patient to call for help before getting out of bed.
c. Explain the association between dysrhythmias and syncope.
d. Tell the patient about implantable cardioverter-defibrillators.
b. Instruct the patient to call for help before getting out of bed.
Rationale: A patient with fainting episodes is at risk for falls. The nurse will plan to minimize the risk by having assistance whenever the patient is up. The other actions may be needed if dysrhythmias are found to be the cause of the patient‘s syncope but are not appropriate for syncope of unknown origin.
Which action by a registered nurse (RN) who is orienting to the telemetry unit indicates a good understanding of the treatment of heart dysrhythmias?
a. Prepares defibrillator settings at 360 joules for a patient whose monitor shows asystole.
b. Injects IV adenosine over 2 seconds for a patient with supraventricular tachycardia.
c. Turns the synchronizer switch to the “on” position before defibrillating a patient with ventricular fibrillation.
d. Gives the prescribed dose of diltiazem (Cardizem) to a patient with new-onset type II second-degree AV block.
b. Injects IV adenosine over 2 seconds for a patient with supraventricular tachycardia.
Rationale: Adenosine must be given over 1 to 2 seconds to be effective. The other actions indicate a need for more teaching about treatment of heart dysrhythmias. The RN should hold the diltiazem until discussing it with the health care provider. The treatment for asystole is immediate CPR. The synchronizer switch should be “off” when defibrillating.
p wave
atria contracts
depolarization
QRS complex
ventricles contract
depolarization
T wave
ventricles relax, get filled with blood
PR interval
Movement of electrical activity from the atria to the ventricles
ST segment
Time between ventricular depolarization and repolarization
(ventricular contraction)
QT interval
Time taken from ventricles to depolarize, contract, and repolarize
depolarization
contract, squeeze
electric
repolarization
relax
T/F
An asystole rhythm is shockable.
false
DO NOT SHOCK ASYSTOLE